Você está na página 1de 63

DATA INTERPRETATION REPLICA QUESTIONS THAT HAVE APPEARED IN

CAT IN THE LAST 4 YEARS


TABLES

1.

It was found that the volume of data transfer in


India is the same as that of Singapore.
Then which of the following statements is true?
(1) Total revenue is the same in both countries.
(2) Total revenue in India is about 2 times that of
Singapore.
(3) Total revenue in India is about 4 times that of
Singapore.
(4) Total revenue in Singapore is about 2 times
that of India.
(5) Total revenue in Singapore is about 4 times
that of India.

2.

It is expected that by 2010, revenue from data


transfer as a percentage of total revenue will
triple for India and double for Sweden. Assume
that in 2010, the total revenue in India is twice
that of Sweden and that the volume of data
transfer is the same in both the countries. What is
the percentage increase in ARDT in India if there
is no change in ARDT in Sweden?
(1) 400%
(2) 550%
(3) 800%
(4) 750%
(5) Cannot be determined

3.

If the total revenue received is the same for the


pairs of countries listed in the choices below,
choose the pair that has approximately the same
volume of data transfer.
(1) Philippines and Brazil
(2) South Korea and Poland
(3) Germany and USA
(4) UK and Spain
(5) Isreal and Norway

Directions for questions 1 to 3: Answer the following


questions based on the information given below.
Telecom operators get revenue from transfer of data
and voice. Average revenue received from transfer of
each unit of data is known as ARDT. In the table
below, the revenue received from data transfer as a
percentage of total revenue received and the ARDT
in US Dollars (USD) are given for various countries.

Country

ARDT
(in USD)

Japan
India
Brazil
Thailand
Israel
Hungary
Ireland
Russia
China
Indonesia
Philippines
UK
Spain
Sweden
Poland
Germany
South Korea
Norway
USA
Singapore

13
1
2
1
4
2
14
1
2
2
3
13
7
6
6
7
10
11
11
9

Revenue from data


transfer as a
percentage of total
revenue
70
9
8
11
13
15
23
18
25
42
54
30
14
18
22
24
24
20
17
21

Directions for questions 4 to 6: Answer the following questions based on the information given below.
For admissions to its postgraduate programme, six institutes conducted a common admission test. The test had
four sections with the maximum marks in each section being 50. The following tables gives the aggregate as well
as sectional cut-off marks fixed by the six institutes. A student will get interview calls only if he/she gets marks
higher than or equal to the cut-off marks in each of the sections and his aggregate marks are at least equal to the
aggregate cut-off marks as specified by the institute.

Institute 1
Institute 2
Institute 3
Institute 4
Institute 5
Institute 6
4.

5.

Section A
41

Sectional cut-off marks


Section B
Section C
41
41
44
44
45

42
44

Section D

44
42
40

Arun got calls from all colleges. What could be


the minimum aggregate marks obtained by him?
(1) 176
(2) 177
(3) 192
(4) 172
(5) 180
Bala got calls from two colleges. What could be
the minimum marks obtained by him in a section?

43
(1) 0
6.

Aggregate cut-off marks


172
171
167
174
176
172

(2) 17 (3) 21 (4) 31 (5) 37

Chandu did not get a call from even a single


college. What could be the maximum aggregate
marks obtained by him?
(1) 179
(2) 174
(3) 182
(4) 194
(5) 188

Directions for questions 7 to 9: Answer the following questions based on the information given below.
There are 100 employees in an organisation across five departments. The following table gives the departmentwise distribution of average age, average basic pay and allowances. The gross pay of an employee is the sum of
his/her basic pay and allowances.
Department
HR
Marketing
Finance
Business Development
Maintenance

Number of
Employees
5
3
20
35
10

Average age
(years)
46
36
31
43
36

Average basic
pay (Rs)
10000
12000
13000
15000
11000

Allowances
(% of Basic pay)
60
70
50
65
40

There are limited number of employees considered for transfer/promotion across departments. Whenever a
person is transferred/promoted from a department of lower average age to a department of higher average age,
he/she will get an additional allowance of 10% of basic pay over and above his/her current allowance. There will
not be any change in pay structure if a person is transferred/promoted from a department with higher average
age to a department with lower average age.
Questions below are independent of each other.
7.

What is the approximate percentage change in


the average gross pay of the HR department due
to transfer of a 40 year old person with basic pay
of `16000 from the marketing department?
(1) 9%
(2) 11%
(3) 13%
(4) 15%
(5) 17%

8.

There was a mutual transfer of an employee


between Marketing and Finance departments and
transfer of one employee from Marketing to HR.
As a result, the average age of Finance
department increased by one year and that of
Marketing department remained the same. What
is the new average age of the HR department?
(1) 31
(2) 36
(3) 41
(4) 46
(5) Cannot be determined

9.

If two employees (each with a basic pay of


`12000) are transferred from Maintenance
department to HR department and one person
(with a basic pay of `16000) was transferred from
Marketing department to HR department, what
will be the percentage change in average basic
pay of HR department?
(1) 10.5%
(2) 12.5%
(3) 15%
(4) 30%
(5) 40%

Directions for questions 10 to 13: Answer the


following questions based on the information given
below.
An athletics coach was trying to make an energy
drink which is best suited for the athletes. For this
purpose he took five of the best known energy drinks
A, B, C, D and E in the market with the idea of
mixing them to get the result he desired. The table
below gives the composition of these drinks. The cost
of each of these energy drinks per litre is A 150,
B 50, C 200 D 500 and E 100.

Composition
Energy
Carbohydrate
Protein Fat Minerals
drink
(%)
(%) (%)
(%)
A
50
30
10
10
B
80
20
0
0
C
10
30
50
10
D
5
50
40
5
E
45
50
0
5
10. For the sprinters, he has to prepare a drink
containing 10% minerals and at least 30% protein.
In how many different ways can we prepare this
drink by mixing at least two ingredients?
(1) One
(2) Two
(3) Three
(4) Four
(5) Five
11. Which among the following is the combination
having the lowest cost per unit for a drink having
10% fat and at least 30% proteins? The drink has
to be formed by mixing two ingredients together.
(1) B and C
(2) B and E
(3) B and D
(4) C and E
(5) D and E
12. In what proportion should B, C and E be mixed
to make a drink having at least 60% carbohydrate
at the lowest cost per unit?
(1) 2 : 1 : 3
(2) 4 : 1 : 2
(3) 2 : 1 : 4
(4) 3 : 1 : 2
(5) 4 : 1 : 1
13. A drink containing 30% each of carbohydrate and
protein, no more than 25% fat and at least 5%
minerals is to be made. Which of the following
two drinks must be mixed in equal quantities to
obtain the required drink?
(1) A and B
(2) D and E
(3) B and E
(4) C and D
(5) A and E

Directions for questions 14 to 17: Answer the following questions based on the information given below:
The following table shows the breakup of actual costs incurred by a company in the last five years (year 2004 to
year 2008) to produce a particular product.

Volume of production and sale (units)


Costs (Rs)
Material
Labour
Consumables
Rent of building
Rates and taxes
Repair and maintenance expenses
Operating cost of machines
Selling and marketing expenses

2004
1500

2005
1350

2006
1650

2007
1800

2008
1800

75000
30000
3000
1500
600
1200
45000
8625

67650
27000
3300
1500
600
1230
40500
8700

82800
33150
2700
1650
600
1170
50250
8700

89850
36225
2400
1650
600
1185
54030
8625

90000
36000
2100
1800
600
1200
54000
8700

The production capacity of the company is 3000 units. The selling price for the year 2008 was `125 per unit.
Some costs change almost in direct proportion to the change in the volume of production, while others do not
follow any obvious pattern to change with respect to the volume of production and hence are considered fixed.
Using the information provided for the year 2008 as the basis for projecting the figures for the year 2009, answer
the following questions.
14. What is the approximate cost per unit in rupees, if
the company produces and sells 2100 units in the
year 2009?
(1) 104
(2) 107
(3) 110
(4) 115
(5) 116
15. What is the minimum number of units that the
company needs to produce and sell to avoid any
loss?
(1) 470
(2) 525
(3) 576
(4) 1120
(5) 1392

How many units should the company produce to


maximize its profit?
(1) 2100
(2) 2400
(3) 2700
(4) 2850
(5) 3000
17. Given that the company cannot sell more than
2550 units, and it will have to reduce the price by
`5 for all units, if it wants to sell more than 2100
units what is the maximum profit, in rupees, that
the company can earn?
(1) 38100
(2) 36600
(3) 1800
(4) 1900
(5) 2000

16. If the company reduces the price by 5%, it can


produce and sell as many units as it desires.

Directions for questions 18 to 21: Answer the following questions based on the information given below.
The table below shows the comparative costs, in US Dollars, of certain items in USA and a select few Asian
countries. The models considered are the most popular ones in the respective countries.
Product
LCD TV
Home gym
Refrigerator
Air conditioner
Washing machine
Music system
Digital camera

Comparative Costs in USA and some Asian countries (in US Dollars)


USA India Thailand Singapore
Malaysia
2300 1000
1100
1850
900
1900
900
1000
1250
900
2000 1100
1300
1300
1100
2300
900
1200
1200
1000
1200
300
450
600
300
2000
850
1000
1300
800
1600
550
700
900
600

The equivalent of one US Dollar in the local currencies is given below.

India
Malaysia
Thailand
Singapore

I US Dollar equalent
40.93
Rupees
3.51
Ringits
32.90
Bahts
1.53
S Dollars

A consulting firm found that the quality of the products was not the same in all the countries above. A poor quality product
would result in higher servicing costs over its life time. The cost of poor quality of the products is given in the table

Products

Note:

Comparative cost of poor quality in USA and Asian


countries (in US Dollars)
USA India Thailand Singapore Malaysia

LCD TV

300

300

200

400

Home gym

500

400

500

500

Refrigerator

500

500

400

600

Air Conditioner

700

500

500

800

Washing machine

500

600

500

400

Music system

900

600

400

400

Digital camera

500

600

500

600

For all questions assume that the models considered are the ones for which the prices are mentioned.

18. A US citizen requires a refrigerator, air


conditioner and a music system. He can buy
them through an internet portal from any of the
given countries without having to pay for any
transportation costs. Which country will result in
the cheapest overall cost, taking cost of poor
quality into account?
(1) India
(2) Thailand
(3) Malaysia
(4) Singapore
(5) USA
19. Taking the cost of poor quality into account,
which country/countries will be the most
expensive if one has to buy a music system?
(1) India
(2) Thailand
(3) Malaysia
(4) Singapore
(5) India and Singapore
20. Approximately what difference in Bahts will it
make to a Thai citizen who is touring India if she
were to get a washing machine from India instead
of her native country, taking into account the cost
of poor quality? One has to pay a duty of 1500
Bahts for transporting the washing machine from
India to Thailand.
(1) 2350
(2) 4050
(3) 5150
(4) 6725
(5) 7500
21. The rupee values increases to `35 for a US
Dollar, and all other things including quality,
remain the same. What is the approximate
difference in cost, in US Dollars, between
Singapore and India for a digital camera, taking
this change into account?
(1) 70
(2) 250
(3) 450
(4) 800
(5) No difference

Directions for questions 22 to 26: Answer the


following questions based on the information given
below.
An all India tour operator connects ten cities A to J.
The table below gives the distance between a pair of
cities and the corresponding price charged by the
operator. Travel is permitted only from the city of
departure to the city of arrival. The customers do not
travel by a route when they have to stop at more than
two intermediate cities.

Sl. City of City of Distance between the


No Departure Arrival two cities. (in kms)
1
A
B
280
2
A
C
395
3
A
D
425
4
A
E
625
5
A
F
672
6
A
G
675
7
A
H
975
8
B
C
825
9
B
H
875
10
B
I
1050
11
B
J
1150
12
C
D
230
13
C
F
205
14
C
G
455
15
D
E
270
16
D
F
312
17
D
G
320
18
D
H
475
19
D
J
825
20
E
F
625
21
E
G
485
22
E
H
425
23
F
G
450
24
F
I
437
25
F
J
485
26
G
I
255
27
G
J
415
28
H
I
395
29
H
J
200
30
I
J
230

Fare
(Rs).
335
675
625
800
850
1225
925
1000
950
1225
1135
225
215
550
295
350
375
625
1225
850
575
435
525
475
575
275
445
485
210
270

22. What is the lowest price, in rupees, a passenger has


to pay for traveling by the shortest route from A to J?
(1) 1135
(2) 1425
(3) 1445
(4) 1465
(5) 1670
23. The operator plans to introduce a direct service
between cities A and J. The market research
results indicate that all its existing passengers
traveling between A and J will use the direct service
if it is priced 5% below the minimum price that they
pay at present. What should the operator charge,
approximately, in rupees, for the direct service?
(1) 995
(2) 1078
(3) 1349
(4) 1372
(5) 1392

24. If cities C, D and H are not accessible due to


security reasons, what would be the minimum
price, in rupees, to be paid by a passenger
traveling from A to J?
(1) 1140
(2) 1310
(3) 1425
(4) 1475
(5) 1545
25. If the prices include a margin of 10% over the
total cost that the operator incurs, what is the
minimum cost per kilometer that the operator
incurs for the service from A to J?
(1) 0.77
(2) 0.88
(3) 0.99
(4) 1.06
(5) 1.08
26. If the prices include a margin of 15% over the
total cost that the company incurs, which among
the following is the distance to be covered in
traveling from A to J that minimizes the total cost
per kilometer for the operator?
(1) 1085
(2) 1090
(3) 1160
(4) 1175
(5) 1195

Directions for questions 27 to 30: Answer the following


questions based on the information given below:
The proportion of male employees and the proportion
of post graduates in a company are given below.
The company has a total of 800 employees, 80% of
whom are in the production department and the rest
equally divided between the marketing and the
accounts department.
Department
Marketing
Accounts
Production
Total

Male
0.60
0.55
0.475

Post graduates

27. What is the percentage of male employees in the


production department?
(1) 40
(2) 45
(3) 50
(4) 55
(5) 60
28. In the marketing department, twenty five per cent
of the post graduates are male. What is the
difference between the number of female post
graduates and male employees who are not post
graduates?
(1) less than 8 (2) 10
(3) 12
(4) 14
(5) 16
29. What percentage of employees in the marketing
department are post graduates?
(1) 40
(2) 45
(3) 50
(4) 55
(5) 60
30. In the production department, 50% of the males
are post graduates. Which of the following
statements is correct?
(1) Except post graduate males, all other groups
have the same number of employees.
(2) Except males who are not post graduates, all
other groups have the same number of
employees.
(3) Except post graduates females, all other
groups have the same number of employees.
(4) Except females who are not post graduates,
all the other groups have the same number of
employees.
(5) All of the above groups have the same
number of employees.

0.50
0.55
0.53

Directions for questions 31 to 33: Buziki motors, a two wheeler manufacturer, introduced a variant in the 125cc
category in the beginning of 2009. The number of bookings received in a city for a period 12 months is as given
below.
Month
Number of bookings
Number of deliveries

Jan
346

Q1
Feb March
412
380
200
362

April
450
356

Q2
May
308
445

June
359
395

July
462
297

Q3
Aug
333
496

Sep
345
300

Oct
250
249

Q4
Nov
506
420

Dec
370
403

The company promises the delivery of the vehicles within a span of two months i.e., a booking made in January
will be delivered in February or March.
The entire period of 12 months is divided into 4 quarters. The price of the motorcycle was increased every
quarter. The customer had to pay the total amount at the time of the booking.
Quarter
Q1
Q2
Q3
Q4

Price (in Rs) per motor cycle


42000
42500
43100
44000

31. In which quarter was the average number of


bookings per month, the highest?
(A) Q1
(B) Q2
(C) Q3
(D) Q4
32. The number of deliveries made in December from
the bookings made in November is how many
times the number of deliveries made in August
from the bookings made in June?
(A) 1.39
(B) 1.58
(C) 2.38
(D) 2.58

33. What is the highest revenue (in `crore) obtained


by the company from the sales of the new 125cc
variant in any of the four quarters?
(A) 4.91
(B) 4.95
(C) 4.99
(D) 5.05

Directions for questions 34 to 36: The following


table gives the relation between the scaled scores of
three teams and the number of wins in the matches
played.

Percentage of investments in the seven companies


under the different schemes are as follows:

Number of
wins
10 12
13 15
16 18
19 21
22 24
25 27
28 30
31 33

GB Holdings
Solar Computers
NLP Industries
Techies
Technologies
Wiz craft solutions
OBCC bank
Live life mutual funds

Team A
18
22
28
30
41
68
62
94

Scaled score
Team B
19
22
36
27
51
64
51
99

Team C
20
24
36
39
46
63
52
95

34. If the scaled score for two of the teams is the


same and greater than that of the third team, then
which of the following could be the range for the
number of wins?
(A) 13 15
(B) 16 18
(C) > 16
(D) Either (A) or (C)

Companies

Scheme Scheme Scheme


X
Y
Z
5%
20%
15%
12%
2.5%
10%
16%
12.5%
7.5%
2
%
3

10%

30%

16

25%
15%

10%
10%

15%
22.5%

17%

15%

13

1
%
3

The brokerage firm promises a rate of return of 2% on


scheme X, 2.5% on Scheme Y and 3% on Scheme Z.
Assume that the rate of return on each of the
companies in a scheme is same as the overall rate of
return of the scheme.

35. For which of the following range for the number


of wins is the difference between the scaled
scores of team A and team B as a percentage of
rd
that of team C, the 3 least?
(A) 19 21
(B) 10 12
(C) 31 33
(D) None of these

37. If Mahesh invested `12 lakh in the stock


brokerage farm, but he immediately withdrew an
amount of `3 lakh, then what is the percentage
change in the investment in NLP Industries
because of the withdrawal?
(A) 2%
(B) 2.5% (C) 3%
(D) 4%

36. If average scaled score of the three teams is 2.5


times the average of the corresponding number
of wins, Then which of the following could be the
range for the number of wins.
(A) 25 27
(B) 31 33
(C) 22 24
(D) All the three

38. Mr. Anil and Ms. Shivani invests an amount of


`7 lakh and `13 lakh respectively. What is the
difference (in `) in their returns on investment
made in Wizcraft solutions?
(A) 250
(B) 300
(C) 400
(D) More than 500

Directions for questions 37 to 39: A stock


brokerage firm accepts the investments and places
the amounts invested in seven different companies
under three schemes. Scheme X is applicable for an
investment of 5 to 10 lakh rupees, scheme Y is
applicable for an investment of 11 to 20 lakh rupees
and scheme Z is applicable for an investment of 21 to
40 lakh rupees.

39. If three persons A, B and C make investments in


the ratio of 10 : 20 : 21, such that their
investments fall under the schemes X,Y and Z
respectively, then what is the increase in their
combined return on investment (in `) if the firm
increases the rate of return on the schemes X, Y
and Z by 10%, 20%, and 10% respectively?
(A) 16100
(B) 16300
(C) 17300
(D) None of these

Directions for questions 40 and 41: The following table gives the numbers of music payers, of two companies NOSY and BOSS, sold in Delhi across three years.
Type of music system
Mono speaker
Dual speaker 1000w
Dual speaker 2000w
Four speaker 5000w
Home theatre
Total

2001
NOSY
BOSS
1000
1600
1800
2000
2300
1200
1400
2200
1600
2400
8100
9400

40. From 2001 to 2003, for how many types of music


systems is there an increase in the percentage
contribution for each of the two companies?
(A) 0
(B) 1
(C) 2
(D) More than 2
41. From 2001 to 2003, which type of music system
has shown the maximum change in percentage

2002
NOSY
BOSS
3000
2700
3100
2500
2900
3200
3000
4200
3200
2400
15200
15000

2003
NOSY
BOSS
2600
4000
4400
3400
3600
4200
4600
3800
4000
5000
19200
20400

points in its contribution to the total sales of the


company NOSY?
(A) Mono speaker
(B) Dual speaker 1000w
(C) Dual speaker 2000w
(D) Four speaker 5000w

Directions for question 42: The following table


gives the break up of the number of cars sold by
two showrooms in the first 10 days of their opening.
Day
1
2
3
4
5
6
7
8
9
10

Showroom A
16
20
35
30
33
24
51
63
60
79

Showroom B
18
19
26
42
39
29
48
52
71
81

42. Which of the following statement (s) is/are true?


. The total number of cars sold by showroom A
at the end of 7 days lies between 90% and
110% of that sold by showroom B.
. In the given period, the total number of cars
sold by showroom A on odd numbered days
is less than 90% of that sold by showroom B
on even numbered days.
(A) Only
(B) Only .
(C) Both and
(D) Neither nor .

Directions for questions 43 and 44: The following data gives the details of the establishment fee and average
number of customers estimated (per year) in franchises of a restaurant chain, Foodies, in class A and class B
centers of different cities in India.

City

Est. fee in class A


centers (in ` lakh)

Est. fee in class B


centers (in ` lakh)

Estimated customers per


year in class A centers

Hyderabad

126

75

51,860

Estimated customers
per year in class B
centers
42,500

Bengaluru

144

90

60.200

50.246

Pune

132

104

70,000

52,400

Chennai

125

95

48,800

40,000

Kolkata

115

65

45,500

37,000

43. If each customer spends an average amount of


`240 in restaurants in class A centers and `180
in class B centers, then in which of the given
cities will the franchise earn revenues more than
the establishment fees (for each type of centre),
in one year?
(A) Hyderabad
(B) Pune
(C) Chennai
(D) Bengaluru
44. If a person owns two franchises of Foodies in
Pune, one in a class A center and the other in a
class B center, and it is estimated that the
average amount each person spends in class A
and class B centers are `300 and `130
respectively, then find the minimum number of
customers, who are required to come to the two
restaurants together in the first year, such that
the revenues are not less than the establishment
fees for each?
(A) 12.4 lakh
(B) 1.24 lakh
(C) 2.4 lakh
(D) 9.6 lakh

Directions for questions 45 and 46: The number of


dropouts from primary schools, as a percentage of
the total enrollments in a year in five districts across
six years are given below. The values represented by
are unknown.
Districts
P
Q
R
S
T

2001
52.3
52.4
45.9
36.5
41.2

2002
51.0
53.2
46.2

43.4

2003
45.6
54.1

37.4
42.6

2004
43.9
57.3
44.5

44.5

2005
42.0
61.3
43.0
38.2
44.1

2006
40.8
61.1

39.6
45.0

45. Which of the following is definitely false?


(A) For more than one of the five districts the
percentages are continuously increasing or
decreasing.
(B) In the given period, the number of dropouts is
the highest in 2006 for at least two districts
(C) The total number of dropouts as a
percentage of total number of enrolments, in
the given period, is not the highest for Q if it
is given that the districts R and S have
registered highest percentages in 2002 and
2006 respectively.
(D) In the given period, the maximum number of
dropouts in all the five districts combined was
registered in 2006, if it is given that R has
registered its maximum percentage in 2006
and in any given year each of the
five districts have equal number of
enrolments in primary schools.
46. If the percentage of dropouts in a year has
decreased with respect to that in the previous
year, than it is considered as an achievement for
a district. What is the minimum number of such
achievements, in the given period, for all the
five districts combined?
(A) 5
(B) 10
(C) 11
(D) More than 12

Directions for questions 47 and 48: The following


table gives the temperatures in six cities at three
different times of the day.
City
P
Q
R
S
T
U

5.00 a.m.
24
25
25
32
30
22

12 noon
42
46
44
49
46
44

52. What is the least difference between the traffic


flowing from a particular city and traffic flowing to
the same city?
(A) 120
(B) 158
(C) 98
(D) 232

6.00 p.m.
29
28
32
31
30
26

47. If the increase in temperature from 5 a.m. to


12 noon is linear, then what is the temperature in
city Q at 10 a.m.?
(A) 30C (B) 32C (C) 36C (D) 40C
48. If the decrease in temperature from 12 noon to
6.00 p.m. is linear, then in which city was the
temperature the highest at 3.30 p.m.?
(A) Q
(B) R
(C) S
(D) U

Directions for questions 49 to 52: The following


table gives the details about the traffic flow on a
particular day through the roads connecting
six different cities. For example, 846 vehicles travel
from P to Q and 964 travel from Q to P. So, total
traffic on the road connecting the cities A and B is
846 + 964 = 1810. Assume that these are the only
cities interconnected and traffic flow is among these
cities only.
P
Q
R
S
T
U

964
536
848
484
672

Q
846

664
242
364
496

R
808
564

624
784
528

S
400
540
248

672
992

T
472
840
888
478

372

51. Traffic flowing from which of the given cities is the


maximum?
(A) P
(B) Q
(C) T
(D) U

U
820
844
200
428
648

49. The maximum traffic flow occurs on the road


connecting which two cities?
(A) P Q
(B) P R
(C) T R
(D) None of these
50. The total number of vehicles passing through the
road connecting any two of the given cities is the
second least for
(A) S U
(B) R S
(C) P T
(D) None of these

Directions for questions 53 and 54: The following


table gives the different payments options for a
`1 lakh loan provided by a rural bank to persons with
different income levels, depending on the time in
which they would repay the loan.
Income level
(in 000 Rs)

Payment options
(interest amount in `)
1 year
2 years
3 years

20 40

2,000

3,000

4,000

41 60

2200

3,600

4,100

61 80

2,600

3,800

4,400

81 100

2,900

4,000

4,800

A person with an income between `41,000 to


`60,000 has to repay the loan of `1 lakh with an
interest amount of `2,200 if the loan is repaid in
1 year, `3,600 if the loan repaid in 2 years and so on.
The interest amount to be paid gets multiplied by the
factor of loan amount (for example a loan of `3.4 lakh
lent to a person with an income between `20,000 to
`40,000, in one year, accumulates to an amount (in `) of
3,40,000 + 3.4 2,000 = 3,46,800.)
53. Mr. A, whose annual income is `45,000 takes a
loan of `2.2 lakh and promises to repay it in
2 years. Mr. B, whose annual income is `76,000
takes a loan of `3.6 lakh and promises to repay it
in 2 years. What is the difference in the interest
amounts (in `) they have to pay?
(A) 5,440 (B) 5,760 (C) 6,120 (D) 6.260
54. A person with an annual income of `96,000 takes
two loans-one of `5.6 lakhs for 3 years and
another of `6.4 lakh for 2 years. What is the
average of the interest amount paid by him?
(A) `24,840
(B) `26,000
(C) `26,240
(D) `28,140

Directions for questions 55 and 56: The following table gives the distribution of number of male employees and
female employees owning a four wheeler or a two wheeler or both in three companies X, Y and Z.
Company
X
Y
Z

Male employees
Female employees
Male employees
Female employees
Male employees
Female employees

Four Wheeler
45%
30%
50%
25%
36%
24%

Two wheeler
65%
70%
60%
80%
54%
67%

Neither
5%
10%
20%
15%
20%
9%

The number of male employees in company X is 70% of the total number of employees in the company and the
number of female employees in company Y and Z is 40% each of the total number of employees of their
respective companies.

55. In which company is the percentage of


employees who own both two wheeler and four
wheeler, the highest?
(A) X
(B) Y
(C) Z
(D) Cannot be determined
56. What percentage of the male employees in
companies Y and Z together own either a four
wheeler or a two wheeler but not both, if it is
given that the total number of employees in both
the companies is the same?
(A) 40%
(B) 50%
(C) 80%
(D) None of these

Directions for questions 57 and 58: The following


table gives the details about the percentage
distribution of the total bikes sold by ACE Motors Ltd.
The percentage distribution was the same in 2007
and 2008. The total number of bikes sold in 2007 is
1,50,000 which is the same as that in 2008.
Model
RL-100
BCZ
Thunder
WB-150
Muzzle

Percentage of total bikes sold


13%
25%
20%
30%
12%

The table below shows the revenue of the company


from each of the five models in both the years.

Model
RL-100
BCZ

Revenue
(in ` crore)
In 2007
78

Revenue
(in ` crore)
In 2008
87.75

93.75

105

Thunder

93

105

WB-150

90

103.5

Muzzle

93.6

99

57. For which model is the percentage increase in


the average selling price per bike, the highest?
(A) RL-100
(B) Thunder
(C) WB-150
(D) Muzzle.
58. What is the average of the selling prices (in `) of
the five models in 2007?
(A) 37,600
(B) 33,600
(C) 32,400
(D) None of these.

Directions for questions 59 and 60: The following table gives the performance of four companies, all listed on
National Stock Exchange (NSE), from 2001 to 2010.
Company
Company
Company
Company V
Share price Dividend Share price Dividend Share price Dividend Share price Dividend
Year
(in `)
(in `)
(in `)
(in `)
(in `)
(in `)
(in `)
(in `)
2001
128
120
283
112
148
128
400
128
2002

132

116

289

115

152

132

420

124

2003

126

123

295

128

163

145

432

136

2004

148

152

296

138

168

148

440

144

2005

158

148

312

142

172

140

453

148

2006

123

121

328

154

184

152

448

132

2007

172

113

324

132

196

128

432

120

2008

164

109

345

106

212

136

464

112

2009

128

105

360

121

252

138

484

128

2010

132

102

364

143

286

140

496

132

For any company,


Px + Px + 1 2Px 1

Gx = Dx +

Where Gx is gain an company;s share in yea x.


Dx is the divided declared by the company in year x.
Px and Px + 1 are share prices of the company in year x and x + c respectively.
59. What was the gain from the shares of company
IV in 2006?
(1) 116
(2) 132
(3) 148
(4) 138

60. What was the highest percentage increase in the


gain from the shares of company III in a year with
respect to the previous year?
(1) 13.48%
(2) 29.38%
(3) 16.32%
(4) 38.45%

61. The following table gives the per capita income of


countries in the year 2004. From the table
determine the number of countries having their
per capita income more than 40% of the median
per capita income of these countries?
(1) 9
(2) 10
(3) 11
(4) 8
Per capita income (gross) in US $
Switzerland
24,369
New Zealand
15,350
Lithuania
4,965
Romania
2,916
Spain
11,692
Sweden
13,746
United States
23,484
France
13,477
Mexico
3,523
Hong Kong
10,372
United Kingdom
19,207
Brazil
5,663
Germany
24,337

Directions for questions 64 and 65: The following


table gives the gender, height, weight and age of
fifteen students of a college who have cleared the
preliminary round of the selection process for being
selected in the Air Force. The names of the students
being denoted by A, B, C, D, . . . . . and O.

Directions for question 62: Bolvo bus service has


the following revenue data (in ` crore) regarding its
operations in 2007.
Non A/c
Non A/C
semiTotal
general
sleeper

A/c
A/c semi
sleeper sleeper
Inter-state
services
Intra-state
services
Total
1200

240

What percent of the coal production of the top


four (i.e, the four highest) countries is the coal
production of the bottom four (i.e. the four lowest)
countries?
(1) 10.66%
(2) 8.98%
(3) 11.32%
(4) 12.45%

2880

A/c sleeper and A/C semi-sleeper accounted for


37.5% of the total revenue whereas non A/c semisleeper accounted for 25% of the total revenue. 50%
of Intra-state services revenue was generated from
non A/c general services.
60% of the total revenue generated was from Intrastate services. The revenue generated from A/c
sleeper in inter-state services to that in intra-state
services was in the ratio of 1 : 2
62. What was the total revenue (in ` crore) generated
from non A/c general in Intra-state services?
(1) 600
(2) 1440 (3) 950
(4) 600

Names

Gender

A
B
C
D
E
F
G
H
I
J
K
L
M
N
O

M
M
F
M
F
M
M
M
F
M
M
F
F
M
F

1
2
3
4
5
6
7
8
9
10
11
12
13
14
15

Height
(in cm)
164
169
152
148
154
172
168
165
145
152
156
146
138
171
162

Weight
(in kg)
64
62
49
68
78
68
67
70
58
78
64
51
56
67
60

Age
(in years)
22
21
17
18
21
23
27
22
20
18
18
19
23
24
25

Air Force follows a selection criterion where the


height and weight of a person must lie in one of the
following ranges:
Male
HEIGHT (in cm)
155 160
160 165
165 170
170 175

WEIGHT (in kg)


58 62
6064
6368
6674

Candidates whose height is less than 155 cm or more


than 175 cm, are not eligible for selection.

Directions for question 63:


Female
63. The following table gives list of the major coal
producing countries in the world along with the
total production of coal in the year 2007(in million
tonnes)
Country
China
Russia
Poland
Ukraine
India
Turkey
United States
Australia
Germany
Indonesia

Coal production
2536.7
314.2
145.8
76.3
478.2
76.6
1039.2
393.9
201.9
174.9

HEIGHT (in cm)


145 150
150 155
155 160
160 165

WEIGHT (in
kg)
48 52
52 56
55 59
58 62

Candidates with weight less than 145 cm or more


than 165 cm are not eligible for selection.
64. Find the ratio of the number of male and female
students who were eligible for selection.
(1) 2 : 1
(2) 5 : 2
(3) 3 : 2
(4) 4 : 1

10

65. If x represents the number of male students


whose age lies in the range of 18 to 22 years
(both inclusive) and y represents the number of
females students whose weight lies in the range
of 49 to 58 kg (both inclusive) then
(1) x > 2y
(2) x < y
(3) x = y
(4) 2x = 3y

Directions for questions 66 and 67: The following


table gives the cost and revenue of a company for a
period of five years from 2005 to 2009. The total cost
is the sum of the costs under three heads H1, H2 and
H3. Operating Expense of the company for each year
is equal to 20% (H1) + 25% (H2) + 30% (H3) the
profitability of the company in each year is defined as
Operating Expense
. Study the given table carefully
Re venue
and answer the questions that follow:
All values are given in '000s of `.
Year
2009
2008
2007
2006
2005

H1
20.8
21.2
29.6
30.8
24.8

H2
30.6
24.3
38.4
23.4
42.8

H3
40.8
38.2
21.6
42.4
36.4

Revenue
104.2
96.6
112.4
128.2
130.6

66. In which of the following years was the


profitability of the company the least?
(1) 2005
(2) 2006
(3) 2007
(4) 2008
67. What was the maximum percentage decrease in
the total cost of the company in a given year with
respect to the previous year?
(1) 7.04%
(2) 7.24%
(3) 7.11%
(4) 8.92%

Directions for questions 68 and 69: The following


table gives the distribution of the number of students
based on the marks obtained in a certain examination
for five sections A, B, C, D and E. Students were
categorised as per their marks being 'less than 45',
from 45 to 85 and greater than 85.
M is the marks obtained.
Sections

M < 45

A
B
C
D
E

28
15
18
29
30

45 < M 85
72
68
52
58
60

M > 85
24
36
28
47
35

68. What percentage of the total number of students


got scores less than 45?
(1) 15.6% (2) 18.8% (3) 21.2% (4) 20%
69. If the qualifying mark in the paper is 48, then the
maximum number of students from a section who
passed in that examination was
(1) 96
(2) 104
(3) 105
(4) 95

Directions for questions 70 to 72: The following


table gives the exports and imports (values given in
millions of `) of the 4 companies in the years 2002
03, 2003 04, 2004 05
Company

Year

2002 - 03
2003 - 04
2004 - 05
2002 - 03
Chandu
2003 - 04
& co
2004 - 05
2002 - 03
Shiva
2003 - 04
& co
2004 - 05
2002 - 03
Kanta
2003 - 04
& co
2004 - 05
Rahul
& co

Exports Imports
7.13
12.15
15.3
12.4
14.1
16.2
5.4
9.3
12.1
6.54
10.41
13.73

5.14
11.67
17.41
11.61
16.31
17.1
8.72
9.39
13.17
7.46
11.51
14.33

Total
trade
12.27
23.82
32.71
24.01
30.31
33.3
14.12
18.69
25.27
14.00
21.92
28.06

70. Which of the following company registered the


highest percentage growth in exports from 2003
04 to 2004 05?
(1) Rahul & co.
(2) Chandu & co.
(3) Shiva & co.
(4) Kanta & co.
71. Which company registered the least growth rate
in imports from 2002 03 to 2003 04?
(1) Rahul & co.
(2) Chandu & co.
(3) Shiva & co.
(4) Kanta & co.
72. Which company had the highest trade-deficit (=
imports exports) in 2004-05?
(1) Rahul & co.
(2) Chandu & co.
(3) Shiva & co.
(4) Kanta & co.

Directions for questions 73 and 74: Five


companies held examinations for all the employees
who are in their probation period.
The following table gives the details of all the
employees who have taken the exam
No. of
% of employees
No. of employees
employees
who got more
who wrote the
Company who
than 90% of
exam
crossed
marks
the cut off
A
180
10
300
B
225
8
450
C
150
12
250
D
400
16
600
E
300
20
575
73. If the employees who do not clear the cut offs are
rejected, then which company rejected the
maximum number of employees?
(1) B
(2) C
(3) D
(4) E
74. What is percentage of employees who got more
than 90% of marks out of the total number of
employees who cleared the cut off for all
companies combined (approximately)?
(1) 20%
(2) 22%
(3) 24%
(4) 28%

11

Directions for questions 75 and 76: The following


table gives the sales of 5 companies in 2008 and
2009

2008 Company

Price/unit
(in `)

No. of units
products
(in 1000)

Closing
stock

8
6
5
10
11

12
14
9
11
13

500
750
675
890
1200

P
Q
R
S
T

2009

P
Q
R
S
T

7
4
8
6
9

11
15
10
9
12

485
690
775
1245
865

75. Which company had the least sales in the year


2009?
(1) P
(2) S
(3) Q
(4) T
76. In which year did R have lower sales?
(1) 2008
(2) 2009
(3) Both 2008 and 2009
(4) None of these

Directions for questions 77 and78: The following table gives the number of members in seven families and the
details regarding their income and expenses.

No. of members

Average income of the


family in (`)

Expenses of the
family in (`)

Overhead
expenses in (`)

Kapoor family

24500

9000

3000

Khanna family

21000

11000

2500

Kirsten family

24000

14000

3750

Kumble family

35000

12500

4250

Khan family

27500

13000

6000

Kittu family
Kala family

3
7

40000
28000

14200
17000

3250
4375

77. What is the total savings made by all the families


(in `)?
(1) 913175
(2) 923175
(3) 923075
(4) 921075

78. If the average income of the Khan family


increased by 2% where as the expenses of the
family decreased by 3%, then the savings of the
Khan family would increase by
(1) `3370
(2) `3570
(3) `3870
(4) `3670

Directions for question 79: The following table gives the diet statistics of 10 Drinks named P to Y. The values
are in percentages.
Drinks
P
Q
R
S
T
U
V
W
X
Y

Proteins
16%
21%
17%
23%
18%
25%
24%
16%
27%
19%

Vitamins
24%
18%
25%
26%
29%
21%
22%
29%
24%
25%

Carbohydrat
12%
18%
20%
20%
19%
16%
14%
15%
14%
18%

Fats
27%
14%
24%
16%
18%
15%
19%
17%
18%
14%

Sugar content
21%
29%
14%
15%
16%
23%
21%
23%
17%
24%

A healthy drink is considered to have at-least 20% of proteins, at-least 23% of vitamins and at-most 20% of other
ingredients. Otherwise, it is considered as an unhealthy drink.
79. What is the ratio of healthy drinks to unhealthy drinks in the given group?
(1) 2 : 3
(2) 3 : 7
(3) 1 : 4
(4) 1 : 9

12

Directions for questions 80 and 81: The following


table shows the number of students in government
schools in six different states of India during 2007,
2008 and 2009.

Directions for questions 82 and 83: The following


table gives the number of students in different
sections A, B, C, D, E and F of a school in 2007

Students in Government schools (in 000)


State
Andhra Pradesh
M.P
U.P
Karnataka
Kerala
Tamil Nadu

2007
15.4
21.2
20.1
18.7
16.3
14

2008
17.2
19.6
21.4
17.3
18.5
19.2

Section

Students

New students

A
B
C
D
E

18
12
20
17
14

12
4
8
10
11

19

2009
16.1
20.9
22.1
19.6
17.9
20.3

82. With respect to the number of students in each


section, how many sections have the number of
students more than the median of the number of
students in a section for the given sections?
(1) 2
(2) 3
(3) 4
(4) 1

80. In 2008, which state experienced the maximum


increase in the number of students studying in
government schools with respect to that in the
previous year?
(1) Andhra Pradesh
(2) U.P
(3) Kerala
(4) Tamil Nadu

83. The student who fails in any class is retained in the


same class and section. Which of the following
sections have the highest number of failed
students?
(1) A
(2) B
(3) C
(4) D

81. Which state has shown a consistent increase in


the number of students studying in government
schools from 2007 to 2009?
(1) M.P
(2) U.P
(3) Kerala
(4) Karnataka

Directions for questions 84 and 85: The following table gives the details of the area utilized and the production
of wheat by 3 countries P, Q and R from 2003 to 2008.
P
Area
2.1
2.6
1.8
1.9
2.3
3.4

2003
2004
2005
2006
2007
2008
Yield return =

Q
Area
1.7
2.2
1.5
2.4
2.5
1.9

Production
5280
1380
2790
5550
5950
5180

R
Area
3.5
2.7
2.6
2.3
2.1
3.2

Production
2850
4850
3950
4800
6800
7800

Production
5450
4250
4900
4650
3350
4880

Pr oduction
Unit area

84. Which of the following is the highest yield return


obtained by country R?
(1) 2021 (2) 1885 (3) 2011 (4) 1914

85. In which year is the percentage increase in the


yield return the highest for country Q?
(1) 2005 (2) 2006 (3) 2007 (4) 2008

Directions for questions 86 and 87: The following graph shows the average marks obtained by the students
and the number of students.
100
90

Average Marks

80

70
60

50
40

30
20

10
50

100

150

200

250

300

350

400

450

Number of Students (in hundreds)

13

The table given below shows the average marks


obtained and the number of students in each class.
Class
I
II
III
IV
V
VI
VII
VIII
IX
X

Average
marks
45
60
31
17
57
37
83
71
62
79

Number of students
(in hundreds)
450
325
120
180
220
110
180
240
305
400

88.

In which state are the number of students the


third highest this year?
(1) Assam
(2) MP
(3) Orissa
(4) AP

Directions for question 89: Select the correct


alternative from the given choices.
89. Lakshmi, an employee, wants to invest in 3 types
of business X, Y and Z. The following table gives
the investment and revenue obtained by Lakshmi
from her investments

86. The statement "the higher the average marks, the


higher the number of students" is true in which of
the following classes?
(1) II, VIII, X
(2) III, V, VII
(3) II, IX, X
(4) I, IV, VI

Name
of
the
X
Y
Z
busine
ss
Investments
(in
16.2 14.5 12.9
lakhs)
Revenue (in
21.2 18.4 16.5
lakhs)

87. The statement "the lower the number of the


students higher is the average marks" is true for
which of the following classes?
(1) VII, VIII, V
(2) I, X, VI
(3) I, II, V
(4) V, VII, IX

Which of the following would be the most


profitable investment for Lakshmi if she spends
20% of the revenue earned from each business
to maintain her house?
(1) X
(2) Y
(3) Z
(4) Both (1) and (3)

Directions for question 88: The following table


gives the details of the number of students in 6 states
No.of students Change in this year
(in lakhs)
(in '000)
AP
13
+21
UP
17
+46
MP
16
210
Bihar
18
+114
Assam
19
612
Orissa
12
112

Directions for questions 90 and 91: The table given below gives the details of income and expenditure for
some states in different regions in 2006 and 2007
Region

Per capita income Per capita expenditure


(in ` crore)
(in ` crore)

2006

2007

2006

2007

2006

2007

2006

2007

North
J&K

16.2

17.3

18.1

17.6

250

265

215

221

Punjab

15.1

16.5

15.5

16.9

261

272

218

224

South
AP

18.3

17.5

18.9

17.9

271

281

224

236

Karnataka

14.6

16.3

15.1

17.3

283

294

241

249

East
West Bengal

17.5

18.4

17.8

18.9

241

261

213

232

12.1

13.5

12.7

13.8

264

270

222

234

19.4

20.2

19.8

20.9

256

271

233

241

19.6

19.9

19.9

20.4

245

259

219

232

Assam
West
Gujarat
Maharashtra
90.

Income (in ` crore) Expenditure (in ` crore)

Which of the following regions had the ratio of income to expenditure in 2007 the highest?
(1) North
(2) South
(3) East
(4) West

14

91.

What is ratio of the number of states in which per capita income increased by more than 5% to the number of
states in which per capita income did not increase by more than 5%?
(1) 1 : 7
(2) 1 : 1
(3) 3 : 5
(4) 1 : 3
BAR GRAPH

Directions for questions 1 to 4: Answer the following questions based on the information given below.

Revenue in million USD

The bar chart below shows the revenue, in million US Dollars(USD), of a cosmetics company. The data covers
the period 2003 to 2007 for the United States(US) and Asia. The bar chart also shows the estimated revenues of
the company for the period 2008 to 2010.
500
450
400
350
300
250
200
150
100
50
0
2003

2004

2005

2006
2007
Year

US
1.

The difference between the estimated revenue in


Asia in 2008 and what it would have been if it
were computed using the percentage growth rate
of 2007(over 2006) is closest to
(1) 25 (2) 40 (3) 10 (4) 5 (5) 0

2.

In 2003, sixty percent of the people who used the


companys products in Asia were men. Given that
women who used the companys products
increase at the rate of 10 percent per annum and
men at the rate of 5 percent per annum, what is
the approximate percentage growth of customers
between 2003 and 2010 in Asia? The prices of
the companys products are volatile and may
change each year.
(1) 62 (2) 15 (3) 78 (4) 84 (5) 50

2008

2009

2010

Asia
3.

Consider the annual percent change in the gap


between revenues in the US and Asia. What is
the year in which the absolute value of this
change is the highest?
(1) 30-04
(2) 05-06
(3) 06-07
(4) 08-09
(5) 09-10

4.

While the revenues from Asia has been growing


steadly towards that of the US, the growth rate in
Asia seems to be declining. Which of the
following is closest to the percent change in
growth of 2007 (over 2006) relative to the growth
rate of 2005 (over 2004)?
(1) 17 (2) 10 (3) 35 (4) 60 (5) 100

Directions for questions 5 and 6: The first part of the bar graph gives the number of animals of a given species
in the Amazon forest as a percentage of the total number of animals of that species in South America and the
second part of the bar graph gives the number of animals of that species in South America as a percentage of
the total number of animals of that species in the world. The total number of deers and wild bisons in the world
are 24,000 and 18,000 respectively. The total number of animals of the five given species in South America is
25,800.
95%
80%
80%
75%
70%
42%

40%
25%

Pythons

Deers

30%

Wild bisons

27%

Wolves

Bears

15

5. If the number of Pythons and Bears in South


America are 4800 and 4200 respectively, then
what is the number of Wolves in the Amazon
forest?
(A) 3,990
(B) 4,050
(C) 4,200
(D) 4,320

6. Using the information in the previous question,


arrange the animals in the decreasing order of
their number in the Amazon Forest?
(A) Deers, Wild bisons, Deers, Wolves, Bears,
Pythons.
(B) Wild bison, Deers, Wolves, Bears, Pythons.
(C) Deers, Wolves, Wild bisons, Bears, Pythons.
(D) Deers, Wolves, Wild bisons, Pythons, Bears.

3000

2006

2007

4500

4200

5000

5600
4500

5400

4800

5400

3500

2800

4000

4350

5000

3000

4200

4480

6000

4850

5800

Directions for questions 7 to 9: The following graphs shows the sales (by volume) of four PC manufacturing
companies P, Q, R and S, in Hyderabad across four years.

2000
1000
0

2008

R
(A) 6.61

7. The sales volume of which company increased


by the highest percentage from 2006 to 2008?
(A) P
(B) Q
(C) R
(D) S

2009

S
(B) 6.73

(C) 6.95

(D) 7.31

9. The market share of which of the following is the


highest if these are the only companies in the
market and price per PC of the companies P, Q,
R and S in each of the given years was in the
ratio 1 : 2 : 1 : 2?
(A) S in 2006
(B) R in 2008
(C) S in 2007
(D) S in 2009

8. If in 2010, company S goes bankrupt and the


sales volume of the other companies increases
by 10% each, when compared to that in 2009,
then what is increase, in percentage points, in the
market share of company Q if these are the only
companies in the market and the cost of PC is
same for all the companies?

Directions for question 10: The following table gives the distribution of the number of handsets sold by a mobilephone manufacturing company from 2004 to 2006. P, Q, R and S denote the different models of mobile-phones sold
every year. Further it is given that the selling prices of the four models of mobile phones in the year 2004,SPP, SPQ,
SPR, SPS were in the ratio 3 : 4 : 5 : 6 and the ratio of the selling price of each model in the year 2004, 2005 and
2006 was 2 : 3 : 4.
3000
2500
2000

S
S

S
R

1500

500

P
2005

2006

1000

2004

10. The sales revenue of R in 2006 was more than the sales revenue of Q in 2004 by what %?

(1) 50%

(2) 56.25%

(3) 66

2
%
3

(4) 75%
16

Sales by volume (in '000)

Directions for questions 11 to 13: The following bar graphs gives the sales by volume of 3 cars sold in the
market during 2007 to 2009 and also the number of units of these cars produced in these years.
25

22
20

20

18

20

18
16

15
10
10

5
0
Alto

Swift
2007

Estilo

2008

2009

30
Production (in '000)

25
25

22

21

21

20

22

20
15

13

15

14

10
5
0
Alto

Swift
2007

2008

11. In which year is the ratio of the total production to


the total sales of all 3 cars the highest?
(1) 2007
(2) 2008
(3) 2009
(4) Both (1) and (2)

Estilo
2009

13. Production sales = Exports


In which year is the ratio of the exports to sales of
Swift the highest?
(1) 2007
(2) 2008
(3) 2009
(4) Both (1) & (2)

12 In which year is the ratio of the production of Alto


to the sales, the highest?
(1) 2007
(2) 2008
(3) 2009
(4) Both (1) and (2)
PIE CHART

Directions for questions 1 to 6: The total scaled scores obtained by a student in five AIMCATs is as shown in
the table below. An AIMCAT has five sections Quantitative Ability (QA), Logical Reasoning (LR), Verbal Ability
(VA), Reading Comprehension (RC) and Data Interpretation (DI).
Exam
AIMCAT 1
AIMCAT 2
AIMCAT 3
AIMCAT 4
AIMCAT 5

Total scaled scores


300
280
360
320
350

17

The following pie charts give the distribution of the scaled scores in each AIMCAT.

AIMCAT 2

AIMCAT 1
VA
20%

RC
20%

DI
19%

DI
25%

LR
11%

QA
17%

RC
15%

VA
12.5%

QA
37.5%

LR
23%

AIMCAT 4

AIMCAT 3
QA
25%

DI
22.5%

QA

DI
25%

331/3%
RC
15%
VA
162/3%

VA
10%

LR
12.5%

RC
20%
LR
20%

AIMCAT 5
DI
18%

QA
24%

RC
20%
VA
26%

LR
12%
NOTE: The scores given are actually the scaled scores. The actual scores are obtained by dividing the scaled
scores by the scoring factors. The maximum scaled scores in the five sections are also given below

Section
Scoring factor
Maximum scaled score

QA
1.5
150

LR
1.25
100

1. The scaled score obtained by the student in the


VA section in AIMCAT 3 is what percent of the
maximum possible actual score in that section?
(A) 48%
(B) 60%
(C) 80%
(D) None of these
2. What is the least difference between the scaled
score in the RC section and the maximum
possible actual score in that section in any of the
given AIMCATs?
(A) 4
(B) 6
(C) 11
(D) 19
3.

In an AIMCAT, if in at least three of the


five sections a candidate has a scaled score in a
section greater than 80% of the maximum
possible actual score in that section, the
performance of the candidate is considered to be

VA
2.5
125

RC
1.25
75

DI
1.2
120

significant. In how many of the five AIMCATs,


the student shows a significant performance?
(A) 1
(B) 2
(C) 3
(D) 5
4.

From AIMCAT 1 to AIMCAT 5, in which section


did the student have the highest percentage
increase in the scaled score?
(A) LR
(B) QA
(C) RC
(D) DI

5. From AIMCAT 1 to AIMCAT 5, in which section,


did the student have the least percentage change
in the actual score?
(A) LR
(B) VA
(C) RC
(D) DI
6. In which AIMCAT was the marks obtained by the
student in the RC section the highest?
(A) AIMCAT 4
(B) AIMCAT 5
(C) AIMCAT 3
(D) AIMCAT 1

18

Direction for question 7: 1000 students in a school


have to choose one extra-curricular activity (in which
they are interested) among 5 activities Dancing,
Singing, Printing, Embroidery classes and Karate.
Only boys chose Karate, and only girls chose
Embroidery classes. The ratio of the number of boys
to girls in painting is 1 : 1. 80% of the students who
choose Dancing are boys and 80% of the students
who choose singing are girls. The following pie chart
gives the distribution of the students in the five
activities.

Painting
15%

Dancing
45%

Karate
15%

Singing
20%
Embroidery
5%

7. If the students who selected Painting and Singing


are made to sit in the same class then what
would be the ratio of boys to girls in that class?
(1) 23 : 45
(2) 1 : 2
(3) 23 : 47
(4) 23 : 49

Direction for question 8: The following pie charts give the number of employees in central government jobs and
state government jobs in 6 states of India

UP
12%

MP
8%

AP
25%

Karnataka
20%

TN
20%

MP
14%

UP
7%

TN
25%

Karnataka
9%

Kerala,
25%

Kerala,
15%

State Govt. Jobs

Central Govt. Jobs

8.

AP
20%

The number of employees in central government jobs and in state government jobs are in the ratio
6 : 1. Find the ratio of the number of employees in central govt. jobs in AP to that in state jobs in Kerala?
(1) 6 : 1
(2) 4 : 1
(3) 5 : 2
(4) 3 : 1

Directions for questions 9 and 10: The following pie charts give the details of all the employees of 2 companies
P & Q:
HR dept
10%

Operations
dept
30%

HR dept
21%

Academic dept
17%

Academic
dept
60%
Operations dept
62%

Total num ber of em ployees in P = 17500

9.

What percent of the employees in both the


companies belong to the HR department?
(1) 17.5% (2) 15.5% (3) 16.1% (4) 17%

Total num ber of em ployees in Q = 18000

10. What is the approximate ratio of the number of


employees in the Academic department to the
Management department (HR dept + operation
dept) in both the companies combined?
(1) 0.71
(2) 0.62
(3) 0.9
(4) 0.84

19

Pie Charts + Bar Charts

Directions for questions 1 and 2: In a management institute, students opt for various disciplines. The
distribution of students across disciplines is shown in the pie chart and the ratio of the number of males and
females in each discipline is shown in the bar chart. The institute has sixteen centres in the country and the
students whose data is represented in the following charts are from those centres in the year 2009-2010.
Study the given charts carefully to answer the following questions.

Total number of students = 10080.


HR
25%

Marketing
37.50%

systems
15%
Finance
12.50%

operations
10%

15

10

0
Marketing

HR

Males

1. In the year 2009 2010, the total number of


female students in the institute was less than the
total number of male students by what
percentage?
(1) 11%
(2) 9.2% (3) 10.6% (4) 12.4%

Finance

Operation

Systems

Females

2. For which discipline was the difference between


the number of male and female students the
highest?
(1) H R
(2) Finance
(3) Marketing
(4) Systems

20

LINE GRAPH

Directions for questions 1 and 2: The following graph gives the relation between the total expenses and the
number of units produced in a factory.
Total expenses (in Rs.)

10000
8000
6000
4000
2000
0
50

100

150

200

250

300

350

400

Number of units produced


Assume that all units produced are sold and that on a normal day 200 units are produced. The selling price of
each unit is `35.
1.

On a particular day if 300 units are sold, find the


percentage change in profit when compared to a
normal day
(A) 100
(B) 150
(C) 200
(D) 250

2.

What is the average additional cost (in `) per unit


produced in comparison to a normal day, when
350 units are produced?
(A) 18
(B) 23
(C) 25
(D) 28

Directions for questions 3 and 4: A family uses the


following electrical appliances TV, Refrigerator,
Geyser, Washing machine and Grinder.
The monthly electricity bill generated has two
components- a fixed cost of `60 and a variable cost
of `0.35 per kWh.
The family uses Refrigerator throughout the day,
Geyser for 2 hours, Washing machine for 0.5 hours,
Grinder 0.25 hours and watches TV for 15 hours
everyday.

Energy Consumption (in KWh)

The line graph given below shows the energy


consumption of the above mentioned appliances in a
week for the family.
50
40
40
30

35

20

39

2 Geyser + Refrigerator
3 Geyser + Refrigerator + TV
4 Geyser + Refrigerator + TV + Washing machine
5 Geyser + Refrigerator + TV + Washing machine
+ Grinder
3.

Which of the following is true?


(1) The energy consumed by TV for 3 days is
more than that of Refrigerator for 3 days.
(2) The energy consumed by Geyser for 4 days
is less than that of Grinder for 7 days.
(3) The energy consumed by Washing machine
in a week is less than that of Geyser for 2
weeks.
(4) The energy consumed by TV for 2 days is
less than that of Washing machine for a
week.

4. If the fixed cost increases by 25%, then what


would be the percentage increase in the total cost
of energy consumption by the family in a month
of 30 days.
(1) 10%
(2) 12.5%
(3) 15%
(4) 19%
DATA SUFFICIENCY

Directions for questions 1 to 4: Each question is


followed by two statements, A and B. Answer each
question using the following instructions:
Mark (1)

21

Mark (2)

10
7

0
0

1 Geyser

Mark (3)
2

Mark (4)

If the question can be answered by using


the statement A alone but not using the
statement B alone
If the question can be answered by using
the statement B alone but not by using
the statement A alone.
If the question can be answered by using
either of the statements alone.
If the question can be answered by using
both the statements together but not by
either of the statements alone.

21

Mark (5)

If the question cannot be answered on


the basis of the two statements.

Numbers

Directions for questions 1 and 2:


1.

2.

3.

4.

In a particular company, sixty employees were


managers. Ten among them were also among
the people who had newly joined. How many
employees in the company were newly joined?
(A) Sixty percent of the newly joined employees
were not managers.
(B) All the newly joined employees were not
necessarily managers.
Five people Amar, Babu, Craig, David and
Edward were the only ones who participated in a
chess tournament. They were ranked on the
basis of the points they scored. David got a
higher rank as compared to Edward while Babu
got a higher rank as compared to Craig.
Craigs rank was lower than the median.
Who among the five got the highest rank?
(A) Amar got the last rank.
(B) Babu was not among the top two rankers.
Thirty percent of the students of a school are
boys. Ten percent of the girls in the school are
athletes. What is the percentage of boys in the
school who are athletes?
(A) Twenty five percent of the students are athletes.
(B) Number of boys in the school who are
athletes is 20% more than the number of girls
who are athletes.
In a basketball match, team A was trailing by
25 points at the end of the first half. Did it win the
match?
(A) In the second half team A scored 35 points.
(B) The opponent scored 35 points in the match.
PPL

Directions for question 1:


(1) If the question can be answered from one of the
statements alone but not from the other.
(2) If either statement alone is sufficient to answer
the question.
(3) If both the statements together are sufficient but
either statement alone is not sufficient.
(4) If the question cannot be answered even by
combining both the statements
1.

Pramod bought a new car after selling his old car.


If the cost of the old car was 40% that of the new
car, find the price of the new car.
. He borrowed an amount which was equal to
60% of the cost of the old car from his friend
and raised the remaining amount by
withdrawing from his personal savings account.
. His total personal savings were `3,00,000

Mark option
(1), if the question can be answered by any one of
the statements alone but not by the other.
(2), if the question can be answered by either
statement alone.
(3), if the question can be answered by combining
both the statements but not by each statement
alone.
(4), if the question cannot be answered even after
combining both the statements.
1.

Which of x, y, z is the maximum?


A. xy = 18 and yz = 21
B. xz = 42, where x, y and z are natural
numbers.

2. If ONE = O + N + E, i.e. considering 'O'=6, N= 7


and E = 8 we will get ONE = O + N + E = 6 + 7 +
8 = 21.
Find the value of SEEN, if SEVEN = 19 and all
alphabets have distinct values which are natural
numbers.
A. FIVE = 14
B. NINE = 7

Directions for question 3: This question is based on


the following information.
Raju's dad goes to various temples on certain days in
a year of 365 days which are numbered 1, 2, .365.
He goes to the Shiva temple on days which are
multiples of 3.
He goes to the Venkateshwara temple on days which
are multiples of 4.
He goes to the Saibaba temple on the days which are
multiples of 7.
3. On how many days did Raju's dad go to only one
temple?
(1) 204
(2) 214
(3) 220
(4) 240
4. If A, B, C, D, E, F and G are distinct single digit
natural numbers from 1 to 7such that A + B + C =
C + D+ E = E + F + G = 11, then how many
ordered pairs (C, E) exist which satisfy the given
relation?
(1) 0
(2) 2
(3) 4
(4) 8
CASELET

Directions for questions 1 to 3: Answer the following


questions based on the statements given below.
(i)
(ii)
(iii)
(iv)

There are three buildings on each side of the


road.
These six buildings are labelled as A, B, C, D, E
and F.
The buildings are of different colours, namely,
Violet, Indigo, Blue, Green, Yellow, and Orange.
The buildings are of different heights.

22

E, the tallest building, is exactly opposite to the


Violet coloured building.
(vi) The shortest building is exactly opposite to the
Blue coloured building.
(vii) F, the Green coloured building is located
between A and D.
(viii) C, the Yellow coloured building is exactly
opposite to A.
(ix) B, the Blue coloured building is exactly opposite
to F.
(x) A, the Orange coloured building, is taller than C,
but shorter than B and D.

(1) P, S and U
(3) T and U
(5) S and U

(v)

1.

What is the colour of the building diagonally


opposite to the Yellow coloured building?
(1) Orange
(2) Indigo
(3) Blue
(4) Violet
(5) None of these

2.

Which is the second tallest building?


(1) A
(2) B
(3) C
(4) D
(5) Cannot be determined

3.

What is the colour of the tallest building?


(1) Violet
(2) Indigo
(3) Blue
(4) Yellow
(5) None of these

Directions for questions 4 to 7: Answer the following


questions based on the information given below.
Six teams (P, Q, R, S, T and U) are taking part in a
cricket tournament. Matches are scheduled in two
stages. Each team plays three matches in stage-
and two matches in stage-. No team plays against
the same team more than once in the event. No ties
are permitted is any of the matches. The observations
after the completion of stage- and stage- are as
given below.
Stage :
One team won all the three matches.
Two teams lost all the matches.
S lost to P but won against R and U.
T lost to Q but won against R and U.
Q lost at least one match.
U did not play against the top team of stage-.
Stage :
The leader of stage- lost the next two matches.
Of the two teams at the bottom after stage-, one
team won both matches, while the other lost both
the matches.
One more team lost both matches in stage-.
4.

The two teams that defeated the leader of stage-


are:
(1) U and S
(2) T and U
(3) Q and S
(4) T and S
(5) U and Q

5.

The only team(s) that won both matches in


stage- is (are):
(1) Q
(2) T and U
(3) P, T and U
(4) Q, T and U
(5) Q and U

6.

The teams that won exactly two matches in the


event are:

7.

(2) S and T
(4) S, T and U

The team(s) with the most wins in the event is (are):


(1) p
(2) P and R
(3) U
(4) T
(5) Q and T

Directions for questions 8 to 12: Answer the following


questions based on the information given below.
Anand, Bala and Chandu are three professional
traders who traded in gold in the commodities market.
Anand followed the strategy of buying at the opening
of the day at 10 a.m. and selling the whole lot at the
close of the day at 3 p.m. Bala followed the strategy
of buying at hourly intervals: 10 a.m., 11 a.m,
12 noon, 1 p.m. and 2 p.m. and selling the whole lot
at the close of the day. Further, he buys an equal
quantity (by weight) in each purchase. Chandu
followed a similar pattern as Bala but his strategy is
some what different. Chandu total investment amount
is divided equally among his purchases. The profit or
loss made by each investor is the difference between
the sale value at the close of the day less the
investment. The return for each investor is defined as
the ratio of the profit or loss to the investment amount
expressed as a percentage.
8.

On a day of fluctuating prices, the price of gold


ends with a gain, i.e., it is higher at the close of
the day compared to the opening value. Which
trader got the maximum return on that day?
(1) Bala
(2) Chandu
(3) Anand
(4) Bala or Chandu
(5) Cannot be determined

9.

Which one of the following statements is always


true?
(1) Anand will not be the one with minimum return.
(2) Return for Chandu will be higher than that for
Bala.
(3) Return for Bala will be higher than that of
Chandu.
(4) Return for Chandu cannot be higher than that
of Anand.
(5) None of the above

10. On a boom day the price of gold keeps rising


throughout the day and peaks at the close of the day.
Which trader got the minimum return on that day?
(1) Bala
(2) Chandu
(3) Anand
(4) Anand or Chandu
(5) Cannot be determined

One day, two more traders, David and Emma joined


Anand, Bala and Chandu for trading in gold.
David followed a strategy of buying equal quantity of
gold at 10 a.m., 11 a.m. and 12 noon and selling the
same quantity at 1 p.m., 2 p.m. and 3 p.m. Emma on
the other hand followed the strategy of buying using all
her money at 10 a.m. and selling all of them at 12 noon
and again buying using all her money at 1 p.m. and
again selling them at the close of the day at 3 p.m.
At the close of the day the following was observed.
(i) Anand lost money in the transaction.

23

(ii) Both David and Emma made profits.


(iii) There was an increase in the price of gold during
the closing hour compared to the price at 2 p.m.
(iv) The price of gold at 12 noon was lower than the
closing price.
11. The price of gold was its highest at
(1) 10 a.m.
(2) 11 a.m.
(3) 12 noon
(4) 1 p.m.
(5) Cannot be determined
12. Which of the following is necessarily false?
(1) The price of gold was not at its lowest at 2 p.m.
(2) The price of gold was at its lowest at 11 a.m.
(3) The price of gold at 1 p.m. was higher than
the price at 2 p.m.
(4) The price of gold at 1 p.m. was higher than
the price at 12 noon.
(5) None of the above

Directions for questions 13 to 15: Select the


correct alternative from the given choices.
13. Four persons A, B, C, and D on a tour to a hill
station booked four consecutive rooms among
101, 102, 103 and 104 (all the rooms being on
the same side in that order) in a hotel for their
stay there. A had a dispute with B and did not
want to stay in a room adjacent to him. C being a
childhood friend of D booked a room adjacent to
D who in turn booked the room adjacent to B. If B
booked an odd numbered room, then which room
did C book?
(A) 101
(B) 102
(C) 103
(D) 104
14. Four girls Dolly, Molly, Polly and Kelly appeared
for their semester paper on Mass Communication
in which two of them failed. When asked as to
who passed in the exam, they gave the following
replies.
Dolly: I did not fail in the examination
Molly: I passed in the examination and so did Kelly.
Polly: Only one among Dolly and Kelly failed in
the examination
Kelly: Only one among Polly and Dolly passed
in the examination.
If it is given that exactly three of them were telling
the truth, then find the person who was lying.
(A) Dolly (B) Molly (C) Polly
(D) Kelly
15. In IIBM, a reputed B school with a total strength
of 270 students, every student opted for at least
one specialization among the three Finance,
HR and Marketing. The number of students who
opted for all three was 37.5% of those who opted
for exactly one. The number of students who did
not opt for Finance was 50% of these who opted
for Finance which in turn was 25% more than
those who did not opt for HR. The number of
students who opted for only marketing and HR
was equal to those who opted for only Finance
and Marketing and also 331/3% of those who
opted for all three categories. If the number of
students who opted for only Finance and HR was
50% of those who opted for only Finance, then
how many students opted for exactly two of the
three specializations?
(A) 36
(B) 72
(C) 45
(D) 54

Directions for questions 16 and 17: Akira and Aroki


read four books each from among A, B, C, D, E, F, G
and H such that each of the eight books was read by
exactly one person. Further the following information
was known.
The person who read D, also read F. Books A and B
were not read by the same person. The person who
read book F, did not read book C.
16. If Akira read books E and G, then Aroki did not
read book
(A) D
(B) F
(C) H
(D) C
17. If books C and E were not read by the same
person, then which of the following two books
was definitely read by the same person
(A) A and C
(B) B and C
(C) G and H
(D) Cannot be determined

Directions for questions 18 to 25: Select the


correct alternative from the given choices.
18. Nine workers W1, W2, W3, W4, W5, W6, W7, W8,
and W9 are to be allotted work in four shifts
Morning, Afternoon, Evening and Night with not
more than three workers in the same shift.
The workers are allotted the shifts as per the
following conditions.
Each worker can be allotted work in only a single
shift.
W1 and W2 do not work in the same shift.
W3 is to be allotted a shift, earlier than W6 but
later than W2.
W2 is to be allotted the morning shift and W4 is to
be allotted a shift which is two shifts after W1.
W5 and W7 are both allotted the same shift and it
is one shift earlier than the shift allotted to W6.
W9 is allotted the same shift as W3.
W8 cannot be allotted work in which of the
following shift?
(A) Morning
(B) Afternoon
(C) Evening
(D) Night
19. Four players A, B, C and D are the members of a
cricket team. They are to lead their team in the
upcoming tournament where they will be playing
five matches: Match1, Match 2, Match 3, Match 4
and Match 5 to be held one after the other with
each match having two leaders, one as captain and
the other as vice captain. No person can Captain
the side in two consecutive matches and neither
can the same person be the vice captain on
consecutive matches. These players are to lead
their team subject to the following conditions.
(1) B was the captain of the side for Matches
1 and 3.
(2) C can be the vice captain only if A is the
captain.
(3) D refused to lead the team as captain if A or
B led the team as captain in the preceding
match.
(4) Each of the four players was the captain and
the vice captain in at least one of the five
matches.

24

Which of the following statements is true?


(A) C was the vice captain in Match 5.
(B) B was the Vice captain in Match 4.
(C) C was the vice captain in Match 2.
(D) A was the captain in Match 5
20. Each of the three persons A, B and C have to
buy three household appliances AC,
Refrigerator and Water Purifier on three days
Monday, Tuesday and Wednesday. No two
persons buys the same type of appliance on any
single day. B buys the Refrigerator on Tuesday,
while he has to buy the Water Purifier before
buying the AC.
Which of the following is true regarding A?
(A) A buys the Water Purifier before the
Refrigerator.
(B) A buys the AC before buying the Water Purifier.
(C) A buys the AC before buying the Refrigerator.
(D) A buys the Refrigerator before buying the AC.
21. The following table gives the average runs scored
by four players in all the matches in a year.
Player
Sachin
Dravid
Mongia
Hussey
Sachin & Mongia
Dravid & Hussey

Average Runs
48
50
40
42
44
46

24. A race is held on three days and there are three


drivers Schumi, Sebastian and Mclaren.
On each day they are ranked from one to three in
the order in which they finish the race. On any
day, Schumi is always ranked ahead of Mclaren.
No driver secures the same rank on more than
two days. Which of the following must be false?
(A) Sebastian wins the race on exactly two days.
(B) If Sebastian wins the race on Day 1, then he
is ranked below Mclaren on the remaining
two days.
(C) Sehbastian comes last on exactly two days.
(D) Sebastian beats Mclaren on all the three days.
25. A company has two branches, one at Kukatpally
and the other at Narayanguda. Six persons
Tom, Raj, Ryan, Mokambo, Sashi and Govind
have to work in these branches. However, no
person works in both the branches. Each branch
has three employees. Further, it is known that
(1) Tom & Raj do not work in the same branch.
(2) Sashi and Govind works in the same branch.

Which of the following must be true?


(A) If Ryan works in the Narayanguda Branch,
then Mokambo does not work in the
Kukatpally branch
(B) Both Tom and Mokambo work in the same
branch.
(C) Both Govind & Raj work in the same branch.
(D) More than one of the above.

Sachin played more number of matches than


Dravid. If n is the average runs scored by all four
players, then what must be true about x?
(A) 44 < x < 45
(B) 45 < x < 46
(C) x = 45
(D) None of the above

Directions for questions 26 to 28: The analysis of


the way the three star players of Mumbai Indians
made runs is given below. The runs made by Pollard,
Dumminy and Bhajji consist of three types of shots
straight drive, pull shot and others.

22. In an international maths olympaid, there were


seven questions. The marks scored by two
candidates in seven questions (though not
necessarily in the same order) from Q1 to Q7 are
as follows

(i) The total runs made by Pollard is 40 more than


that made by Bhajji.
(ii) Pollard scored 20% of his runs through the pull
shot.
(iii) The runs made by Dumminy is the average of the
runs made by Pollard & Bhajji.
(iv) Bhajji scored 25% of his runs, i.e., 20 runs,
through he pull shot.
(v) The runs scored by Dumminy through others is
15% of the sum of the total runs scored by
Pollard and Bhajji.
(vi) The runs scored by Dumminy through straight
drive is 60% of the total runs made by him.
(vii) Each player scored at least one run through each
type of shot.

Ramesh: 26, 30, 34, 42, 46, 50, 54


Sanjay:
13, 34, 42, 46, 50, 54, 62
One of the questions was declared invalid
(after the exam and the marking was over).
The average marks of both the candidates in the
remaining six questions were calculated. Both
candidates scored same marks in the invalid
question, and so as a result of its exclusion, the
average score of one candidate increased while
that of the other candidate decreased. What was
the marks scored by each candidate in the
invalid question?
(A) 42
(B) 46
(C) 34
(D) 50
23. (1) If A wins, then B wins.
(2) If B wins, then C does not win.
(3) Only if D wins, then at most one of A or C wins.
If there are only four players A, B, C and D, then
which of the following must be true?
(A) D wins
(B) D does not win
(C) B wins
(D) A does not win

26. Find the maximum possible difference between


the runs scored by Pollard through Straight drive
and that by Dumminy through Pull shot.
(A) 110
(B) 86
(C) 50
(D) None of these
27. The runs scored by Bhajji through straight drive is
x% of the total runs scored by all the three players
put together. Find the maximum possible value of x.
(A) 20
(B) 30
(C) 19
(D) 192/3

25

28. Find the runs scored by Bhajji through others.


(A) 50
(B) 60
(C) 40
(D) Cannot be determined

Directions for questions 29 to 41: Select the


correct alternative from the given choices.
29. (1)
(2)
(3)
(4)

All shoes are pens.


Not all pens are pencils.
All pens are chocolates.
Not all chocolates are pens.

Which of the following must be true?


(A) Some chocolates are not shoes.
(B) Some shoes are chocolates.
(C) Some pencils are not chocolates.
(D) More than one of the above.
30. Ramesh was gifted a wonderful watch by his
father. When the watch was showing 4 p.m.
Ramesh started for his friend Umeshs house to
show off his watch. He was driving at a constant
speed. His friends watch was showing 6:10 p.m.
at the moment Ramesh arrived at Umeshs
house. Ramesh immediately drove back at
5
th of the earlier speed. As soon as he reched
4
home, he saw that the time shown by his watch
was 7:45 p.m. By how many minutes is Rameshs
watch faster/slower as compared to Umeshs
watch?
(A) No difference
(B) 10 mins slower
(C) 10 mins faster
(D) 5 mins slower
31. In a selection processes, each candidate has to
appear for two types of tests T1 and T2.
200 candidates failed in T2 while 300 failed in T1.
Ratio of the number of candidates who failed in
both T1 and T2 to those who passed in both T1
and T2 is same as the ratio of the number of
candidates who passed in T2 to those who
passed in T1. This ratio is an integral value. Find
the number of candidates who passed in both the
tests.
(A) 100
(B) 50
(C) 20
(D) Cannot be determined
32. Four persons W, X, Y, Z secured distinct ranks
from 1 to 6 in four events Swimming, Running,
Cycling and Walking.

Swimming Running Cycling Walking Total


W
5
6
15
X
1
6
Y
5
2
4
14
Z
1
18
Which of the following was ranked 3?
(A) X in Cycling
(B) W in Swimming
(C) Z in Running
(D) More than one of the above.
33. Each of the five students Adam, Ben, Cathy,
Dimitry and Emmanuel wrote four exams, one
each in Mathematics, Physics, Chemistry and

Biology and were ranked from 1 to 5 (1 being the


highest and 5 being the lowest) in each of the
exams. The sum of the ranks obtained by them in
the four exams were 11, 15, 12, 8 and 14
respectively in the same order where no two
students got the same rank in any exam. Further
it was known that
(1) Dimitry got the 4th rank in Physics and his ranks
in Chemistry and Biology were the same.
(2) The rank of Emmanuel in Chemistry was the
same as that of Dimitry in Mathematics.
(3) The ranks secured by Ben in Mathematics
and Chemistry was the same.
(4) Ben had the least rank in Physics and Cathy
got the 3rd rank in Mathematics.
(5) Emmanuel got distinct ranks in all four
subjects and did not get the best rank in any
subject.
Which of the following is definitely true.
(A) Adams rank in Maths was 2.
(B) Adams rank in Physics was 2.
(C) Adams rank in Chemistry was 4.
(D) Cathys rank in Chemistry was 3.
34. There are 120 families living in a housing society
where each family owns at least one among a
Refrigerator, an Air Conditioner and LCD TV.
24 families have only a Refrigerator.
20 families have only an Air Conditioner.
26 families have only a LCD TV.
At least 40 families own both Refrigerator as
well as Air conditioners.
At most how many families own a refrigerator and
an LCD TV but not an air conditioner?
(A) 10
(B) 12
(C) 15
(D) 30
35. Five lecturers L1, L2, L3, L4 and L5 are to deliver
lectures in a college on four consecutive days
from Monday to Thursday with two lectures to be
delivered on each day. No lecturer is to deliver
more than two lectures in the given period.
Further it is known that,
L1 delivers lectures on Monday and Thursday only.
L3 delivers a lecture on a day only if L2 delivered
a lecture the preceeding day.
L1 and L4 do not deliver lectures on the same
day. Each lecturer delivered at least one lecture
in the given period.
L3 and L4 do not deliver lectures on consecutive
days.

On which day did L3 deliver the lecture?


(A) Monday
(B) Tuesday
(C) Wednesday
(D) Thursday.
36. A committee consisting of five members is to be
formed from five boys among B1, B2, B3, B4 and
B5 and three girls among G1, G2 and G3.
There must be three boys and two girls in the
committee. Further it is known that,
(1) If B1 is selected, then B2 cannot be selected.
(2) If B3 is selected, then B4 cannot be selected.
(3) Both G1 and G2 cannot be selected at the
same time.
(4) If B4 is selected then B5 must also be selected.
(5) If B2 is selected, then G2 must also be selected.

26

The total number of ways in which the committee


can be formed is
(A) 5
(B) 4
(C) 3
(D) 6
37. Three products P1, P2 and P3 needs to be
machined in two machines M1 and M2. A product
cannot be machined in two machines at the same
time and the entire machining of a product in a
machine must be completed in one go. A product
can be machined in the machines in any order
(i.e., M1 before M2 or M2 before M1). The duration
(in hours) for which the products needs to be
machined in each machine is given in the table
below.

P1
P2
P3

M1
3
4
2

M2
2
3
5

If the total time taken to finish machining all the


three products is the minimum, then which of the
following is definitely false.
(A) Product P3 is machined in M1 before Product P2
(B) Product P2 is machined in M2 before Product P1.
(C) Product P2 is machined in M1 before Product P1.
(D) None of these.
38. In a group of 30 members belonging to a Sports
Club, each member played at least one of the three
games from football, cricket and hockey.
18 members played at least two games. If the
number of members playing exactly one game was
three times that of those playing all the three games,
then how many played exactly two games?
(A) 12
(B) 14
(C) 16
(D) 10
39. Ten athletes M1 to M10 competing in an athletics
meet represented five countries among UK,
Germany, France, Switzerland and Turkey with
each country being represented by two athletes.
Further it was known that M1 and M3 represented
the same country.
Both M2 and M4 were either from France or from
Germany.
M5 and M9 were from different countries.
M6 belonged to UK and so did M8
M7 was from Turkey and M9 did not belong to
France or Switzerland.
Which country did M10 belong to, if M5 did not
belong to France or Switzerland?
(A) France
(B) Turkey
(C) Germany
(D) Switzerland
40. Each of the four friends Sneha, Shikha, Sushma
and Sushmita bought a birthday present for their
common friend Rahul. The gifts bought by them
were a shirt costing `1200, a tie costing `800,
trousers costing `2000 and a pair of shoes
costing `2800 (not in the same order). The sum
of the costs of the gifts bought by Sneha and
Sushma was equal to that of the gift bought by
Shikha. Further the difference between the cost
of Sushmas gift and Sushmitas gift was equal to
the cost of Snehas gift. What was the gift bought
by Shikha?
(A) Shirt
(B) Trousers
(C) Tie (D) Shoes

41. Two teams, each with three members, are to be


selected from among the seven students P, Q,
R, S, T, U and V for the Inter School Meet
consisting of a Debate and an Elocution contest.
In addition it is also known that
(1) P was the only student who represented his
school in both the contests.
(2) If Q was selected for Debate, then R must be
selected for the Elocution contest.
(3) Both S and T cannot be selected for the
same event.
(4) V represented his School in Debate where as
U was selected for one of the two categories.
(5) If R was selected for the Elocution contest,
then U cannot be selected for Debate.
Which of the following statement is definitely
false?
(A) Both U and Q cannot be selected for the
same event.
(B) If both V and U are selected for the same
event, then Q must be selected.
(C) If S is selected for Debate then either R or T
or Q must be selected for Elocution.
(D) None of these.
NETWORKS

Directions for question 1 and 2: The following


network gives the bus routes of APSRTC in
Hyderabad for its new A/C buses introduced in the
previous month. Any passenger boarding a bus is
charged `5 as local service charge and `8 as fixed
charge in addition to a charge of `4 per km. Further it
is also known that
(1) a bus does not visit the same city more than
once.
(2) between any two cities only one mode of
transport is available, i.e. the bus.
(3) in the network shown, the values in brackets
denote the distance in kilometers.
B

(5)

(6)

(4)
D
(1)

A
(2)
E

(3)

(7)

(5)
(1)

G
(3)
(8)

(3)

1.

Find the minimum cost incurred by a person to


travel from A to H.
(1) `53
(2) `75
(3) `61
(4) `73

2.

If the road connecting A to E is under repair, then


what is the minimum cost incurred by a person to
travel from A to H?
(1) `57
(2) `59
(3) `61
(4) `73

27

QBR (Miscellaneous)

Directions for question 1: Select the correct


alternative from the given choices.
1.

A group of 450 persons was tested for HIV


infection, but there was an error in the testing
process due to which four types of results were
observed. The results identified the following four
categories of persons.
C1: Persons infected with HIV but reported
negative in the test.
C2: Persons not infected with HIV but reported
positive in the test
C3: Persons not infected with HIV and reported
negative in the test
C4: Persons infected with HIV and reported
positive in the test
If it is known that the results of 275 people were
correctly reported and that the number of infected
people is 50% that of non-infected people, find the
difference in the number of people under
categories C2 and C4.
(1) 11
(2) 25
(3) 91
(4) 100

Directions for questions 2 and 3: There are two


different investment schemes and yielding
different percentage returns based on the market
conditions. Market conditions are catagorised into
three types, namely bearish, steady and bullish. The
probabilities of the market conditions and the
respective yield percentages for the two schemes are
given below
Scheme
Market conditions Probabilities Yield percentage
Bearish
0.25
30
Steady
0.55
80
Bullish
0.20
100
Scheme
Market conditions
Bearish
Steady
Bullish

1.

Which of the following will give maximum profit?


Selling price
Number of balls sold
(1)
`89
400
`93
360
(2)
(3)
`85
440
(4)
`99
300

Yield percentage
10
60
100

0.4

2.

If the total yield from both the schemes is the


same, what is the probability of the market
condition being bearish for scheme ?
(1) 15%
(2) 20%
(3) 25%
(4) 35%

3.

If the probabilities for the market conditions being


bearish, steady and bullish for scheme are 0.2,
0.45 and 0.35 respectively instead of what is
given in the table, then for the same yield
percentage (as given in the table), what would be
the percentage increase in the total yield from
scheme ?
(1) 8.9% (2) 9.1% (3) 10.2% (4) 15%

DI (Miscellaneous)

Directions for question 1: A company manufacturing


cricket balls incurs a manufacturing cost of `50 per
ball. The sale of the number of balls decreases
linearly such that the number of balls sold decreases
by 20 for every `2 rise in the selling price. It is known
that when the selling price of each ball is `59, the
number of balls sold is 700.

Probabilities

Directions for questions 4 and 5: The following figure gives the per capital income and the Happiness Quotient
of fifteen countries in the world vs the Per capita income. (Only for families with 3 or more members)
1.0

J
H L

Happiness Quotient

0.8

0.4
0.2

G
N

0.6
M

3000 4000 5000 6000 7000 8000 9000 10000 11000 12000 13000 14000

Per capita Income (in US $)

The following table gives the names of the countries represented in the above figure.
A-Taiwan
B-Australia
C-Japan
D-South
Korea

E-China I-Philippines M-Thailand


F-Czech
J-Poland
N-Singapore
Republic
G-Israel K-Slovakia O-Austria
H-Brazil L-Latvia

28

4. Which of the following gives the list of the


countries which indicate Low per capita Income
with low happiness Quotient
(1) China, Thailand, Slovakia
(2) China, Slovakia, Brazil
(3) China, Poland, Brazil
(4) China, Slovakia, Poland

5. Which of the following lists indicate high


happiness quotient with high per capita income.
(1) Australia Latvia, Austria
(2) Australia, South Korea, Austria
(3) Australia, Austria, Israel
(4) Australia, Latvia, Czech Republic.

Direction for question 6: The selection of a candidate into a B - school consisted of evaluating various factors Written score, Group Discussion, Essay writing, Interview and Work experience. The B - school assigns some
weightage to each of these factors. The cumulative score of a student is the sum of the product of the scores of
the student attained in these factors and the weightage assigned to the respective factor. The following diagram
gives the scores of two students Rahul and Ramya and also the weightage assigned to each factor.
WRITTEN SCORE 30%
5

GROUP
DISCUSSION
25%

4
3
2
1
3

WORK EXPERIENCE 10%

2
3

_____

Rahul
Ramya

5
ESSAY WRITING 10%

INTERVIEW 25%

6. What is the difference in the cumilative values of the scores obtained by Rahul and Ramya?
(1) 0.1
(2) 0.2
(3) 0.25
(4) 0.15

Directions for questions 7 and 8: In a game of cards


played by two gamblers Raju and Ramu, the payout
when a card out of King, Ace and Joker turns up, is
given below along with the probability of occurrence.
Card

Gambler

King
Ace
Joker
King
Ace
Joker

Raju
Raju
Raju
Ramu
Ramu
Ramu

Expected pay-out for any gambler is the weighted


average of probability of occurrence and payout.

Pay out Probability of


(in `)
occurrence
80
0.5
40
0.3
0.2
20
80
0.5
60
0.3
0.2
20

7.

What is the expected pay-out for Raju?


(1) 40
(2) 48
(3) 60
(4) 70

8.

If the probability of getting a King and a Ace for


Ramu is interchanged then what will be the
percentage decrease in pay-out after the
interchange as compared to the original payout?
(1) 7.4% (2) 7.2% (3) 6.25% (4) 12.5%

Direction for question 9: An agency "Book a Hotel" offers its customers an incertive to book a hotel room in
advance. It also offers its customers a half-a-day tour and also a tour of the Hunters Valley. The data is given in
the following flow chart:
Hotel

Booking

5 star

3 star
S/o $600

CWB $250

D/o $1100

S/o $1000

CWNB $125

CWB $350
City tour

Note: S/O Single Occupancy


D/O Double Occupancy
CWB Child With Bed
CWNB Child With No Bed.

D/o $1900

CWNB $200

Yes($ 40)

No
Hunters Valley
No

Yes($ 35)

29

9.

10.

If Ramu, an employee of EMIT, books a hotel


room in a 3 star hotel, goes on a city tour and
visits the Hunters Valley, then what is the
amount spent by Ramu?
(1) $685
(2) $665
(3) $675
(4) $655
3 groups for 3 different modules have to be
made from 4 junior software engineers (JSE) P,
Q, R, S and 4 senior software engineers (SSE)
X, Y, Z, W.
(1) P and S are in the group.
(2) X and Y cannot be in the same group as Q.
(3) R is a member of a group which has 3
people.
(4) R, Z, W are males.
(5) There has to be a male in every module.
(6) There has to be a JSE and a SSE in every
module.
The person who is in the group having 3
members is
(1) X
(2) Q
(3) Z
(4) W

12.

Directions for questions 13 and 14: Ram, a saree


seller bought 400 sarees of 3 different types
(Kanchipattu, Benarasi and Mangalagiri) in the ratio
5:3:2. He sells the sarees based on market demand.
On the 1st day he sells 20% of the total volume. On
the 2nd day he sells 50% of the total volume. On the
3rd day, he sells 30% of the total volume. The cost of
each Kanchipattu, Benarasi and Mangalagiri saree is
`350, `400 and `375 respectively. On each day he
sells the sarees of each variety in the same ratio as
he bought.
13.

Directions for questions 11 and 12: The following


table shows some integers in a 5 5 grid.
a
a
b
c
d
e

b
4

d
13

8
14
17

e
12

10
12

The numbers in the blocks are placed according to


the following conditions.
1.
The number in column 'b' and row 'd' is 1/3 the
sum of the numbers in column 'b'.
2.
The numbers in column 'a' are squares of prime
numbers in ascending order starting with the
first odd prime number.
11.

Which of the following is the value of the


number in column 'b' and row 'b'?
(1) 23
(2) 19
(3) 17
(4) 16

What is the total sum of the numbers in column


'a'?
(1) 208
(2) 312
(3) 216
(4) 373

If on the 3rd day he sells Benarasi saree for a


price which is more than 20% than their cost
price whereas the other two sarees are sold at
their cost price, then find the total amount
received by Ram by selling the sarees on the 3rd
day?
(2) `47280
(1) `45280
(3) `51280
(4) `49280

14. It is known that 25% of the sarees were slightly


damaged. If Ram managed to sell the sarees
which were slightly damaged at 20% loss, then
what is the amount made by Ram by selling the
damaged sarees. [consider the ratio of the
damaged sarees in the three categories is the
same as the ratio in which he bought]
(1) `37500
(2) `35000
(3) `29600
(4) `28000
15. The population of China in 2009 is 1.6 billion and
it is expected to grow at 12% every year till 2020.
If in 2020 the population of China is 15% of the
total population of the world, then find the total
population of the world in 2020 approximately?
(1) 30 billion
(2) 37 billion
(3) 45 billion
(4) 51 billion

Direction for question 16 and 17: The following graph gives the visibility index of a certain number of people:
1
I

Visibility Index

0.8
J

0.6

0.4
0.2

R S
K
D
C X L T A
Q

B U
M W
F
V
Y N
O P
Z
E

16.

For what percentage of the total number of


persons is the visibility index more than U?
(1) 50% (2) 54%
(3) 60%
(4) 64%

17.

For which pair of persons the visibility index are


equal?
(1) F, M
(2) C, L
(3) R, S
(4) A, T

30

18.

Six friends A-F play a game of cards. The winner of the game is found by adding the points allotted to the
cards. The person having the highest number points is declared the winner of the game. The six friends are
sitting around a circular table as given below
B

K-1
J-1
A-1
Q-1

K-1
J-2
A-2
Q-1

K-1
J-2
A-1
Q-0

K-2
J-1
A-1
Q-3

K-1
J-0
A-2
Q-2

K-2
J-2
A-1
Q-1

Card

Points

K - King

30

J - Jack

20

A - Ace

10

Q - Queen

Who among the following is the winner?


(1) A
(2) B

Quant SI CI

Directions for question 1 and 2: A company offers


four schemes for investors to invest. The schemes
are as follows.
Scheme 1: Gives a return of 8% p. a. interest being
compounded annually
Scheme 2: Gives a return of 15% p. a. simple
interest
Scheme 3: Gives a return of 10% p. a. interest being
compounded semi-annually
Scheme 4: Gives a return of 20% p. a. compounded
annually but 10% of the amount at the
end of each year is paid as
administrative charges and only the
remaining 90% is reinvested.
1. If a person decides to invest an amount of
`20000 equally in scheme 2 and scheme 3, then
after how many years will his interest from
scheme 3 be more than that from scheme 2?
(1) 7
(2) 9
(3) 10
(4) 11
2. If a person invests his entire money equally in all
four schemes, then which scheme will fetch him
the maximum interest after 8 years?
(1) scheme 1
(2) scheme 2
(3) scheme 3
(4) scheme 4
LA (Venn Diagram)
1. In a class all the students applied in at least one
of the three examinations among CAT, FMS and
XAT. 24 students applied for FMS, 32 students
applied XAT and 36 students applied for CAT.
12 students applied for both FMS and XAT,
15 students applied for both XAT and CAT
whereas 9 students applied for both FMS and
CAT. If the number of students who applied for all
the three was 25% of those who applied for at
least two of the three examinations, then how
many students were there in the class?

(3) C

(4) E

(1) 54

(2) 60

(3) 62

(4) 69

Directions for questions 2 and 3: Out of 800


persons living in a locality, 50% own a car, 60% can
speak in Hindi, and 500 can speak in English. 15% of
the total number of persons living in the locality own a
car and can speak both in Hindi and English. Of the
people who own a car, 240 cannot speak in English.
Out of the people who can speak in Hindi, 180 own a
car. There is no one in the locality, who does not own
a car, cannot speak in Hindi and cannot speak in
English.
2. What percent of the people who can speak in
English, can also speak in Hindi?
(1) 48%
(2) 54%
(3) 72%
(4) 75%
3. What proportion of the people in that locality do
not own a car or cannot speak in English?
(1) 0.44
(2) 0.64
(3) 0.72
(4) 0.8
4. In a survey conducted among 120 families living
in a locality regarding the daily newspaper they
read among The Telegraph, The times of India
and The Hindu, the following data was recorded.
The number of families reading The Hindu, The
Times of India and The Telegraph were 64, 48 and
45 respectively.21 families did not read any of the
three newspapers whereas 9 families read all
three newspapers.
How many families read exactly one newspaper?
(1) 40
(2) 45
(3) 50
(4) Cannot be determined

Directions for questions 5 and 6: 300 employees


work in a company. 20% of the employees are HOD's.
70% of the employees have weekly off on Sunday. 180
employees own a car. Half of the HOD's own a car. All
the HOD's have their weekly off on Sunday. 30
employees have a car but do not have their weekly off
on a Sunday.

31

5. What percentage of the company HOD's own a


car and have their weekly off on Sunday?
(1) 5%
(2) 10%
(3) 15%
(4) 20%
6. How many employees are not HOD's and do not
own a car?
(1) 60
(2) 70
(3) 80
(4) 90
7. 300 students passed engineering and chose only
one of the two PG courses MBA and MS.
70 students chose MS. In MBA, students chose
at least one of the three specialisations among
Finance, Marketing and HR. 20 chose general
specialisation
which
includes
all
three
specialisations. 100 students chose Marketing.
150 chose Finance. It is also known that the
number of students who chose only Marketing
and Finance is twice the number of students who
chose only HR and Marketing which in turn is
equal to the number of students who choose all
three specialisations. How many students chose
only HR specialization?
(1) 60
(2) 50
(3) 20
(4) 30
8. A few children visit a toy shop and buy some
toys. 16 children buy toys of T & J, 26 children
buy toys of C & H and 34 children buy toys of B &
B. It is known that each child buys exactly two
toys. How many children visited the toy shop?
(1) 28
(2) 32
(3) 34
(4) 38

Directions for questions 9 and 10: A survey was


conducted to find out which of the four movies P, Q,
R and S is liked by most people. The number of
people who like P, Q, R and S are 24, 36, 29 and 25
respectively. It is further known that

every person surveyed likes at least one movie

the number of people who like P and Q are 12.

the number of people who like R and P, Q and


R are 18 and 16 respectively.

the number of people who like only P, R and S


are 8 and those who like P, Q and R are 6

people who like S also like R.


9.

How many people like only Q?


(1) 10
(2) 11
(3) 12

(4) 14

10. How many people like all 4 movies?


(1) 7
(2) 6
(3) 5
(4) 4
LA (Miscellaneous)
1. Six pizza stores are ranked as per the price
offered by them on a particular variety of pizza
and the time taken by them to deliver the order.
The store offering the least price is ranked 1 and
when ranked as per the delivery time, the store
which delivered the earliest was ranked 1. If more
than one store offered the same price or
delivered in the same time, then they are given
the median of all the rankings. The price ranking
is given a weightage of 0.7 whereas the delivery
time ranking is given a weightage of 0.3. Study
the following table carefully and answer the given
question:

Pizza store
A
B
C
D
E
F

Price
(in `)
120
155
85
95
125
120

Delivery time
(in minutes)
30
25
20
45
30
40

Which of the following pizza stores got the third


lowest net score?
(1) A
(2) D
(3) E
(4) F

Directions for questions 2 to 4: In the Recruitment


process, a company asked the people who applied to
write four objective papers each consisting of four
questions. In the first paper, for every correct answer
15 marks were awarded whereas for every wrong
answer 5 marks were deducted. In the second paper,
for every correct answer 20 marks were awarded and
for every wrong answer 15 marks were deducted, but
if a person got all 4 questions correct, an additional
30 marks were awarded. In the third paper, for every
correct answer 25 marks were awarded and for every
wrong answer 15 marks were deducted. In the fourth
paper, 20 marks were awarded for every correct
answer and 5 marks deducted for every wrong
answer but if a student got three or more questions
wrong, 50 marks were deducted from his total.
2. What is the least score obtained by a person in
these four papers, if he gets 4 wrong answers
across all four papers?
(1) 200
(2) 150
(3) 180
(4) 165
3. What is the difference between the highest marks
obtained by two persons one of whom wrote paper
1 and paper 3 and the other wrote paper 2 and
paper 4, each getting 2 wrong answers overall?
(1) 20
(2) 30
(3) 40
(4) 60
4. What is the least total score obtained by a person
who got 2 questions wrong and attempted 3
papers?
(1) 145
(2) 150
(3) 160
(4) 165

Directions for questions 5 and 6: In a 9 seater van


having 3 rows of 3 seats each, the seats in the first
row are numbered 1, 2 and 3, those in the 2nd row are
numbered 4, 5 and 6 and those in these last row are
numbered 7, 8 and 9. The seats are arranged in a
grid form with 3 rows and 3 columns. These seats are
occupied by 9 persons from P, Q, R, S, T, U, V, W
and X not necessarily in this order. The seats are
occupied subject to the following conditions..
1. Q the driver occupies seat 3 and both R and V
are in his column.
2. P is the son of W and occupies the seat in the
same row as V and S.
3. T occupies seat 5 and is in the same row as W
and is not seated immediately in front of S.
4. X and T are not seated in the same column.

32

5. If persons sitting in the seats swap their positions


in the following order (1,4) (2,6) (4,6) (5, 7), (4, 5),
(7,2), then which of the following persons will be
seated beside U?
(1) X
(2) R
(3) S
(4) T
6. If swapping of seats is done in the order (1,2)
(2,8) (9,2) (4,9), (7,5) (5,8) (8,4), then which of
the following is true?
(1) S is in the fifth seat.
(2) W is in the ninth seat.
(3) T is in the second seat.
(4) P is in the second seat.

Directions for questions 7 to 9: A survey was


conducted in four colonies A, B, C and D whose
populations are in the ratio 3 :5 : 3 : 4, to find (i) the
proportion of residents who prefer watching movie to
eating out & (ii) the proportion of people who prefer
surfing the internet to chatting with friends. The
following was the data collected.

Colony A
Colony B
Colony C
Colony D

Proportion of
residents
who preferred
watching
movies
0.6
0.45
0.65
0.48

Proportion of
residents who
preferred
surfing
0.64
0.55
0.68
0.75

7. If 61% of the residents in colony C preferred both


watching movies and surfing the net, then what
percentage of the residents in colony C did not
prefer any of the two?
(1) 30%
(2) 28%
(3) 25%
(4) 35%
8. In which colony do the maximum number of
residents prefer chatting with friends?
(1) A
(2) B
(3) C
(4) D
9. In how many colonies is the number of residents
who prefer surfing the net more than the average
number of residents in the four colonies who
prefer the same?
(1) 0
(2) 1
(3) 2
(4) 3
10. There are eight oil tanks A, B, C, D, E, F, G & H
in a refinery by which oil is being transferred. Due
to the lack of sufficient connecting routes, oil
cannot be transferred from any tank to any other
tank but can only be transferred subject to the
following conditions.
(a) Oil can be transferred from A to E or F
(b) From E oil can be transferred to A, C or D
(c) From D oil can be transferred to A, B or C
(d) From F oil can be transferred to D or G
(e) From H oil can be transferred to A or E
(f) from G oil can be transferred to E, C or H
(g) from C oil can be transferred to A or G
(h) from B oil can be transferred only to E.

revisited in the route, i.e. oil can pass through a


particular tank only once?
(1) 5
(2) 6
(3) 7
(4) 8
11. Arun, Varun and Kiranmala, subscribed to three
business magazines, among India-Day-to Day,
India-Everyday and India-These-Days subject to
the following conditions:
1. If Arun subscribes to India-Day-to-Day, then
Varun subscribes to India-Every-day.
2. If Arun subscribes to India-Every-day, then
Kiranmala subscribes to India-These-Days.
3. If Varun subscribes to India-These-Days,
then Arun subscribes to India-Every-Day.
4. If Kiranmala subscribes to India-Day-to-Day,
then Varun subscribes to India-These-Days.
5. If Kiranmala subscribes to India-Every-Day,
then Arun subscribes to India-These-Days
6. If Varun subscribes to India-Day-to-Day, then
Kiranmala subscribes to India-Every-Day.
From the above constraints, which of the following
can be definitely concluded, if it is given that each
person subscribed for exactly one business
magazine.
(1) Arun subscribed to India-Day-to-Day
(2) Kiranmala subscribed to India-Every-day
(3) Varun did not subscribe to India-These-Days.
(4) More than one of the above.
12. If it is given that Kiranmala did not subscribe to
India-These-Days, then which of the following
magazines did Varun subscribe to?
(1) India-Day-to-Day
(2) India-These-Days
(3) India-Every-day
(4) Cannot be determined
13. There are 6 baskets 1, 2, 3, 4, 5 and 6. Two
baskets have gold, two have silver and the other
two have nothing in them. They are placed one
beside the other in a row. Rajini, a contestant,
has to select one of the baskets and can get
whatever is present in the basket. No two baskets
having the same thing inside are kept side by
side. What is the probability that Rajini goes
home by taking something?
1
2
1
1
(1)
(2)
(3)
(4)
3
3
2
6
14. In a forest there are P tigers and a goat. Grass is
eaten by both the goat and the tigers but the
tigers would rather prefer eating a goat. If a tiger
eats the goat then the tiger transforms into a goat
and hence runs the risk of being eaten by another
tiger. The tigers are intelligent and would not risk
being eaten.

Which of the following is true?


(1) If P is odd, no tiger eats the goat.
(2) If P is odd tiger close to the goat eats the
goat.
(3) For any value of P, the goat is not eaten.
(4) For any value of P, the goat gets eaten.

In how many ways can oil be transferred from


tanks B to tank H if the same tank cannot be

33

Directions for questions 15 and 16: During his


holidays Ram wanted to learn Tabla and Piano. Ram
went to learn Tabla for 12 days and to learn Piano for
8 days.

16. It is known that on 6 days he went to learn only


Tabla and not Piano, then on how many days in
all did Ram go to learn playing musical
instruments?
(1) 8
(2) 10
(3) 14
(4) 16

15. If on 4 days he went to learn both the


instruments, then on how many days in all did
Ram go to learn playing musical instruments?
(1) 8
(2) 10
(3) 14
(4) 16

Directions for question 17: The following gives the process of arranging words by a word arrangement
machine. Read the pattern below and answer the questions.
Input

Sunday

Monday

Tuesday

Wednesday

Thursday

Friday

Saturday

Step 1

Saturday

Sunday

Monday

Tuesday

Wednesday

Thursday

Friday

Step 2

Friday

Saturday

Sunday

Monday

Tuesday

Wednesday

Thursday

Step 3

Thursday

Friday

Saturday

Sunday

Monday

Tuesday

Wednesday

Which step will be the same as the input?


(1) Step 24
(2) Step 26

17.

(3) Step 29

LA (Circular Arrangements)
1. Eight persons A, B, C,D,E,F,G and H are seated
around a circular table, such that each person
has another person seated exactly opposite him.
B is seated opposite to E who is seated to the
immediate right of G.D is seated between E and
C. If F is not seated adjacent to A, then who is
seated opposite to H?
(1) D
(2) C
(3) G
(4) either C or D
2. Ten persons P, Q, R, S, T, U, V, W, X, and Y are
seated around a circular table with equal distance
between any two adjacent persons such that any
person can only see the person seated opposite
him and the two persons seated on either side of
that person (i.e on either side of the person
seated opposite).
Further the following information is known. P does
not want to see Q, R, S, or T but wants to see X and
Y.
U does not want to see R, S, T or V, but wants to
see P, W and Q.
T wants to see X and Y.
Which of the following gives the correct seating
arrangement?
(1) P T R U V X Y S Q W
(2) P W Q V Y X U R S T
(3) P S T R U Y X VQ W
(4) P Q W V Y X R U S T
3. A company "XYZ Ltd" asks some of its Branch
coordinators to attend a Round Table
Conference.

The coordinators sit around a circular table.

The Mumbai coordinator sits to the left of the


Bengaluru coordinator, who is opposite the
Hyderabad co-ordinator

(4) Step 28

The Delhi co-ordinator is not opposite the


Chennai coordinator but is to the left of the
Hyderabad coordinator

Who is opposite to the Delhi coordinator?


(1) Mumbai co-ordinator
(2) Chennai co-ordinator
(3) Bengaluru co-ordinator
(4) Hyderabad co-ordinator
4. Six employees P, Q, R, S, T and U of EMIT Pvt
Ltd. go to a party. They sit around a circular table.
P and S do not sit together and R and T always
sit together. In how many ways can they be
seated?
(1) 12
(2) 24
(3) 6
(4) 18
LA (Distribution)

Directions for questions 1 to 3: Twelve persons


who are members of a rock band live in a building
having twelve floors from the 1st floor to the 12th floor
with the ground floor and the basement being used
for parking. All the persons live in distinct floors.
Among them there are 4 singers, 3 guitarists, 2
drummers, 2 instrumentalists and 1 keyboard player.
There are seven male members in the group A, B, C,
D, E, F and G and 5 female members P, Q, R, S and
T. These members live in the building subject to the
following conditions:
1. No two female members lived on adjacent floors
and no singer lived on floors at the extreme ends,
i.e. the topmost floor or the bottom most floor.
2. D and R were both guitarists and lived on
adjacent floors but neither of them lived on the
topmost or the bottommost floor.
3. P and C were both singers and had exactly one
guitarist living on an adjacent floor.
4. A was neither a drummer nor a guitarist whereas
F was not a drummer.

34

5. T lived on a floor adjacent to at least one


drummer where as Q, the keyboard player lived
on the 7th floor.
6. All the singers were 3 floors apart from each
other where as all the guitarists were at least 2
floors apart from each other.
7. There were 3 floors between Q and B, who was
an instrumentalist, and there was only one female
guitarist.
8. Both the drummers were males and neither lived
in the topmost floor or the bottommost floor.
1.

How many male singers were there?


(1) 1
(2) 2
(3) 3
(4) 4

2. How many floors were above the floor on which


G lived?
(1) 4
(2) 6
(3) 8
(4) Cannot be determined
3.

Who lived on the second floor?


(1) A
(2) S
(3) F
(4) Cannot be determined

Directions for questions 4 and 5: Six couples got


married on different dates in the years between 1993
and 1999. Only two couples got married in the same
year and only two couples got married in the same
month. The couple who got married on April 23rd, got
married before the couple who got married on
October 15th, but after the couple who got married on
November 5th. There is only one couple who got
married in 1993 and they got married in the same
month as the couple who got married in 1998, there
being only one couple who married in 1998. The
couple who got married on October 15th was not the
last couple to got married and the couple who got
married on January 3rd was not the 3rd couple to have
got married. Two couples got married on the same
day in consecutive months but not in the same year.
4. Both the couples who got married in the same
month were married in the month of
(1) October.
(2) April.
(3) November.
(4) January.
5. What was the least difference between the
marriage dates of any two couples (in days)?
(1) 49
(2) 78
(3) 160
(4) None of these
6. From a group of nine friends consisting of three
boys Jalan, Jagan, and Jeevan and six girls
Kekul, Kokila, Kavya, Kavita, Kaya and
Kadambari, three groups each consisting of three
members are formed from the above persons
subject to the following conditions:
1. Kavya and Jeevan should be there in the
same group.
2. Kavya and Kekul should not be there in the
same group.
3. Kokila wants Jalan in her group.
4. Kaya and Kavitha must be in the same
group.
5. Jalan prefers not to be in the same group as
Kekul.

6. In a group, the number of boys must not be


more than the number of girls.
Jagan is in the same group as
(1) Kaya and Kavita.
(2) Kaya and Kadambari.
(3) Kavita and Kadambari.
(4) None of these

Directions for questions 7 and 8: There are


5 cricketers, L, M, N, O and P from different countries
(India, Australia, S.A, SL, England) and playing for
5 different teams in IPL (KKR, RCB, DC, DD, MI).
These cricketers are made to stand in a row for post
match presentation.

L, who is from SL, does not play for MI and is


standing at the extreme right end of the row.

The cricketer who is to the immediate left of N


plays for KKR and neither of them are from
Australia.

The cricketer who is from India plays for RCB,


and is adjacent to only one player P who plays
for DC.

O is to the immediate left of L, and is from SA.


7. What is the correct combination of country and
the team for which M plays?
(1) India, RCB
(2) England, KKR
(3) England, MI
(4) SA, KKR
8. The player from Australia plays for which team in
the IPL?
(1) DD (2) MI
(3) KKR
(4) DC
9. Four faculty members A, B, C, and D visited a
college on 4 consecutive days starting from
Monday and taught 4 different subjects Maths,
Physics, Chemistry and Biology. Further it is
known that A and C went to the college on
consecutive days and taught Biology and
Chemistry respectively. B went to the college on
the last day and did not teach Physics. If Physics
was taught immediately after Chemistry then
when was Biology taught?
(1) Monday
(2) Tuesday
(4) Wednesday
(3) Thursday

Directions for questions 10 and 11: In a hockey


tournament, 15 teams from A to O participated. The
teams are arranged in ascending order of the points
they scored in the tournament.
The following information is know.
(a) Team L scored the least number of points, i.e 96
(b) Team I scored the maximum number of points, i.e
364
(c) Team D, team G and team C were placed 4th, 7th
and 10th respectively with 116, 182, and 218
points.
(d) Team N and team B got 108, 165 points less than
Team F.
(e) The points scored by Team 'O' = 251
(f) Team A got 361 points.
(g) Team H got 4 points more than team M and team
E got 18 points less than Team N.

35

(h) The sum of the total points for the following


positions are as follows.
1 + 2 + 3 = 302; 4 + 5 + 6 = 412; 7 + 8 + 9 = 590;
10 + 11 + 12 = 753; 13 + 14 + 15 = 1046
(i) The ascending order of the teams is O, J, F, OJF.
10. What is the difference in the points obtained by
teams J and K?
(1) 136
(2) 144
(3) 151
(4) 161
11. What is the position of team E?
(1) 7th
(2) 8th
(3) 9th

(4) 10th

Directions for questions 12 and 13: Seven friends


P, Q, R, S, T, U and V went to a restaurant. Each
friend is wearing a different coloured shirt among
Violet, White, Orange, Blue, Green, Yellow and Red.
They order seven different cool-drinks among Pepsi,
Thums up, Sprite, Mazaa, Coke, Mountain dew and
Fanta.

P, who wears an orange shirt drinks Sprite.


The friend who wears a green shirt drinks Pepsi.
R wears a red shirt.
U wears a blue shirt.
T drinks Thums up and Q drinks Coke.
The friend who wears a green shirt, the friend
who drinks Fanta and V ordered the same dish.
The friend who wears the violet shirt drinks
Mazaa.

12. Who among the following drinks Pepsi?


(1) Q
(2) S
(3) T
(4) V

But at the time of the seminar, some of the students


were missing. So the teacher had to change the order
of making presentations.

X presented before T.

R presented before V but after U.

Q presented after W, P and S

W presented the seminar before U and X but


after P and S.
14. Which among the following is the group of
students who presented their seminar in the 1st
group?
(1) W, P, S
(2) W, U, X
(3) P, S, U
(4) P, Q, R
15. Who among the following cannot be the first
person to make his presentation in the 2nd group?
(1) X
(2) U
(3) Q
(4) V
16. Seven students P, Q, R, S, T, U and V go to
tuition in different subjects-Mathematics, Physics
and Chemistry. Out of 7 students, 3 were girls. It
is further known that
R goes for Physics tuition
P and U go to the same tuition
Every tuition is attended by at least 2 people
Q who goes to Maths tuition does not go to
the same tuition as S.
T and V go to same tuition.
Who among the following definitely goes to
the tuition in which 3 students attend?
(1) S
(2) T
(3) V
(4) Q
Quant ERPV

13. Which of the following is a correct combination of


person, colour of shirt worn and the cool-drink he
ordered?
(1) V, violet, Maaza
(2) U, blue, Fanta
(3) R, red, Mountain Dew
(4) Q, yellow, Coke

Directions for question 14 and 15: P, Q, R, S, T, U,


V, W and X are students learning a foreign language.
and were divided into 3 groups of 3 each for the
purpose of presenting a seminar.
Group I
P, Q, R
Group II
S, T, U
Group III V, W, X

1. Two auditoriums are designed to be constructed


such that their floors are square shaped with the
ratio of the sides of the squares being 1 : 2. A
th
group of N workers start working in A1. After 1
6
th
3
of the workers working in A1 move
of a day,
4
over to A2 and start working. At the end of the
day the work in A1 just gets completed and the
work in A2 gets completed when N workers work
7
th of the next day as well. Find the value of
for
8
N.
(1) 24
(2) 36
(3) 48
(4) Cannot be determined

36

Line + Bar graph

Accident Severity index


(person killed per 100 accidents)

Directions for questions 1 to 4: The following bar graph gives the accident severity index and the type of
vehicles responsible for accidents in the year 2005. A total of 75000 accidents occured during 2005.
50

45

45
40

40
30

35
30

35

34

28%

26%

25

18%

20

16%
12%

15
10
5
0

Trucks

Buses

1. How many persons were killed in 2 wheeler


accidents?
(1) 6780 (2) 7200 (3) 8400 (4) 9600
2. The ratio of people killed to people who got
injured in the accidents is the highest for
(1) 2 wheeler accidents
(2) Other types of accidents
(3) Car accidents
(4) Truck accidents
3. How many persons were injured by car
accidents?
(1) 6400 (2) 7800 (3) 4200 (4) 7200
4. By how much is the number of persons killed in
truck accidents more than the persons injured in
accidents caused by other types of vehicles?
(1) 1440 (2) 1720 (3) 1480 (4) 1680

Cars

1. Which of the following policies is the 3rd to


mature?
(2) Jan 10th, 2001
(1) Feb 29th, 2000
th
(4) August 8th 1998
(3) August 8 , 1999
2. In which of the following years did Atul receive
money from 2 policies?
(1) 2000
(2) 2001
(3) 1999
(4) 2002

Others

LA(Linear Arrangement)

Directions for questions 1 to 3: Ten friends who


stay in 3 different countries among USA, UK and
Australia meet at a reunion party in their school in
India. They stand in a row according to their roll
numbers in the school. There are 5 male friends
among P, Q, R, S and T and 5 female friends among
F, G, H, I and J. 5 friends stay in USA, 3 stay in the
Australia and 2 stay in the UK.

DI (Distribution)

Directions for questions 1 and 2: Atul has 6 policy


accounts which are going to mature in between the
years 1997 to 2002 (both the years included). Two
policies have their maturity in the same month whereas
two policies have their maturity in the same year. One
of the policies matures on Feb 29th. It matures before
the policy which matures on January 10th but after the
policy which matures on August 8th. The first policy
matures on August 24th 1997. The policy which
matures on September 17th matures immediately
before the policy which matures on May 21st which
inturn is the last policy to mature.

2 wheelers

1.

No two friends from USA stand next to each


other.
P is from Australia and stands in between I and J.
The friends from UK are at least 4 places away
from each other.
rd
S is from UK and is 3 in the row from the left
end.
The extreme ends are occupied by the friends
who stay in USA. They positions are of different
genders.
F, Q, G stand in the 7th, 6th and 5th from the right
end of the row respectively. Two of these stay in
the USA.
R is adjacent to I.
How many female friends stay in the USA?
(1) One
(2) Two
(3) Three
(4) Four

2. How many friends stand in between friends from


UK?
(1) Three
(2) Four
(3) Five
(4) Six
3. Who among the following is the male friend from the
USA?
(1) Q
(2) T
(3) R
(4) Both A and B

Directions for question 4 and 5: Seven friends


Pradip, Qureshi, Raju, Shyam, Tarun, Uttam and
Vamsi stand in a row. It is further known that

37

(i)
(ii)
(iii)
(iv)

Shyam is at the extreme left end of the row.


Qureshi is two places to the right of Tarun.
There are exactly 2 persons in between Uttam
and Vamsi.
Raju has the same number of people to his left
as to his right.

1.

Which student got the highest marks?


(1) A
(2) B
(3) C
(4) E

2.

Which two students got the same marks?


(1) B and D
(2) B and C
(3) A and D
(4) C and D

4. Who is standing 3 places away to the left of


Qureshi?
(1) Tarun
(2) Shyam
(3) Pradip
(4) Raju

3. Which of the following is the descending order of


the students according to their marks?
(1) ECBDA
(2) EACDB
(3) EBADC
(4) ECADB

5. Who is standing to the immediate right of


Shyam?
(1) Qureshi
(2) Uttam
(3) Pradip
(4) Vamsi

Directions for questions 4 and 5: Five movies P, Q,


R, S, T were set to release on every Friday of a
month. P was neither the first nor the last movie to be
released. Q was released immediately before R and
there were at least 2 movies to be released before R.
There was one movie released in between S and T
(Assume that no movie was scheduled to release
during this period apart from these 5)

LA (Sequencing)

Directions for questions 1 to 3: Five students A, B,


C, D and E are arranged according to their marks
under the following conditions
C got more marks than A but less than E.
D got less marks than 3 other students.
Two students got the same marks.
The first and the last ranked students did not get
the same marks as any other student.

4.

Which of the following movies was released last?


(1) S
(2) T
(3) R
(4) Q

5.

How many movies were released before P?


(1) 1
(2) 2
(3) 3
(4) 4

Line Graph + Table

Directions for question 1: The following line graph gives the details of the production of different plants per unit
area:
50
Coffee
Steel
40
Coffee
Tea
Tea
30
Rubber
Rubber
20
Steel
10
0
1996

1997

1998

1999

2000

Profit/ton = Revenue/ton Cost/ton


The following table gives the cost and revenue obtained by these plants per ton in 2000.

1.

Coffee

Tea

Rubber

Steel

Cost/ton

`12340

`13460

`8900

`15800

Revenue/ton

``15610

`14280

`12300

`19960

Which of the following plants has the maximum profit in 2000?


(1) Coffee
(2) Tea
(3) Rubber

(4) Steel

38

SOLUTIONS FOR DATA INTERPRETATION REPLICA QUESTIONS THAT HAVE


APPEARED IN CAT IN THE LAST 4 YEARS
TABLES

108800
= 18133
6
18133 16000
percentage increases =
100
16000
 13%
Choice (3)

Average gross pay =

Solutions for questions 1 to 3:


1.

2.

3.

Let the volume of data transfer in India and Singapore


be 100 units each.
Revenue from data transfer in India = 100 1 = $100
Revenue from data transfer in Singapore = 100 9 = $900
100
Total revenue in India = 100
= $1111
9
100
Total revenue in Singapore = 900
= $4285
21
Total revenue in Singapore is about 4 times that in
India.
Choice (5)

Revenue from data transfer as a percentage of total


revenue for India in 2010 = 27%
Revenue from data transfer as a percentage of total
revenue for Sweden in 2010 = 36%
Let total revenue in India in 2010 be $200 and that in
Sweden be $100
ARDT of Sweden = $6
36
Volume of data transfer in Sweden =
=6
6
Volume of data transfer in India = 6
54
ARDT in India =
=9
6
9 1
The percentage increase =
100 = 800%
1
Choice (3)
It can be seen that if the total revenue received is the
same for the given pairs of countries, only UK and
Spain would have approximately the same volume of
data transfer.
Choice (4)

8.

As after the mutual transfer, the average age of the


Finance department increase by one, it means that the
age of the person who came from the Marketing
department was 20 years older than the age of the
person who was transferred from the Finance
department. Now after the transfer of the employee to
the HR department, as the average age of the
employees left in the Marketing department remained
the same, the age of the employee transferred to the
HR department, was 20 years younger than the
average age, i.e., 36 20 = 16 years.
The new average age of the employees in the HR
department
46 5 + 16 1 246
=
Choice (3)
=
= 41 years
6
6

9.

The new average basic pay of employees in the HR


10,000 5 + 12,000 2 + 16,000 1
department =
8
90000
50,000 + 24,000 + 16,000
=
=
= 11250
8
8
The percentage change = 12.5%
Choice (2)

Solutions for questions 10 to 13:


10. The drink must contain 10% minerals. As there are only
two drinks (A and C) with 10% minerals, the drink can
be prepared in only one way. As A and C have 30%
protein each, they can be mixed to form the drink.
Choice (1)

Solutions for questions 4 to 6:


4.

To get calls from all the colleges, Arun should have


scored at least the highest value of cut-off in each
section, i.e., 44 + 44 + 45 + 44 = 177 and also at least
the highest value of aggregate cut-off for any institute,
i.e., 176.
Choice (2)

5.

The minimum aggregate marks to get calls from two


colleges is 171. If he scores 50 each in three sections
he needs to score at least 21 marks in the fourth
section.
Choice (3)

6.

Four colleges have a cut-off for section C and the


remaining two colleges have a cut-off for section D.
If a student misses the cut-off in these two sections,
he/she would miss calls from all the colleges.
The maximum possible marks such a student gets is
50 + 50 + 40 + 42 = 182.
Choice (3)

Solutions for questions 7 to 9:


7.

The new gross pay of the employee transferred


80
= 16,000 +
16000 = 16,000 + 12,800 = 28,800
100
The gross pay of the current employees in HR
department = 16000 5 = 80000
New gross pay of the six employees = 80,000 + 28,800
= 1,08,800

11. None of the choices (1), (2) and (3) can be used to form
the drink with 10% fat and at least 30% protein. For C
and E to form the drink with 10% fat and at least 30%
protein, if they are mixed in the ratio x : y (say)
x (50 )+ (0 )
= 10, x : y = 1 : 4
x+y

1(200 )+ 4 (100 ) 600


= 120
=
5
5
Similarly the ratio for D and E is 1 : 3 and the cost per
800
unit is
= 200
4
The cost per unit is the least for C and E.
Choice (4)
cost per unit =

12. The drink should have at least 60% carbohydrate.


Further in the mixture formed by B, C and E, the
proportion of B should be maximum and the other two
should be minimum to get the lowest per unit cost.
Among the given options only Choice (2) and (5) satisfy
the condition having 60% carbohydrate and of these,
choice (5) has the lowest per unit cost.
Choice (5)
13. A and B when mixed in equal proportions, the protein
30 + 20
content will be only
= 25%, which is less than
2
required. D and E when mixed in equal proportion, the

5 + 45
= 25% which is
2
less than required. Similarly B and E and C and D when
mixed in equal proportion the combination will have less
than the required percentage of minerals and
carbohydrate respectively. Only A and E when mixed in
equal proportion would yield a mixture with all the
contents in the required amount.
Choice (5)

carbohydrate content will be only

Solutions for questions 14 to 17:


14. If one observes the values given for the different
parameters, the values that were varying with
production, i.e., value was increasing when production
increased and value decreasing when production
decreased are material, labour and operating cost of
machines. All the remaining costs, i.e., rent of building,
consumables, rates and taxes, repair and maintenance
expense and selling and marketing expenses are fixed.
Hence, there will be no change in these costs. The total
fixed cost = 1800 + 600 + 1200 + 8700 + 2100 = 14400
The cost/unit for different variable costs is as follows.
Material = `50 per unit.
Labour = `20 per unit
Operating cost of machine = `30 per unit
Total = `100 per unit
Selling price per unit = `125 per unit
14400
Total cost/unit for 2100 units is `100 +
2100
= `107
Choice (2)
15. For one product,
Selling price = `125
Variable cost = `100
_________________
Difference = `25
_________________
Now, to avoid loss, the company has to offset the fixed
cost (i.e., 14400) for which it has to produce a total of
14400
= 576 units.
Choice (3)
25
16. The reduction in selling price per unit = 5% of 125
= 6.25
New selling price = 118.75
Total fixed costs = `14400
Variable cost per unit = `100
Now the total profit increases with the increase in
number of units sold and the maximum profit is
obtained when the company sells and 3000 units.
Choice (5)
17. The given condition is that if the company sells upto
2100 units, the selling price per unit is `125 and if the
company sells 2550 units, the selling price per unit for
all the units is `120. The profit of the company
increases upto a production figure of 2100 units, from
the 2100th unit to the 2101st unit, the total profit
decreases drastically and from the 2101st unit to the
2550th unit, the profit again increases.

Hence, the profit would be maximum at the production


figure of 2100 units or at 2550 units.
Production
Selling price / unit (s)
Variable cost / unit (v)
SV
(S V) production
Total fixed cost
Total profit

2100 units
`125
`100
`25
25 2100
= 52500
14400
38100

The maximum profit is `38100

2550 units
`120
`100
`20
20 2550
= 51000
14400
36600

Solutions for questions 18 to 21:


18. The costs of a refrigerator, an air conditioner and a
music system in different countries are.

(00 U.S. dollars)

Refrigerator
Air conditioner
Music system
Total

India Thailand Malaysia Singapore USA


11 + 5 13 + 5
11 + 6
13 + 4
20
9+7
12 + 5
10 + 8
12 + 5
23
8.5 + 9 10 + 6
8+4
13 + 4
20
49.5
51
47
51
63

The cheapest is in Malaysia.

Choice (3)

19. As given in the previous question, the total cost will be


highest in India (850 + 900 = 1750)
Choice (1)
20. Cost in India = 300 + 500 = 800
Cost in Thailand =450 + 600 = 1050
Difference = 250 32.9 = 8225
Duty = 1500
Required difference = 6725

Choice (4)

21. Cost in India with dollar at `40.92 = 550 40.92


2500
22500
Cost in India with dollar at 35 =
= 650
35
Cost in Singapore = 900
Required difference = 250
Choice (2)
Solutions for questions 22 to 26:
22. Let us check the possible short routes from A to J.

Total cost

Total distance

`1470

1430 km

Rs. 625
Rs.1225
D
J
Rs. 25 km
825 km

`1850

1250 km

Rs. 850
Rs. 575
F
J
670 km
485 km

`1425

1155 km

Rs.1225
Rs. 445
G
J
675 km
485 km

`1670

1090 km

Rs. 925
Rs. 210
H
J
975 km
200 km

`1135

1175 km

Rs. 675
Rs. 215
C
395 km
205 km

`1465

1085 km

Rs. 335
Rs.1135
A
B
J
280
1150 km

Rs. 575
J
485 km

The shortest possible route is A C F J.


The cost is `1465.
Choice (4)
23. The route with the least cost is A H J, with a total
cost of `1135. As the cost of the new service is 5% less
then `1135, it should be 1135 (5% of 1135) = 1078.
Choice (2)
24. If C, D and H are closed, then the minimum cost of
Choice (3)
travel is for A F J, i.e., `1425.
Pr ice
to be as minimum as possible.
dis tan ce
It is less than 1 in only the cases A H, B J and

25. We want the

Choice (1)

C D. Considering the cases involving the above


routes.
Route

Price / Distance

AHJ

1135
1175

Taking margin of 10%


into Account
1135
10

1175
11

ABJ

1470
1430

1470 10

1430
11

It will be the least for A H J and is


=

103.2
= .88
117.5

27. Percentage of male employees in the production


288
100 = 45
Choice (2)
department =
640
28. Post graduates in the marketing department = 32
25
32 = 8
Male postgraduates =
100
Female post graduates = 32 8 = 24
Male non post graduates = 48 8 = 40
Required difference = 40 24 = 16
Choice (5)
29. Percentage of male post graduates in the marketing
32
department =
100 = 40
Choice (1)
80

1135 10

1175 11

Choice (2)

26. The cost / kilometer is the least for A H J and the


distance is 1175 km.
Choice (4)
Solutions for questions 27 to 30:

30. The number of male post graduates in the production


department = 144.
Female post graduates = 352 144 = 208
The number of male and female post graduates and
male and female employees who are not post
graduates are as follows.

With the given information we can deduce the number of males


and postgraduates in the different departments as follows:
Department
Marketing
Accounts
Production
Total

Total
80
80
640
800

Male
48
44
288
380

Post graduates
32
40
352
424

Post graduate

Non Post graduates

Male

Females

Males

Females

144

208

144

144

It can be seen that except female post graduates all


other groups (male posts graduates, male and female
non post graduates) have the same number of
employees.
Choice (3)

Solutions for questions 31 to 33:


31. The total number of bookings made is the highest in Q3 and so the average number of bookings per month is also the
highest.
Choice (C)

Month
Number of bookings

Jan
346

Feb
412

Number of deliveries

200

March
380
216
(146)

April
450
160
(196)

May
308
225
(220)

32. The values shown in the brackets are of the booking


made 2 months ago.
Number of deliveries made in August from the bookings
made in June = 200.
Number of deliveries made in December from the
bookings made in November = 278
278
Choice (A)
= 1.39.
200
33. We only need to check the revenue for quarters Q3 and
Q4.
Revenue (in `) from Q3 = (462 + 333 + 345) 43,100
= 4,91,34,000
Revenue (in `) from Q4 = (250 + 506 + 370) 44,000
= 4,95,44,000
the highest revenue is obtained from Q4 i.e.,
`4,95,44,000
Choice (B)
Solutions for questions 34 to 36:
34. The given condition occurs in the case where the
number of wins is in the range 16 18. Choice (B)
rd

35. From the given table, the 3 least percentage occurs in the
99 94
100
100 =
last row i.e., for 3133, which is
95
19
5
Choice (C)
= 5 %
9

June
359
170
(225)

July
462
159
(138)

Aug
333
296
(200)

Sep
345
134
(166)

Oct
250
50
(199)

Nov
506
125
(295)

Dec
370
278
(125)

36. The given condition is satisfied in the case, where the


number of wins is in the range 25 27 i.e.,
68 + 64 + 63
Choice (A)
= 65 = 2.5 26
3
Solutions for questions 37 to 39:
37. Investment (in `) in NLP Industries before withdrawal
= 12.5% (12,00,000) = 1,50,000
Investment (in `) in NLP Industries after withdrawal
= 16% (9,00,000) = 1,44,000
the percentage change in investment
1,50,000 1,44,000
=
Choice (D)
100 = 4%
1,50,000
38. The return on investment for Mr. Anil
2
25
=

7,00,000 = 3500
100 100
The return on investment for Ms. Shivani
2.5 10
=
13,00,000 = 3250

100 100
Therefore the required difference = (3500 3250)
Choice (A)
= `250

39. The three persons A, B, and C made an investment of


`10 lakh, `20 lakh and `21 lakh respectively such that
their investments fall under the schemes X, Y, Z
respectively.

Their combined return on investment (in ` )


= 2% (10,00,000) + 2.5% (20,00,000) + 3% (21,00,000)
= 20,000 + 50,000 + 63,000 = 1,33,000
Their combined return on investment after the firm
increased the rate of return
= 2.2% (10,00,000) + 3% (20,00,000) + 3.3% (21,00,000)
= 22,000 + 60,000 + 69,300 = 1,51,300
The required increase (in `)
= 1,51,300 1,33,000 = 18,300
Choice (D)

The franchise in Bengaluru will earn more revenue then


the establishment fees (in each of the two centres) after
one year.
Choice (D)
44. If a customer spends on an average `300 and `130 at a
Foodie restaurant in class A center and class B center
respectively, then the total number of customers who
are required to come such the revenues are not less
than the
establishment fees would be the i.e.,
132 10 5
104 10 5
+
= 44,000 + 80,000 = 1,24,000
300
130
Choice (B)

Solutions for questions 40 and 41:


40. Percentage contribution of mono speaker of the
1000
100 = 12.3%
company NOSY in 2001 =
8100
Percentage contribution of mono speaker of the
1600
100 = 17%
company BOSS in 2001 =
9400
Percentage contribution of mono speaker of the
companies NOSY and BOSS in 2003 are 13.5% and
19.6% respectively.
the percentage contribution of mono speakers of
both the companies increased.
Proceeding, similarly we observe that for no other type
of music systems of both the companies, the
percentage contribution increases.
Choice (B)
41.

Type of music system

Mono speaker
Dual speaker 1000w
Dual speaker 2000w
Four speakers 5000w
Home theatre

Percentage
contribution
in 2001
12.3%
22.22%
28.4%
17.3%
19.75%

Percentage
contribution
in 2003
13.54%
22.9%
18.75%
23.95%
20.8

The maximum change in percentage points occurs for


Dual speaker 2000W.
Choice (C)

42. The total number of cars sold by showroom A and


showroom B at the end of 7 days are 209 and 221
respectively.
209
=
100 94.5%. Hence statement is true.
221
The total number of cars sold by showrooms on odd
numbered days = 16 + 35 + 33 + 51 + 60 = 195
The total number of cars sold by showroom B on even
numbered days = 19 + 42 + 29 + 52 + 81 = 223.
90%(223) = 200.7
Statement is also true.
Choice (C)
Solutions for questions 43 and 44:
126 10 5
= 52,500 > 51,860;
240

75 10 5
< 42,500
180
a restaurant in class B center but not class A center
will earn more revenue then the establishment fee in
one year.

Bengaluru:

144 10
240

45. As no information is given regarding the percentage of


dropouts for districts R and S in few years, (A) cannot
be definitely a false statement. As no information is
given about the number of enrolments in each districts
in any of the years, statements (B) and (D) cannot be
confirmed.
(C) is definitely false because the dropout percentage of
district Q in any of the given years is greater than that of
each of the other districts. So, the overall dropout
percentage would also be the highest.
Choice (C)
46.

District
P
Q
R
S
T

Minimum number of achievements


5
1
2
0
2

total number of achievements (minimum = 5 + 1 + 2


+ 0 + 2 = 10)
Choice (B)
Solutions for questions 47 and 48:
47. The rise in temperature (in C) per hour =

Solution for question 42:

43. Hyderabad:

Solutions for questions 45 and 46:

= 60,000 < 60,200

90 10 5
= 50,000 < 50,246
180

46 25
=7
7

temperature (in C) in city Q at 10 a.m.


= 3 (10 5) + 25 = 40
Choice (D)
48. Temperature in city P at 3.30 p.m.
1
42 29
= 42
3
6
2

= 42

91
34.5C
12

Similarly,

City
Q
R
S
T
U

Temperature at 3.30 p.m.


35.5 C
37 C
38.5 C
36.66 C
33.5 C

city S has the highest temperature at 3.30 p.m.


Choice (C)
Solutions for questions 49 to 52:

P
Q
R
S
T
U
Traffic flowing from 3,346 3,752 2,536 2,620 2,952 3,060
Traffic flowing to 3,504 2,612 3,308 2,852 3,050 2,940
49. Total traffic through the route P Q = 964 + 846 = 1810.
Similarly verifying it is easy to see that the maximum
traffic flow occurs through the road connecting PQ.
Choice (A)

50. Looking at the table and relating the diagonal elements,


it is easy to see that the 2nd least traffic flow occurs
through the road connecting Q S.
Choice (D)
51. From the above table, traffic flowing from city Q is the
greatest i.e., 3752 vehicles.
Choice (B)
52. From the above table, the difference in traffic flow is the
least for city T i.e., 3050 2952 = 98.
Choice (C)
Solutions for questions 53 and 54:
53. Interest amount for Mr. A (in `) = 3,600 2.2 = 7,920
Interest amount for Mr. B (in `) = 3,800 3.6 = 13,680
The required difference (in `) = 13,680 7,920
= 5,760
Choice (B)

1
[5.6 4,800 + 6.4 4,000]
2
1
=
[26,880 + 25,600]
2
Choice (C)
= `26,240.

54. The required average =

Solutions for questions 55 and 56:


55. Let the total number of employees in company X,
company Y and company Z be x, y and z respectively.
Male employees in company x who owns both four
wheeler and two wheeler = 0.7x 0.15 (Q 45 + 65 + 5
100 = 15) = 105x
Female employees in company y who owns both four
wheeler and two wheeler = 0.3x 0.1 = 0.03x.
The total number of employees in company X who
owns both four wheeler and two wheeler
= (0.105 + 0.03)x = 0.135x
percentage of employees = 13.5%
Similarly for company Y, the required percentage is 26%
Similarly for company Z, the required percentage is 6%.
Choice (B)
56. Let the number of employees in either of the companies
be n.
The number of male employees in companies Y who satisfy
1
[100 (30 + 20)]n = 0.5n
the given condition =
100
similarly the required number of employees in company

Z=

1
[100 (20 + 10)]n = 0.7n.
100

the required percentage =

No. of Bikes Sold

RL-100
BCZ
Thunder
WB-150
Muzzle

19,500
37,500
30,000
45,000
18,000

Average selling
price (in `) 2007
40,000
25,000
31,000
20,000
52,000

2008

45,000
28,000
35,000
23,000
55,000

57. From the above table, the percentage increase in the


average selling price is the highest for WB-150.
23,000 20,000
100 = 15%
Choice (C)
20,000
58. The required average (in `)
40 + 25 + 31 + 20 + 52
=
103
5
168
=
10 3 = 33,600
5

59. The gain from the shares of company IV in 2006 was =


[ 448 + 432 2( 456)]
132 +
2
= 132 16 = 116
Choice (1)
60. We can only evaluate the return from the shares of
company III in the years 2002 to 2009. The returns
were as follows:

Year
Gain

2002 2003 2004 2005 2006 2007 2008 2009


122.5 158.5 155 150 170 148 172 195

The highest percentage increase is from 2002 to 2003


and it is 29.38%
Choice (2)
Solutions for question 61:
61. To find the median, arrange the per capita incomes in
descending (or ascending) or order.

Per capita income ($)


24,369
24,337
23,484
19,207
15,350
13,746
13,477
11,692
10,372
5,663
4,965
3,523
2,916

Country
Switzerland
Germany
United states
United kingdom
New Zealand
Swedes
France
Spain
Hong Kong
Brazil
Latherier
Mexico
Romaine

Out of the 13 countries, the median is the country


placed 7th.
France with a per -capita income of $13,477.
40% of 13,477 = $ 5390.8.
There are 10 whose per capita income is more than
$5390.8.
Choice (2)
Solutions for question 62:

0. 5 n + 0. 7 n
100 = 60%.
2n
Choice (D)

Solutions for questions 57 and 58:


Model

Solutions for questions 59 and 60:

62. Given, intra-state services accounted for 60% of total


revenues.
2880
Total revenues =
= `4800 crore.
0.60
Total revenues from non A/C general category in
intra-services is given to be 50% of revenues from intrastate services.
Revenues from non A/C general category in intrastate services = 50% of 2880
= `1440 crore
Choice (2)
Solutions for question 63:
63. The total production of the top four coal producing
countries is 2536.7 + 1039.2 + 478.2 + 393.9 = 4448 mt
The total production of the bottom four coal producing
countries is 76.7 + 76.6 + 145.8 + 174.9 = 474 mt.
474
The required percentage =
100 = 10.66%
4448
Choice (1)

Choice (B)

Solutions for questions 64 and 65:


64. Male students who were eligible for selection were A, F,
G and N and the female students who were eligible for
selection were L and O
Therefore the required ratio is 2 : 1
Choice (1)
65. x = 6 and y = 4
There fore 2x = 3y is in the correct choice. Choice (4)
Solutions for question 66 and 67:

The Total costs, Operating Expenses, Revenue and the


Profitability of the company in the five years are given in the
following table.
Year
2009
2008
2007
2006
2005

Total cost (in


`)
92200
83700
89600
96600
104000

Operating
Revenue Profitability
Expense (in `)
(in `)
(in `)
24050
104200
0.2308
21775
96600
0.2254
22000
112400
0.1957
24730
128200
0.1929
26580
130600
0.2035

66. The profitability of the company was the least in the


year 2006
Choice (2)
67. With respect to the previous year in the years 2006,
2007 and 2008 were 7.11%, 7.24% and 6.58%
respectively. In 2009 the total cost increased when
compared to 2008. Therefore the maximum decrease
was in 2007 and it was 7.24%.
Choice (2)
Solutions for questions 68 and 69:

Sections
A
B
C
D
E
Total

Score
Score less Score from
Total no of
greater than
than 45
45 to 85
students
85
28
72
24
124
15
68
36
119
18
52
28
98
29
58
47
134
30
60
35
125
120
310
170
600

68. Percentage of the total number of students getting


120
scores less than 45 =
100 = 20% Choice (4)
600
69. For the sections A, B, C, D and E, the maximum
number of students getting 48 or more in the
examination was 96, 104, 80, 105 and 95 respectively.
Thus the highest among the above values is 105.
Therefore the maximum number of students from a
section who passed in the examination was 105.
Choice (3)
Solutions for questions 70 to 72:
70. The growth in exports of the 4 companies from 2003 04 to 2004 - 05 are as follows..
3.15
Rahual & co:
100 = 25.9%
12.15
2 .1
Chandu & co:
100 = 14.9%
14.1
2 .8
Shiva & co:
100 = 30.1%
9 .3
3.32
Kanta & co:
100 = 31.9%
10.41
Hence Kanta & co has the highest growth in exports.
Choice (4)

71. The growth rate in imports of the 4 companies


from 2002 03 to 2003 04 are as follows:
6.53
Rahual & co:
100 = 127%
5.14
4 .7
Chandu & co:
100 = 40.5%
11.61
0.67
Shiva & co:
100 = 7.7%
8.72
4.05
Kanta & co:
100 = 54.3%
7.46
Hence the growth rate of imports is the least for
Shiva & co.
Choice (3)
72. Trade deficit = imports exports
The trade deficit of the 4 companies in 2004 05 are as
follows
Rahul & co: 2.11
Shiva & co: 0.9
Chandu & co: 1.07
Kanta & co: 0.6
Rahul and co has the highest trade deficit in 2004 2005
Choice (1)
Solutions for questions 73 and 74:
73. The number of employees who did not cross the cut off
for all the 5 companies are as follows.

A
B
C
D
E

No. of employees who did not cross the cut off.


120
225
100
200
275

Hence E has rejected the maximum number of


employees.
Choice (4)
74. The number of employees who got more than 90% for
the 5 companies are as follows:

A
B
C
D
E
Total

Greater than
90% marks
30
36
30
96
115
275

The required percentage is

Cut off cleared


180
225
150
400
300
1255

307
= 24.5% 24%
1255
Choice (3)

75. The sales of a company = The no .of units produced


the closing stock.
The sales of all the 5 companies in 2009 are as follows

Company
P
Q
R
S
T

Sales
10515
14310
9225
7755
11135

Hence S had the least sales in 2009.

Choice (2)

76. The sales of R in 2008 = 9000 675 = 8325


The sales of R in 2009 = 10000 775 = 9225
R had lower sales in 2008.
Choice (1)

Solutions for question 77 and 78:

Solutions for questions 84 and 85:

77.

84. The yield return of R in the years are as follows:

Expenses +
Overhead
(In `)
12000
13500
17750
16750
19000
17450
21375
923175

Family:

Total income
(in `)

Savings
(in `)

Kapoor
Khanna
Kirsten
Kumble
Khan
Kittu
Kala

147000
105000
168000
140000
165000
120000
196000

135000
91500
150250
123250
146000
102550
174625

The total savings made by all the families was `923175


Choice (2)

Hence the highest yield return is in 2021


Choice (1)
85. The yield return for Q in the years are as follows

2004
2005
2006
2007
2008

Solution for question 79:

Hence the highest percentage increase is in 2008


Choice (4)
Choice (3)

Solutions for 80 and 81:


80. The difference in the number of students studying in
government schools in all the states in 2008 are as
follows.

AP
MP
UP
Karnataka
Kerala
Tamil Nadu

Difference
1800
1600
1300
1400
2200
5200

Solutions for questions 86 and 87:


86. From the table we can easily observe that the average
marks are the highest for , X and X.
Hence these classes would satisfy the statement
"the higher the average marks, the higher are the
number of students".
Choice (3)
87. The statement the lower the number of students, the
higher the average marks can be verified through the
options.
Classes and have higher number of students, hence
they do not satisfy the statement.
The correct choice is (A)
Choice (1)
Solutions for question 88:

The maximum increase is for Tamil Nadu,


Choice (4)

88.

81. We can observe that the state which has consistent


increase in the number of students from 2007 to 2009 is
UP.
Choice (2)

82. The median of the total number of students is


18 + 17
=17.5
2
Hence A, C and F have more number of students than
the median.
Choice (2)
83. The number of failed students in each section is given
the table.
Hence the number of failed students is the highest more
in section C

No. of students failed


6
8
12
7
3
10

The number of students this year in the 6 states


Number of students
13,21,000
17,46,000
13,90,000
19,14,000
12,88,000
10,88,000

AP
UP
MP
Bihar
Assam
Orissa

Solutions for questions 82 and 83:

Section
A
B
C
D
E
F

Yield Return
1676.5
2204.5
2633.3
2000
2720
4105.3

2003

78. The increase in income of the Khan family is `3300


The decrease in expenses is `570
The increase in saving is 3300 + 570 = `3870
Choice (3)

79. The healthy drinks are S are X


The other drinks are unhealthy.
Hence the required ratio is 2 : 8 = 1 : 4

Yield Return
1557.14
1574.1
1884.6
2021.7
1595.2
1525

2003
2004
2005
2006
2007
2008

Hence MP has the third highest number of students


this year
Choice (2)
Solutions for question 89:
89.

Given Lakshmi spends 20% of the revenue earned


from each investment to maintain her house. So let us
calculate the revenue for each business in which she
invested.

Investment
Revenue

X
16.2
16.96

Y
14.5
14.72

Z
12.9
13.2

Hence the maximum profit is obtained from X


Choice (1)

Choice (3)

Solutions for questions 90 and 91:

90. The income and expenditure for the four regions in


2007 are as follows.

Income

Expenditure

Ratio

North

33.8

34.5

0.98

South

33.8

35.2

0.96

East

31.9

32.7

0.975

West

40.1

41.3

0.971

5.

100
4800 = 12,000
40
100
4200 = 10,000
Number of bears in the world =
42
The number of deers and wild bisons in South America
are 6,000 (25% of 24,000) and 5,400 (30% of 18,000)
respectively.
Number of wolves in South America
= 25,800 (4,800 + 6,000 + 5,400 + 4,200) = 5,400
total number of wolves in the Amazon forest = 75%
of 5,400 = 4,050.
Choice (B)
Number of pythons in the world =

6.

Species
Pythons
Deers
Wild Bisons
Wolves
Bears

BAR GRAPH
Solutions for questions 1 to 4:
1.

2.

The percentage growth rate in 2007 over 2006


250 190
=
100 = 31.5%
190
Had the percentage growth from 2007 to 2008 been
31.5%, the estimated revenue would have been
131.5
250
= 329
100
The required difference 329 305 = 25 (approximately)
Choice (1)

Solutions for questions 7 to 9:


7.

Year
2003 2004 2005 2006 2007 2008 2009 2010
Gap in
150 170 150 140 110 85 60 50
million USD
Absolute
percentage
13 12
6
21 22 30 17
change
The absolute value of the percentage change in the
growth rate was the highest in 2008-09. Choice (4)
4.

The growth rate in 2005 (over 2004)


135 90
=
100 = 50%
90
The growth rate in 2007 (over 2006)
250 190
=
= 32%
190
The required percentage

The percentage increase is maximum in case of


company R i.e.,

5400 4480
100 20.5%
4480

Choice (C)
8.

Since the cost of PC is same for all the companies


market share of Q in 2009
4200
=
21.76%
5600 + 4200 + 5000 + 4500
Market share of Q in 2014
110% ( 4200 )
100 = 28.37%
=
110% (5600 + 4200 + 500 )
the difference in percentage points = 28.37 21.76
= 6.61.
Choice (A)

9.

Looking at the options it is enough if we check for the


market share of S in 2006 and 2007.
Let the price per PC of A, B, C and D `x, `2x, `x and
`2x respectively.
Market share of S in 2006
2 5800
100
=
4200 + 3000 2 + 448 + 5800 2)

Year 2003 2004 2005 2006 2007 2008 2009 2010


Men
60
63 66.15 69.5 73 76.65 80.5 84.5
Women 40
44 48.4 53.25 58.5 64.5 71
78
Total
100
162.5

3.

Number in Amazon Forest


80% (4,800) = 3,840
70% (6,000) = 4,200
80% (5,400) = 4,320
75% (5,400) = 4,050
95% (4,200) = 3,990
Choice (B)

Let the number of people who used the company's


products in Asia in 2003 be 100.
The number of men and women who used the product
in the different years are

the approximate percentage growth = 62


Choice (1)
The percentage change in the gap between the revenues
from the US and Asia in the different years are

Choice (3)

Solutions for questions 5 and 6:

Hence the required ratio is the highest for the North


region.
Choice (1)
91. The states in which the per capita income increased by
more than 5% are J and K, West Bengal, Gujarat and
Maharashtra
In the remaining states the per capita income did not
increase by more than 5%
Hence the required ratio is 1 : 1
Choice (2)

50 32
100 = 35 (approximately)
50

116
100 44%
42 + 60 + 44.8 + 116
Market share of S in 2007
108
54 2
=
100 =
100
43.5 + 28 2 + 48.5 + 54 2
256
= 42.2%
Choice (A)
=

10. Let us consider the selling prices of the four models in


2004 as 3k, 4k, 5k and 6k respectively.
Selling prices in the years.

Models
P
Q
R
S

2004
3k
4k
5k
6k

2005
4.5k
6kl
7.5k
9k

2006
6k
8k
10k
12k

Sales revenue of Q in 2004 = 750 (4k) = 3000k


Sales revenue of R in 2006 = 500(10k) = 5000k

Therefore the sales revenue of Q in 2004 by


2
5000k 3000k
Choice (3)
100 = 66 %
3
3000k

Solutions for questions 11 to 13:

Car

2007
Production

2008
Sales

Production

Sales

Production

Alto

13000

8000

15000

10000

14000

9000

22000

20000

21000

18000

25000

22000

Estio

20000

18000

21000

20000

22000

16000

4.

Section
QA
LR

Choice (3)

VA
RC
DI

Choice (1)
13. The exports of Swift in
2007 = 2000
2008 = 3000
2009 = 3000
The ratio of exports to sales in
2007 = 0.1
2008 = 0.17
2009 = 0.14
The required ratio is the highest in 2008

6.

7.

Solutions for questions 1 to 6:

QA

LR

VA

RC

DI

Total

51
105
120
80
84

69
30.8
45
64
42

60
35
60
32
91

45
56
54
64
70

75
53.2
81
80
63

300
280
360
320
350

VA
50

The marks obtained by the student in the RC section is


the highest in AIMCAT 5
Choice (B)

Solutions for question 7:

PIE CHART

LR
80

Percentage change
56 34
100 > 50%
34
55.2 33.6
100 40%
55.2
36.4 24
100 > 50%
24
56 36
100 > 50%
36
62.5 52.4
100 16%
62.5

Choice (D)

Choice (2)

QA
100

The student showed the highest percentage increase in


84 51
Choice (B)
QA section i.e.,
100 = 64.7%
51

5.

12. The ratio of production to sales of Alto in


2007 = 1.625
2008 = 1.5
2009 = 1.55
The required ratio is the highest in 2007

Maximum actual score

Sales

Swift

11. The total production of all three Cars in


2007 = 55000
2008 = 57000
2009 = 61000
The total sales of all three Cars in
2007 = 46000
2008 = 48000
2009 = 47000
The required ratio in
2007 = 1.19
2008 = 1.18
2009 = 1.29
Hence the ratio is the highest in 2009

Section
Exam
AIMCAT 1
AIMCAT 2
AIMCAT 3
AIMCAT 4
AIMCAT 5

2009

RC
60

DI
100

60
100 = 120%
50
Choice (D)

1.

The required percentages =

2.

Maximum possible actual score in RC section = 60.


The least difference occurs in the AIMCATs 2 and 4 i.e.,
60 56 = 64 60 = 4
Choice (A)

3.

From the above tables, the given condition is satisfied


in AIMCAT 3 and AIMCAT 5.
Choice (B)

Given the total number of students = 1000


Dancing = 45% = 450
Embroidery Classes = 5% = 50
Singing = 20% = 200
Karate = 15% = 150
Painting = 15% = 150
Now only Boys chose Karate. Hence a total 150
students in Karate are all boys.
Only girls chose Embroidery classes. Hence a total 50
students in Embroidery are all girls.
Also 80% of students in Singing are girls.
Hence 160 students in Singing are girls and 40 are
boys.
Similarly 80% of students in Dancing are boys.
Hence 360 are boys and 90 are girls, who are in
dancing
In Painting the ratio of boys to girls is 1 : 1
Let us tabulate the data.

Dancing
Singing
Painting
Karate
Embroidery Class
Total

Boys
360
40
75
150
0
625

Girls
90
160
75
0
50
375

Total
450
200
150
150
50
1000

If Painting & Singing are mixed then the ratio of boys to


girls is 115 : 235 = 23 : 47
Choice (3)

LINE GRAPH

Solutions for question 8:


8.

Given the ratio of the number of employees in central to


state government jobs is 6 : 1.
Let the central government jobs be 600 and the state
government jobs be 100.
The number of central government employees in A.P =
150.
The number of state government employees in Kerala =
25.
The required ratio is 150 : 25 = 6 : 1
Choice (1)

Solutions for questions 1 and 2:


1.

Profit on a normal day = 7000 6500 = `500


Profit when 300 units are sold = 10,500 9000 = `1500
1500 500
Required percentage =
100 = 200%
500
Choice (C)

2.

Cost when 200 units are produced = `6500


Cost when 350 units are produced `10,000

Solutions for 9 and 10:


9.

Additional cost /unit =

The number of employees in each department of P


and Q.

HR
Academic
Operations

P
1750
10500
5250

Q
3780
3060
11160

Total number of employees in both companies in


the HR department = 5530
5530
= 15.5%
Hence the required percentage =
35500
Choice (2)
10. The number of Academic employees = 13560
The number of Management department employees
= 21940
13560
The required ratio =
= 0.618 0.62
21940
Choice (2)

Solutions for questions 3 and 4:

The energy consumption of Geyser in a week is 7 kWh and


we know the family uses the Geyser for 2 hrs in a day. Hence
for 14 hrs in a week the energy consumption is 7 kWh.
Hence the energy consumption of a Geyser per day is 1 kWh.
Now, energy consumption of Refrigerator in a week is
14 kWh and the family uses Refrigerator throughout the day.
Hence, the energy consumption of Refrigeration per day is
2 kWh.
Similarly the energy consumption for TV in a day is 2 kWh.
The energy consumption for Washing machine in a day is
1
4
kWh and for Grinder is
kWh
7
7
3.

(a) Energy consumed by TV for 3 days = 6 kWh.


Energy consumed by Refrigerator for 3 days
= 6 kWh.
Hence Choice (1) is false.
(b) Energy consumed by Geyser for 4 days = 4 kWh.
Energy consumed by Grinder for 7 days = 1 kWh.
Hence Choice (2) is false.
(c) Energy consumed by Washing Machine in a week
= 4 kWh.
Energy consumed by Geyser for 2 weeks = 14 kWh.
Hence Choice (3) is true.
(d) Energy consumed by TV for 2 days = 4 kWh.
Energy consumed by Washing machine for one
week = 4 kWh
Hence (d) is false
Choice (3)

4.

The fixed cost increased by 25 %. Hence the new fixed


1
cost is `60 +
(60) = `75.
4
Hence the increment in the total cost is
15
15
100 =
100 = 12.5%
4
120
60 + (0.35 30)
7
Choice (2)

Pie Charts + Bar Charts


Solutions for questions 1 and 2:

Number of students in each discipline is as follows:


Number of students
Discipline
Marketing
Finance
Operations
Systems
HR

Number of students
3780
1260
1008
1512
252

Number of males and females in each discipline are as


follows:

Marketing
Finance
HR
Systems
Operations
Total

Males
2079
819
1008
840
576
5322

Females
1701
441
1512
672
672
4758

Difference
378
378
504
168
144

1.

The total number of female students in the institute was


less than the total number of male students by
5322 4758
100 = 10.6%
Choice (3)
5322

2.

The difference between the number of male and female


students was the highest for HR.
Choice (1)

10,000 6500 3500


=
= `23.
150
150
Choice (B)

DATA SUFFICIENCY
Solutions for questions 1 to 4:
1.

From A, as 60% of the newly joined employees were


not managers, the remaining 40% of the newly joined
employees were managers. It is given that
10 managers had newly joined.
40% = 10 100% = 25
Hence, A alone is sufficient.
B gives no data, it is just an assumption. Choice (1)

2.

From A and the given condition, either Babu or David


got the highest rank.
Hence, A alone is not sufficient.

10

From B and the given condition, either David or Amar


can be the highest ranker. Hence, B alone is not
sufficient.
Combining A and B, David must get the highest rank.
Choice (4)
3.

4.

It is given that, 30% of the students are boys, which


implies 70% of the students are girls. Also 10% of the
girls are athletes. 10% (70%) = 7% of the students
are female athletes.
From A, 25% of the students are athletes
Hence 25 7 = 18% of the students are boys who are
athletes. So, A alone is sufficient.
From B,
Number of boys who are athletes = 120% of the girls
who are athletes.
As 7% of the students are girls who are athletes, 120%
(7%) = 8.4% of the students are boys who are athletes.
So, B alone is also sufficient.
Choice (3)
Clearly, A alone is not sufficient, as we do not know
how many points the opponent scored.
B alone is also not sufficient, as we do not know how
many points team A scored.
Combing A and B,
If the score at the half time was say 0-25, then the
match would have ended in a tie at 35-35. So, team A
did not win. Had the score at half time been, say, 10-35,
then in the end it would have been 45-35 and team A
would have won. So, we cannot answer the question
even after combining both the statements.
Choice (5)

Again yz = 21 the different possibilities are


1 21
37
But we do not know if x , y, z are natural numbers or
not. For eg if y = 9, we can get.
7
xy = 18 as 29 and yz = 21 as 9
3
There will be infinite possibilities like this, so statement
A is not sufficient.
Statement B above is also not sufficient as it gives
information regarding x and z only and nothing about y.
Combining both the statements, we can conclude that
x = 6, y = 3 and z = 7. Thus z is the maximum.
2.

Using statement A alone we have SEVEN = 19 and


FIVE = 14, without knowing anything about the values
of individual alphabets, we cannot answer the question.
Using both the statements together we can conclude
that 2 (N) + I + E = 7
N = 1 and I and E are 2and 3 or 3 and 2
So F + V = 14 5 9. F and V could be 4, 5 or 5,4.
SEVEN = 19
If E = 2 and V = 5, we get S = 9 whereas if E 3 and
V = 5, we get S = 7.
So we cannot determine S uniquely.
Thus the question cannot be answered even by
combining both the statements.

3.

The number of days that Raju's dad goes to the Shiva


365
temple in a year is
= 121 days.
3
The number of days that Raju's dad goes to the
365
Venkateshwara temple is
= 91 days.
4

PPL
1.

Let the cost of the new car be `x.


2
x.
5
From statement , we cannot answer the question as
neither the cost of the new car nor the cost of the old
car is given.
Statement alone is not sufficient.
From statement , we only know regarding his personal
saving but nothing about the cost of the cars.
Statement alone is not sufficient.
By combining both the statements, we have the
following information.
2
Amount borrowed from his friend = 60% x
5

The number of days that Raju's dad goes to the

Therefore the cost of the old car = 40% (x) =

365

Saibaba temple is
= 52 days.
7
The number of days that Raju's dad goes to both the

365

Shiva and the Venkateshwara temple is

12
= 30 days.
The number of days that Raju's dad goes to both the

365

Venkateshwara and the Saibaba temple is

28
= 13 days.
The number of days that Raju's dad goes to both the

6
32
x
x =
5 5 25

365

2
x
5
Money withdrawn from personal savings account to
meet the cost of the new car.
6
2
9
x x=
x
=x
25
5
25
9
x was what portion of his
Now, it is not known that
25
personal savings balance.
Thus the question cannot be answered even by
combining both the statements.
Choice (4)

Shiva and the Saibaba temple is


= 17 days.
21
The number of days he goes to all the three temples is

Money realised by selling the old car =

(Numbers)
1.

365
84 = 4 days.

Hence the number of days Raju's dad goes to exactly


one temple is 121 + 91 + 52 30 13 17 4
= 204 days.
Choice (1)
4.

A + B +C + C + D + E +E +F + G = A + B + C + D + E
+F +G + C + E
(1 + 2 + 3 + 4 + 5 + 6 + 7) + C +E
Now 28 + C +E = 33.
C + E = 5
C, E could be (1, 4), (4, 1) (2, 3) OR (3, 2)there are four
possible ordered pairs of (C, E).
Choice (3)

From statement A,
xy = 18 the different possibilities are:
118
29
36

11

CASELET

Solutions for questions 8 to 12:

The trading pattern followed by each of the three traders is


as follows

Solutions for questions 1 to 3:

The arrangement of the buildings according to the given


conditions is
Yellow Blue Indigo
B
E
C

Anand
Buy
Sell
10 a.m. 3 p.m.

Bala
Chandu
Buy
Sell
Buy
Sell
10 a.m., 3 p.m. 10 a.m., 3 p.m.
11 a.m.,
11 a.m.,
12 noon,
12 noon,
1 p.m.,
2 p.m.

1.
2/3.
4.
5.
6.

Height
E
B/D
A
C
F

A
F
Orange Green

The colour of the building diagonally opposite to the


yellow coloured building is Violet.
Choice (4)

2.

The second tallest building is either B or D.


Choice (5)
The colour of the tallest building is Indigo.
Choice (2)

3.

Solutions for questions 4 to 7:

Stage
As P, Q, S and T won at least one match, R and U lost all
the three matches.
As Q, S and T lost at least one match, P won all the three
matches.
In stage-, there are a total of 9 matches and so 9 wins.
Q, S and T won two matches each.
As P (the top team in stage-) did not play against
U, P played matches against Q and R.
The ninth match was between Q and U.
So the nine matches that have taken place are as follows.
Won
P
Q
P

Lost
S
T
Q

Won
S
T
P

8.

As the direction of the price movement is not known,


the profits of Bala and Chandu depends upon the prices
at which they bought gold i.e., if they buy at lesser price
than that bought by Anand, their profits would be more,
if not, the profits of Anand would be more than that of
the other two. Hence the answer cannot be determined.
Choice (5)

9.

Anand buys the entire quantity at a single point of time,


whereas each of the other persons buy once every
hour. As the direction of movement of gold is not given,
we cannot compare the returns of Anand with the other
two persons.
Bala: Bala buys the same quantity of gold every time,
irrespective of the price.
Chandu: Chandu spends the same amount every time,
his buying depends on the price of gold at the time he
buys. The more the price, the lesser quantity he buys.
As his strategy is based on prices, whenever the prices
are changing, Chandus returns will be more than that
of Bala. But if there is no change in the price of gold the
returns of Bala and Chandu would be equal. Hence no
conclusion can be made.
Choice (5)

D
Violet

1.

Lost
R
R
R

Won
S
T
Q

Lost
U
U
U

Stage-
As each team played a total of five matches, in stage , the
matches take place between the following pairs of teams.
P T, P U, Q R, Q S, T S and R U
Given that, in stage-, three teams lost all the two matches.
Given P lost both the matches in stage-
Each of T and U won the two matches.
R and S lost the two matches.
Q also won two matches.
4.

T and U defeated P (the top team in stage-)


Choice (2)

5.

Only Q, T and U won both their matches in stage-.


Choice (4)

6.

S and U won exactly two matches in the event.


Choice (5)

7.

Q and T won exactly four matches each in the event.


Choice (5)

1 p.m.,
2 p.m.

10. On a boom day, the price of gold keeps rising, hence it


will be the least in the morning. Hence, Anand who
bought all his holdings in the morning will get the
maximum profit. Between the remaining two, Bala
bought the same quantity at every time, i.e he bought
the same quantity even at higher prices whereas
Chandu spent the same amount. Hence, Chandu
bought less quantity of gold when prices were high and
more when prices were less. Hence, Chandus returns
are more than that of Bala's. Bala will have the least
returns.
Choice (1)

Let the prices of gold at different timings be as follows.


Time 10 a.m.
Price
a

11 a.m.
b

12 noon 1 p.m. 2 p.m. 3 p.m.


c
d
e
f

We will look at the additional information given:


The quantity bought by Anand at 10 a.m. is the same as the
quantity he sold at 3 p.m. As it is given that Anand lost money,
we can ignore the quantity bought/sold and can conclude that
the price at 3 p.m. must be less than that at 10 a.m.
a>f
()
Similarly the quantity of gold bought/sold by Emma in each
instance is the same and it is given that Emma made a
profit. Hence we can conclude that (c + f) > (a + d) ()
Also using similar logic in case of David, we conclude that
(d + e + f) > (a + b + c) ()
It is given that the price increased from 2 p.m. to 3 p.m.
e<f
(V)
It is given that price at 12 noon was lower than the opening
price c < a (V)
From (i) and () we can conclude that c > d (V)
From (), () and (V) we conclude that e > b (V)
Hence a > f > e > b and a > c > d a is the highest.

12

Students who opted for Finance = a + e + d + g.


(a + e + d + g) 50% = b + f + c. (1)
Students who did not opt for HR = a + e + b.

11. The price of gold was the highest at 10 a.m.


Choice (1)
12. As d < c, choice (4) is also necessarily false.
Choice (4)

a+e+d+g=

13. The different possibilities in which they could have


booked the rooms are as follows.
Case :
102
C

103
D

104
B

101
B

102
D

103
C

104
A

f=e=

Since B booked an odd numbered room, we can


conclude that as per case , B must have booked room
number 101, in which case C would have booked room
number 103.
Choice (C)
14. It is given that two girls failed in the examination. Now
we have six possibilities in which we can select the two
girls who failed. They are as follows:
Dolly
Pass
Pass
Pass
Fail
Fail
Fail

Molly
Pass
Fail
Fail
Pass
Pass
Fail

Polly
Fail
Pass
Fail
Pass
Fail
Pass

Kelly
Fail
Fail
Pass
Fail
Pass
Pass

1
T
L
T
T

2
T
L
T
L

3
T
L
L
T

4
L
L
L
T

5
L
T
T
L

6
L
L
T
T

As exactly three of them were telling the truth, only in


case it is so. Thus Molly was the person who was
lying.
Choice (B)
15.

Finance

Marketing

a = 4k

b = 3k
e=k

It is given that books D and F was read by the same person,


A and B was not read by the same person and F and C was
not read by the same person.
The different combinations in which the books were read are
as follows:

B
C
G
H

A
D
F
E

A
D
F
G

A
D
F
H

V
B
C
E
G

A
C
G
H

B
D
F
E

V
A
C
E
H

B
C
E
H

B
D
F
G

A
C
E
G

B
D
F
H

16. As Akira read books E and G, the books that Akira read
could be either A, C, E and G or B, C, E and G. In either
case, we can conclude that Aroki did not read book C.
Choice (D)
17. As books C and E, were not read by the same person,
as in cases and V, books G and H were read by the
same person.
Choice (C)

g=3k
d = 2k f = k
c=k

Solutions for questions 18 to 25:

HR
n

It is given that n = O
g = 37.5% of (a + b + c)
i.e., g =

1
g.
3

Solutions for questions 16 and 17:

Let us denote a true statement by T and a false


statement by L (lie)
Cases
Dolly
Molly
Polly
Kelly

(2)

f = e = k.
From equation (1), a + e + d + g = 2 (b+ f + c)
a + k + d + 3k = 2 (b + c) + 2k
a + d = 2(8k a) 2k.
3a + d = 14k (3)
From equation (2), we get, 4d + 11k = 5b + a (4)
Again number of students who opted for only Finance &
HR, i.e., d was 50% of those who opted for only
Finance i.e., a.
d = 50% a.
Substituting in equation (3), we get d = 2k and a = 4k.
Substituting in equation (4) we get b = 3k.
Now (a + b + c + d + e + f + g) = 15k
15k = 270 k = 18
Exactly two = d + e + f = 4k
= 4 (18) = 72 students.
Choice (B)

Case :

Cases
1
2
3
4
5
6

Again number of students who opted for only Finance


and Marketing was 331/3% of those who opted for all
three
Number of students who opted for only Marketing and
HR = Number of students who opted for only Finance
and Marketing.

Solutions for questions 13 to 15:

101
A

5
(a + e + b).
4

3
(a + b + c)
8

Let (a + b + c) be 8k.
g = 3k
Students who did not opt for Finance = b + f + c

18. W5 and W7 are allotted a shift, one earlier than W6 and


W3 and W9 are also allotted a shift earlier than W6.
Again as W3 is allotted a shift lower than W2, if we allot
the afternoon shift for W3 and W9; W5 and W7 being one
shift earlier than W6, we will have four workers in the
Afternoon shift if W6 is allotted the Evening shift. Thus
the only shift that can be alloted to W6 is the Night shift.
The following table gives the workers and the shift they
were allotted to.

13

Morning
Afternoon
Evening
Night

Case (3)
Schumi
Sebastian
Sebastian

W2
W1, W3, W9
W5, W7
W4, W6

If Schumi beats Mclaren on all the three days, then


Mclaren will come last all the three days (not possible).
Choice (D)

Thus W8 can be allotted any shift other than the


afternoon shift.
Choice (B)
19. The following table lists down the matches and the
corresponding players who led the team as captain and
vice captain.
Match
Match 1
Match 2
Match 3
Match 4
Match 5

Captain
B
A
B
C
D

Vice Captain
A/D
C
A/D
A/B/D
A/B/C

As D refused to lead the team as captain if A or B led


the team as captain in the preceding match, we can
conclude that D can be the captain of the team only in
Match 5.
Again with D as Captain in Match 5, A must have
captained the side in Match 2 for A cannot be the
captain in Match 4.
Now with A as the captain in Match 2, C must have
been the vice captain in that match.
Thus C was the vice captain in Match 2.
Choice (C)

25. (i)

Appliances
Water Purifier
Refrigerator
AC

26. Let the runs scored by Bhajji be x

Straight drive Pull shot


Others
Total
x + 40
Pollard
x + 40
5
Dumminy (0.6) (x+ 20)
(0.15) (2x + 40) x + 20
x
= 20
x
Bhajji
4

x
= 20
4
x = 80.
Given

21. Number of matches played by Sachin is equal to that


played by Mongia. Number of matches played by
Dravid is equal to that played by Hussey. Since Sachin
has played more matches than Dravid, the average
runs must be less than 45.
Choice (A)

282
= 40.28.
7
301
= 43.
Average (Sanjay) =
7
If the average score after the exclusion lies between
40.28 and 43, then the average of Ramesh will
decrease while that of Sanjay will increase. Since,
92 is the only value lying in that range, so their score in
the invalid question is 42.
Choice (A)

22. Average (Ramesh) =

23. A wins B wins


B wins C does not win.
That implies both A and C do not win together.
That means at most one of A or C wins. That further
implies that D must win.
Choice (A)
24. Case (1)
Schumi
Schumi
Sebastian
Case (2)
Schumi
Schumi
Sebasian

Pollard
Dumminy
Bhajji

Day
Monday
Tuesday
Wednesday

For A or C:
AC must be bought before the water purifier.
Choice (B)

Mclaren
Mclaren
Schumi

Day 1 / 2 / 3
Sebastian
Sebastian
Mclaren

Sebastian
Mclaren
Schumi

Day 1 / 2 / 3
Mclaren
Sebastian
Mclaren

Both Sashi and Govind work together. This implies


Ryan and Mokambo will work together
Choice (A)

Solutions for questions 26 to 28:

20. For B:
Person

Day 1 / 2 / 3
Sebastian
Mclaren
Mclaren

Mclaren
Schumi
Schumi

Straight drive Pull shot Others Total


24
120
60
10
30
100
20
80

Maximum possible difference = 95 10 = 85


Choice (D)
27. Maximum runs scored by Bhajji through straight drive = 59.
59
2
Required percentage =
100 = 19 /3%
300
Choice (D)
28. Runs scored by Bhajji through others cannot be
determined.
Choice (D)
Solutions for questions 29 to 41:
29. (1)
(2)
(3)
(4)

All shoes are pens.


Not all pens are pencils.
All pens are chocolates.
Not all chocolates are pens.

Analyzing the options:


(A) Combining (1) and (4), we get Some chocolates
are not shoes.
(B) Combining (1) and (3), we get that some shoes
are chocolates.
Choice (D)
30.
t

Rameshs house

4
5

Total time = t +

Umeshs house

4
9
t =
t
5
5

9
t = (7 : 45 4 ) pm = 225 min.
5
t = 125 min

14

When Ramesh reaches Umeshs house, his watch was


showing 4 p.m. + 125 mins = 6:05 p.m.
Umeshs watch was showing 6:10 p.m. So, Rameshs
watch is 5 mins slower than Umeshs watch.
Choice (D)

Emmanuels rank in Physics must have been 3.


Ben got the same rank in Mathematics and Chemistry.
Remaining ranks of Ben is be 4, 4 and 12. Therefore
Ben or Emmanuel did not get the 1st rank in
Mathematics. Thus Adams rank in Mathematics was 1.
Cathy got 3rd rank in Mathematics, therefore Ben got
the 4th rank in Mathematics.
Proceeding like this we can conclude that Cathy got the
1st rank and Adam got the 2nd rank in Physics
Choice (B)

31.

T2

T1

200 x x

34.

300 x

Air Conditioners

Refrigerators
y
b

24
x 300 x + y
=
y 200 x + y

Let

x
=2
y

32.

Swimming Running Cycling Walking Total


5
6
15
1
6
5
2
4
14
1
18

In case of Z:
A sum total of 18 is possible when two 6s and one 5
is there in addition to 1.

It is given that at least 40 families own both a


Refrigerator as well as a Air conditioner.
b + d is at least 40.
We have to find the maximum value of a.
a will be maximum when (b + d) is minimum.
i.e., when (b + d) is 40.
Now a+ c+ (b + d) + 24 + 20 + 26 = 120.
a + c = 120 70 (b + d)
a + c = 10.
The maximum value of a is 10, when c is 0.
Thus at most 10 families owes a refrigerator and a LCD
TV but not an Air Conditioner
Choice (A)
35. The lectures and the days on which they deliver the
lectures are tabulated in the following figure.

Lecturers
Swimming Running Cycling Walking Total
2
5
2
6
15
1
6
5
2
4
3
14
6
6
1
5
18

The above table gives the ranks obtained by the four


persons in the four events.
In case of X, a sum total of 6 is possible only if
1 + (1 + 3 + 1) is there. He has to get rank 3 in cycling
and rank 1 in each of running and walking.
Choice (A)
33.

Name
Adam Ben Cathy Dimitry Emmanuel
Subjects
Mathematics
Physics
Chemistry
Biology
Total

LCD TVs

then y > 100 and x > 200. This is not possible so only
one value exists
Choice (A)

W
X
Y
Z

20

26

400 2y = 300 y
y = 100
x
If
>2
y

W
X
Y
Z

1
2
3/5
5/3
11

4
5
4
2
15

3
1
5/3
3/5
12

2
4
1
1
8

5
3
2
4
14

Since the sum of the ranks of Dimitry was 8 and he got


the same rank in Chemistry and Biology, his ranks in
Chemistry and Biology was 1.
Therefore Dimitrys rank in Mathematics was 2 which
was the same as Emmanuels rank in Chemistry.

Monday
L1

Tuesday Wednesday Thursday


L4
L2
L1
L3

L4 can deliver the lecture either on Tuesday or on


Wednesday.
Now, if L4 delivers his lecture on Wednesday, then L3
cannot deliver his lecture on any of the given days.
[Since L3 delivers a lecture only if L2 delivered a lecture
on the preceding day and L3 and L4 do not deliver
lectures on consecutive days.]
Thus L4 delivers the lecture on Tuesday.
Now, the only day on which L3 could have delivered the
lecture was Thursday
Choice (D)
36. The different ways in which the committee can be
formed is as follows:
1. B2 B4 B5 G2 G3
2. B1 B4 B5 G1 G3
3. B1 B4 B5 G2 G3
4. B1 B3 B5 G1 G3
5. B1 B3 B5 G2 G3
6. B2 B3 B5 G2 G3
Therefore there are six ways in which the committee
can be formed
Choice (D)
37. In order to have the total machining time as minimum,
none of the machines must be idle at anytime and the
total time taken must be 10 hours. (i.e., higher of the
total machining times in the two machines).
Let us consider the answer options and check if it is
possible.
Option A

15

M1

M2

P3

P1

Case 1 M9 belonged to Germany.

Duration
2
3

P1

P2

P2 (4)

P3 (5)

UK Germany France Switzerland Turkey


M6
M9
M2
M1
M7.
M8
M10
M4
M3
M5
Case 2 M9 belonged to Turkey.
UK
M6
M8

Total = 10
Option B

Germany France Switzerland Turkey


M5
M2
M1
M7
M10
M4
M3
M9

(As M5 did not belong to France or Switzerland)


Thus M10 belonged to Germany
Choice (C)

M1

M2

P1

P2

P3

P1

P2 (4)

P3 (5)

Duration

Total = 10
In case of option (C), if product P2 is machined in M1
before Product P1, since P2 takes 4 hours in M1, it can
be done as follows:
Duration 0 4
47
79
M1
P2
P1
P3
Duration 0 5
57
7 10
M2
P3
P2
P1
The total time taken is again 10 hours
Choice (D)
38.

Football

Cricket
a
d

41. As per the conditions given the different ways in which


the terms for the two contest can be selected as
follows:
Debate: PVQ PVT PVS PVR PVT PVRPVR PVS PVU PVU PVR
Elocution:PURPUR PURPUT PUS PUSPUT PUT PQS PQT PUQ

e
g

40. It is given that sum of the costs of the gifts bought by


Sneha and Sushma was equal to the cost of the gift
bought by Shikha.
We have four possibilities which satisfies this.
Sneha Sushma Shikha Sushmita
Case 1.
800
1200
2000
2800
Case 2.
1200
800
2000
2800
Case 3.
800
2000
2800
1200
Case 4.
2000
800
2800
1200
Again the difference between the cost of Sushmas gift
and Sushmitas gift was equal to the cost of Snehas
gift. This is satisfied only in Case 3. Thus the cost of the
gift bought was Shikha was `2800 and she bought a
pair of shoes.
Choice (D)

Debate: PVS PVT


Elocution:PUQ PUQ
Option A is false as can be seen in the following cases:
Debate: PVR PVS PVT
Elocution:PUQ PUQPVQ
Option B is true. If V and U are in the same category it
must be for Debate. We know that U being in debate
implies R is not in elocution. Again since only one
among S and T can be selected for a particular
category, Q must be selected.
Option C is true as can be seen in the following cases:
Debate: PVR PVS PVT
Elocution:PUQ PUQPVQ
Thus only statement given in option A is false
Choice (A)

e
Hockey

Those playing exactly 3 games = g


Those playing exactly 1 game = a + b + c
Those playing exactly 2 games = d + e + f
Those playing at least 2 games = d + e + f + g
It is given that d + e + f + g = 18 (1) and
(a + b + c) + (d + e + f + g) = 30 (2).
Therefore a + b + c = 12.
Now (a + b + c) = 3 (g).
or, 3g = 12
or, g = 4.
Therefore d + e + f = 18 4 = 14.
Thus the number of members playing exactly two
games is 14
Choice (B)

NETWORKS
1.

The cost incurred will be minimum when the distance


travelled is the minimum.
The distance travelled is minimum when he takes a bus
going via A E D F G H.
The minimum cost incurred by him = 5 + 8 + 4 (10)
= `53
Choice (3)

2.

If the road connecting A to E is under repair, then to


incur minimum cost, one must board a bus going via the
route A D F G H. Since the total distance travelled
along this route is the least.
The cost incurred = 5 + 8 + 4 (12) = `61
Choice (3)

39. We can list down the names of the countries and the
athletes belonging to them as follows.
Countries UK Germany France SwitzerlandTurkey
1.
M6
M7
M8
2.

It is given that M6 and M8 belonged to UK where as M7


belonged to Turkey.
Now M5 and M9 were from different countries and M9
did not belong to France or Switzerland.
So M9 belonged to either Germany or Turkey.
Now M1 and M3 belonged to the same country and so
did M2 and M4.
Let us consider two cases.

QBR (Miscellaneous)
1.

Given that the number of people rightly reported is 275.


This includes people under C3 and C4.
C3+ C4 = 275 .......... (1)

16

Number of people wrongly reported = 450 275 = 175


C1+ C2 = 175 . (2)
Given the number of infected people is 50% that of
non-infected
Number of infected people = 150 = C1 + C4 . (3)
And the number of non-infected people = 300
= C1 + C3 . (4)
Required difference is between C2 and C4, obtained
by (2) (3) C2 C4 = 175 150 = 25 Choice (2)

Given that for every `2 increase in the selling price


per ball, the number of balls sold decreases by 20.
If the selling price of each ball is increased k times,
selling price = `59 + 2k.
Profit per ball = (59 + 2k) 50 = 9 + 2k.
Number of balls sold = 700 20k.
Profit obtained = (9 + 2k) (700 +20k).
Profit = 10 ((9 + 2k) (70 2k))
= 10 (630 18k + 140k 4k2)
= 10 (630 (4k2 122k))
2
2

61

61
= 10 630 2k

2
2

61
61

= 10 630 + 2k

2
2

61
61
Profit is maximum when 2k
= 0 2k =
2
2
But since k is an integer, 2k must be an integer.
60
62
2k can be taken to be
or
2
2
If 2k = 30, profit = 10 ((9 + 30) (70 30)) = 10 39 40
= `115600
If 2k = 31, profit = 10 ((9 + 31) (70 31)) = 10 40 39
= `115600
For maximum profit,
Selling price = 59 + 2k = 59 + 30 = `89 or 59 + 31 = `90
When selling price is `89, balls sold = 700 300 = 400
When selling price is `90, balls sold = 700 320 = 380
Of the given choices, only (A) satisfies. Choice (1)

2.

3.

4.

From the choices only option (a) indicates low per


capita income and low happiness quotient.
Choice (1)

5.

From the choices only option (b) indicates high


happiness quotient and high per capita income.
Choice (2)

Solution for question 6:

DI (Miscellaneous)
1.

Solutions for questions 4 and 5:

Total yield from scheme = 0.25 (3) + 0.55(80) +


0.2(100) = 7.5 + 44 +20 = 56.5
Therefore the total yield from scheme II was also 56.5.
Let the probability of the bearish market be p.
The probability of the bullish market = 1 0.4 P
= 0.6 P
Now, p ( 10) + 0.4 (60) + (0.6 p) 100 = 5605
110 p = 60 + 24 56.5
p = 0.25
Choice (3)
Scheme 1
Market
conditions

Probability

Yield percentage

Bearish

0.2

30

Steady

0.45

80

Bullish

0.35

100

The yield from scheme = 0.2 ( 30) + 0.45 (80) + 0.35


(100) = 6 + 36 + 35 = 65
Increase in the total yield from scheme 1
65 56.5
Choice (4)
=
100 = 15%
56.5

6.

Written

WE

Interview

Essay
writing

GD

Rahul

Ramya

The cumulative score of Rahul is


5 0.3 + 3 0.1 + 2 0.25 + 3 0.1 + 4 0.25= 3.6
The cumulative score of Ramya is
5 0.3 + 1 0.1 + 3 0.25 + 4 0.1 + 3 0.25 = 3.5
The required difference is 0.1
Choice (1)
Solutions for 7 and 8:
7.

8.

The expected pay-out for Raju is


80 0.5 + 40 0.3 20 0.2 = 48

Choice (2)

The expected pay out for Ramu is


80 0.5 + 60 0.3 20 0.2 = 54
After the change of probability the expected pay out for
Ramu is 80 0.3 + 60 0.5 20 0.2 = 50
4
The required percentage decrease is
100
54
= 7.4%
Choice (1)

Solutions for question 9:


9.

The hotel cost for Ramu = $600


The cost incurred for city tour = $40
The cost incurred for tour of the Hunters valley = $35
Hence the total cost incurred by Ramu = $675
Choice (3)

10. Given that there has to be a male in every group.


Hence only three groups can be formed.
Also given P, S are in same group and each group has
atleast one JSE and one SSE.
As both P and S are JSE. The team should have one SSE.
Given R is in a group of 3 people
Hence the three groups should have 3, 3 and
2 members in each.
Now considering the condition one JSE and one SSE in
each group we get the following possibilities.

Group 1:
Group 2:
Group 3:

(i)
R XY
ZPS
WQ

(ii)
RXY
ZQ
WPS

From the above possibilities we can conclude that X


should definitely be a member of a group which has 3
people.
Choice (1)

17

Solutions for questions 11 and 12:


11. Let the value of the number in column 'b' and row 'd' be
'x' and that in column 'b' and row 'b' be 'A'.
1
(A + 26 + x)
Given x =
3
1
2x = A + 26 x = ( A + 26)
2
As we know the grid contains only integers
Therefore x should be an integer.
Hence A should be even.
From the choices only 16 is possible Choice (4)
12. Given the numbers in column 'a' are squares of the
prime numbers, starting with the first odd prime number.
Hence the numbers should be 32, 52, 72. 112 and 132.
Thus, the required sum is 32 + 52 + 72 +112 + 132
= 373
Choice (4)

17. By observing we can easily find that R and S have the


same visibility index.
Choice (3)
Solutions for question 18:
18. The points of A = 1 30 + 2 20 + 2 10 + 1 5 = 95
The points of B = 2 30 + 1 20 + 1 10 + 3 5
= 105
The points of C = 3 30 + 2 20 + 1 10 + 1 5
= 115
The points of E = 1 30 + 2 20 + 1 10 = 80
The winner is C
Choice (3)
Quant SI CI
1.

Solutions for questions 13 and 14:

Given total number of sarees = 400


The ratio of Kanchipattu, Benarasi and Mangalgiri sarees is
5:3:2
The number of Kanchipattu sarees = 200
The number of Benarasi sarees = 120
The number of Mangalgiri sarees = 80
Given on day 1 he sells 20% of the total which is 80 on day
2 he sells 200 and on day3 he sells 120.
Also on each day he sells the sarees in the same ratio as he
bought i.e 5 : 3 : 2
K
40
100
60
200

Day1
Day2
Day3
Total

B
24
60
36
120

M
16
40
24
80

Total
80
200
120
400

13. On the 3 day he sold Benarasi saree at `480.


Total amount received by him on the 3rd day is
60 350 + 36 480 +24 375 = `47280
Choice (2)
rd

2n

10

10000.
Interest from scheme 3, 3= 10000 1 +
200

For n = 8, 3 = `11829 & 2 = `12,000


For n = 9, 3 = `14066 & 2 = `13,500
Choice (2)
After 9 years, 3 > 2.
2.

Solutions for question 15:


15. The population of China in 2009 = 1.6 billiion
As it increases by 12% per annum it becomes
5.56 billion in 2020
The population of China in 2020 is 15% of the total
population
5.56
Hence the total population is
= 37 billion
0.15
Choice (2)
Solutions for questions 16 and 17:
16. The graph gives visibility index of 26 people.
The visibility index of 14 people are more than U.
14
Hence the required ratio is
100 = 53.8 54%
26
Choice (2)

Scheme 1 Scheme 2
80
150
166.4
300
259.7
450
360
600
469
750
587
900
714
1050
851
1200

Scheme 3 Scheme 4
102.5
80
216
166.4
340
259.7
477
360
629
469
796
587
980
714
1183
851

In scheme 4, amount at the end of the year = 1000 (1.2)


= 1200.
Amt. remaining after paying the administrative charges
= 0.9 (1200) = 1080
Amt. at the end of the second year = 1080(1.2) = 1296
Amt. remaining after paying the administrative charges
= 1296 (0.9) = 1166.4
This scheme is similar to scheme 1.
Therefore scheme 2 produces the maximum interest at
the end of 8 years.
Choice (2)

1
(400) = 100
4

The ratio of the damaged sarees of each type is 5 : 3 : 2.


Hence the number of damaged Kanchipattu sarees
= 50
The number of the damaged Benarasi sarees = 30
The number of the damaged Mangalgiri sarees = 20
He sold all the damaged sarees at 20% loss.
Total amount = 50 280 + 30 320 + 20 300
= `29600
Choice (3)

Let us consider the interests received by him from the


four schemes across the year with `1000 invested in
each scheme.

Year 1
Year 2
Year 3
Year 4
Year 5
Year 6
Year 7
Year 8

14. 25% of the total number of sarees were slightly


damaged.

Let the number of years after which his interest in


scheme 3 will be more than his interest from scheme
2 be n.
15
Interest from scheme 2, 2 =
(10000 ) n = 1500n.
100

LA (Venn Diagram)
1.

Let the number of students who applied for all three


examinations be x.
The number of students who applied for at least 2 of the
3 examinations = 36 2x
FMS

CAT
9x
x
15x

12x
XAT

It is given that 25% (36 2x) = x


36 2x = 4x
x=6
So the completed venn diagram will be as follows.

18

We have the following information.


The Hindu was read by 64 families.
A + d + e = 55 (1)
The times of India was read by 48 families.
B + d + f = (2)
The Telegraph was read by 45 families.
C + e + f = 36 (3)
Adding equations (1), (2) and (3), we get A + B + C + 2
(d + e+ f) = 130
Again A + B + C + d + e + f = 90
d + e + f = 40
A + B + C = 90 40 = 50
Therefore exactly one newspaper was read by 50 families.
Choice (3)

FMS

CAT
3

18
9

11
XAT

Number of students in the class = 18 + 3 + 6 + 9 + 6 + 11


= 62
Choice (3)
Solutions for questions 2 and 3:

Speak in
English (500)

Speak in Hindi (480)


e=
240

d = 120 t =
60
40
C = 180

Own a car (400)


As per the data provided in the question,
A + e + d = 360 (1)
B + e + f = 380 (2)
C + d + f = 280 (3)
d + c = 24 (4)
(3) (4) gives f = 40
Again d + 120 = 180
d = 60
c = 180
From (1), A + e = 300 and from (2), e +B = 340.
Now A + e + e + B = 400 [ the total number of persons
= 800]
A + e + e + B = A + e + B + e = 300 + 340.
e = 240
A = 60 and B = 100
2.

The number of persons who can speak in both Hindi


and English = 240 + 120 = 360
360
The required percentage =
100 = 72%
500
Choice (3)

3.

The proportion of people in the locality who do not own


B+e+ A +d+C
a car or cannot speak in English =
800
640
= 0.8
Choice (4)
=
800

4.

A + B + C + D +E +F = 120 (21 + 9) = 90
The Times
of India

The Hindu

A
e

9
f
C

The Telegraph

21

Solutions for questions 5 and 6:

Given GT = 300 1
GT 300
a + d + f + g = 60 2
H
S
b + d + e + g =120 3
b
c + e + f + g = 180 4
d
a
a+d=f+g5
g
e
f
a=f=06
c + f = 30 7
c
Equations 5 and 6 d = g
n
C
Equations 6 and 7 c =30
Using GT formula a + b + c + d + e + f + g + n = 300
0 + (b + d + e + g) + 30 + 0 + n = 300
n = 60
Using equations 2, 5 & 6, we get d = g = 30.
5.

g
30
100 =
100 = 10 %
GT
300
Choice (2)

From 4 and c = 30, we get e = 120


Substituting in 2, Eqn. we get b = 30
Therefore b + n = 90.

6.

Choice (4)
7.

Given, of the 300 students, 70 choose MS


230
Hence 230 choose MBA.
Given g = 20 1
M
F
n=0
b
d
a + d + f + g = 100 2
a
g
b + e + d + g = 150 3
e
f
d = 2f = g
d = 20, f = 10 4
c
From 1, 3 and 4, we get
n=0
H
b + e = 110
GT = a + b + c + d + e + f + g + n
230 = 100 + b + e + c
c = 20
Choice (3)

8.

Given the number of children who buy T & J (A) = 16


The number of children who buy C & H (B) = 26
The number of children who buy B & B (C) = 34
We know A + B + C = Ex1 + 2Ex2 + 3Ex3
Where Ex1, Ex2 and Ex3 denote the number of children
buying exactly one, exactly two and exactly three toys
respectively
Given every child buys exactly 2 toys
Hence Ex1 = Ex3 = 0
76 = 2Ex2
Ex2 = 38
Hence there are 38 children who visited the shop
Choice (4)

Solutions for questions 9 and 10:

Given P = 24 (1)
Q = 36 (2)

19

R = 29 .. (3)
P
a e b Q
S = 25 .. (4)
e + l + o + n = 12 .. (5)
f + l + k + o = 18 (6)
R
f
l g
g + m + l + o = 16 .......... (7)
c
K = 8 and l + o = 6 .. (8)
k o m
h
From (5) & (8) we get
i
j
n
d S
e+n=6
P
From (1) & (6) we get
a+e+f+l+k+o+j+n=
24
a+e+n+j=6
but e + n = 6 a = j = 0
As the people who like S also like R.
Hence d = i = n = j = 0. h + m + k + o = 25
Therefore e = 6 and b = 4.[As b + (g + l + m + o)= 36, where
(g + l + m + o) = 16]
From (6) & (8) f = 4
From (7) & (8) g + m = 10
From (3) c + h + g +m + l + o + f + k = 29
c + g + l + f + 25 = 29
c=g=l=0
Therefore h = 1, m = 10 and o =6
9.

Therefore the difference between the highest marks


obtained is 20.
Choice (1)
4.

Correct
Wrong
Score

Stores

Ranking as
per price.

A
B
C
D
E
F

3
6
1
2
5
3

5.

6.

Solutions for Questions 2 to 4:

The least score obtained by a person in the four papers


can be obtained as follows.

Correct
Wrong
Score

Paper 1 Paper 2
4
3
0
1
60
45

Paper 3
1
3
20

Paper 4
4
0
80

Therefore the minimum net score that the person can


get is 165.
Choice (4)

Paper 1
2
2
20

Paper 2
4
0
110

After the given swapping the final arrangement will be


as follows:
1

After the given swappings the final arrangement is as


follows:
1

7.

The proportion of residents who prefer watching movies


PA is 0.65
The proportion of residents who prefer surfing the net,
PB is 0.68
The proportion of residents who prefer doing both, PAB
is 0.61
The proportion of residents who prefer at least one
between watching movies or surfing net is PA + PB
PAB
= 0.65 + 0.68 0.61 = 0.72
Proportion of residents who neither watch movies nor
surf net is 1 0.72 = 0.28

8.

Let the population of A be 3k.


Population of B is 5k, that of C is 3k and that of D is
4k.

Paper 3
4
0
100

Paper 4
1
2
30
Total score = 140

Solutions for questions 7 to 9:

Total score = 120


Correct
Wrong
Score

From the choices only "W is in the 9th seat" is correct.


Choice (2)

3.

Correct
Wrong
Score

Choice (2)

Therefore X will be seated beside U

Therefore pizza store D got the third lowest net score.


Choice (2)

2.

Paper 4
4
0
80

The final arrangement of the persons in the 9 seater van


was as follows:

Ranking of the stores and the total net scores for the
pizza stores are as follows.
Ranking as
per delivery
Total net score
time
3
3(0.7) +3(0.3) = 3.0
2
6 (0.7) +(0.3) = 4.8
1
1(0.7) + 1 (0.3) = 1.0
6
2(0.7) + 6(0.3) = 3.2
3
5(0.7) + 3 (0.3) = 4.4
5
3(0.7 + 5(0.3) = 3.6

Paper 2
2
2
10

Solutions for questions 5 and 6:

LA (Miscellaneous)
1.

Paper 1
4
0
60

Total score = 150

The number of people who like only Q = 14


Choice (4)

10. The number of people who like all 4 movies is 6


Choice (2)

The least score is obtained when the person attempts


the following 3 papers.

20

No. of residents who prefer chatting with friends


in A is 0.36 3k = 1.08k
in B is 0.45 5k = 2.25k
in C is 0.32 3k = 0.96k
in D is 0.25 4k = 1.00k
Highest number is in B
Choice (2)
9.

No. of residents who prefer chatting with friends was


calculated in the previous question.
The average number of residents who prefer surfing net
1.9k + 2.75k + 2.04k + 3k
= 2.4275k
=
4
2 colonies have more than the average number.
Choice (3)

10. The different ways in which oil can be transferred from


tank B to tank H are
1. B E A F D C G H
2. B E A F G H
3. B E C A F G H
4. B E C G H
5. B E D A F G H
6. B E D C A F G H
7. B E D C G HS
Thus there are seven possibilities in all.
Choices (c)

Hence the probability of Rajini taking something home


4

is

C1 2
= [since all the baskets are equally likely to
C1 3

get selected by Rajini]


Solutions for question 14:

14. Let us consider P = 1. When there is one goat and one


tiger then the tiger eats the goat and gets transformed
into a goat and stays happily in the forest.
Now if P = 2 when there are two tigers. Now, if one of
the tigers eat the goat then it gets transformed into a
goat and then the second tiger would kill it. Hence when
two tigers are there they would not kill the goat.
Let P = 3. When three tigers are there. One of the three
tigers kills the goat and becomes a goat. Hence the
remaining tigers would not kill the goat.
Hence when the tigers are odd numbered then they
would kill the goat, else the goat is not eaten by any
tiger.
Choice (2)
Solutions for questions 15 and 16:

Given a + g = 12
b+g=8

GT
T

11. Based on the conditions given in the question, we get


the following possibilities.

Arun
Varun
Kiranmala
1. India-Day-to-Day India-Every-day India-These-days
2. India-These-Days India-Day-to-Day India-Every-day
We can conclude that Varun did not subscribe to
India-These Days.
Choice (3)
12. If Kiranmala did not subscribe for India-These Days,
then Varun subscribed for India-Day to Day.
Choice (1)
Solutions for question 13:
13. Given out of the six baskets four baskets have either
gold or silver.

Choice (2)

P
g

b
n

15. g = 4
The number of days that Ram learnt an instrument is
a + b + g = 16
Choice (4)
16. Given a = 6
Hence g = 6 and b = 2
He learnt an instrument on a + b + g i.e 14 days in all.
Choice (3)
17. From the given pattern we can understand that it is a
cyclic pattern.
Hence the input is repeated in every 7th step.
Therefore step 28 would be the same as the input
Choice (4)

LA (Circular Arrangements)
1.

The sitting arrangement was as follows

B
F/H

A/H

H/A

H/F

Therefore from the above possibilities we can conclude that H is sitting opposite to C or D.
2.

From option (a)


T

W
Q

U
V

Y
X

Choice (4)

U cannot see P, W and Q, so option (a) is not correct


From option (b)
P
T
W
Q

R
Y
X

21

All the conditions are satisfied, so (b) can be the answer

Hence we can say that the Mumbai co-ordinator is opposite


the Delhi co-ordinator.
Choice (1)

From option (c)


P

Solutions for question 4:

Given R and T sit together. We can arrange them in 2


ways.
P and S do not sit together, so we can arrange them in
6 ways.

4.

V
U

X
Y
T cannot see y, so option (c) is not correct
From option (d)
P
T
Q
W

T/R
R/T

S
S

V
Y

T/R

Bring the line below the diagram U cannot see P, so


Choice (4) is not correct
Choice (2)

T/R
R/T

R/T
P

Solutions for questions 3:

Using the first clue we can draw the figure as follows:

H
T/R
R/T
We can arrange the remaining two employees Q & U
in 2 ways.
Therefore, the total number of ways = 2 6 2
= 24 ways

M
B

Alternate solution:

Considering R and T as a single unit, we get 3 units


(R, T), Q and U which can be arranged around the
table (3 1)! 2! ways [2! since RT can be arranged
among the themselves] Now the remaining 2 persons
can be placed in 2 of the 3 positions in 3C2 2! ways.
Therefore the total number of arrangements = (3 1)!
Choice (B)
(2!) (3C2) 2! = 24 ways.

Using the 2nd clue we get,

M
B
LA (Distribution)
Solutions for questions 1 to 3:

From the given data we can conclude that the twelve persons were living in the building as follows.

Guitarist Singer Instrumentalist Drummer Singer Drummer


F

Floor 1

Floor 2

Floor 3

E/B

G/E

Floor 4 Floor 5 Floor 6

Key board
Singer Guitarist Guitarist Singer Instrumentalist
player
Q
Floor 7

Floor 8 Floor 9 Floor 10 Floor11

A
Floor 12

22

1.

The four singers were S, T, C and P. Therefore only


one male singer was there in the band. Choice (1)

2.

Above the floor in which G lived there were 6 floors or 8


floors.
Choice (4)

3.

S lived in the second floor.

on November 15th, 1998.and the one who got married


on January 3rd, 1999. And it was 49 days. Choice (1)
6.

Based on the conditions given, the groups are as


follows:
Jalan, Kokila and Kadambar
Jagan, Kavya and Kavita, Jeevan, Kavya and Kekul
Therefore Jagan is in the same group as Kavya and
Kavita
Choice (1)

7.

Given B went to college on Thursday and did not teach


Physics. As A, B and C did not teach Physics, we can
conclude that D taught Physics.
As A and C went to college on consecutive days it can
be either on Monday and Tuesday, or Tuesday and
Wednesday.
They cannot go to college on Tuesday and Wednesday
as Physics is taught after Chemistry.
Hence A and C go to college on Monday and Tuesday
respectively.
Biology is taught on Monday.
Choice (1)

Choice (2)

Solutions for questions 4 and 5:


4.

From the given data, the only possibility is


1993 November 5th
1994
1995
1996 February 29th
1997 April 23rd, October 15th
1998 November 15th
1999 January 3rd.
Both the couples who got married in the same month
got married in the month of November. Choice (3)

5.

The least difference between the marriage dates of any


two couples was between the couple who got married

Solutions for questions 8 and 9:

Given Team I scored maximum number of points, 364 is at the 15th place and Team A got 361 points.
Given the sum of the points scored by teams at (13 + 14 + 15) is 1046.
364 + 361 + x = 1046
x = 321
Hence Team A is in the 14th place.
As team C got 218 points and is in the 10th place and Team O got 251 points.
Hence Team O should be placed between 11 -13.
Given the ascending order of teams according to their points is O J F.
Hence Team O is in the 11th place, team J is in the 12th place and Team F is in the 13th place with 321 points.
Now we know the points of the teams in the 10th and the 11th place. Hence the points of the team in the 12th place is 284 points
(from 10 + 11 + 12 = 753).
As Team N and team B got 108 and 165 points respectively less than team F. Team N got 213 points and Team B got 156 points.
The clue 7 + 8 + 9 = 590 implies that we can calculate the points of the third team in the above group.
[We know N is one of the teams in the group]
Hence 213 + 182 + x = 590.
x = 195 points.
Hence Team N is in the 9th place.
The clue Team E got 18 points less than Team N, implies that team E is in the 8th place.
From the clue 4 + 5 + 6 = 412, we know B is one of teams in the group.
Hence 116 + 156 + x = 412
x = 140
Hence Team B is in the 6th place.
The clue Team H got 4 points more than Team M, implies that
Team K is in the 5th place.
From the clue 1 + 2 + 3 = 302, we get
96 + x + 4 + x = 302
x = 101
Hence Team M got 101 points and is in the 2nd place.
Therefore Team H got 105 points and is in the 3rd place.
The final arrangement is as follows.
Team
Points

1st
L
96

2nd
M
101

3rd
H
105

4th
D
116

5th
K
140

6th
B
156

8.

The required difference is 284 140 = 144


Choice (2)

9.

The position of team E is 8th .

7th
G
182

Choice (2)

Solutions for questions 10 and 11:

Given P wears the Orange shirt and he orders Sprite and


the person wearing the Green shirt orders Pepsi.
From the clues, we know R wears the Red shirt, U wears the
Blue shirt, T drinks Thumsup and Q drinks Coke.
Hence R, Q, T and U neither wear Green nor order Pepsi.
So the Green shirt is worn either by S or V. But from the
clue, the person wearing Green shirt is the person who

8th
E
195

9th
N
213

10th
C
2182

11th
O
251

12th
J
284

13th
F
321

14th
A
361

15th
I
364

orders Fanta and V orders the same dish. Hence we know


that V does not wear the Green shirt. Therefore S wear the
Green shirt and he orders Pepsi.
Let us tabulate the data.
P
Q
R
S
T
U
V

Shirt
Orange
Red
Green

Drink
Sprit
Coke
Pepsi
Thums up

Blue

23

It is given that the total work required to complete work


3 5
7 36
in A2 =
N D + N D =
(ND)
4 6
8 24
This will be true for infinite values of N.
Thus a unique value of N cannot be determined from
the given information.
Choice (4)

Now from the last clue the person wearing the Violet shirt
ordered Maaza.
Hence V ordered Maaza and is wearing the Violet shirt.
10. S drinks Pepsi.

Choice (2)

11. V Violet shirt, Maaza is the correct combination.


Choice (1)

Line + Bar graph


Solutions for questions 12 and 13:
Solutions for questions 1 to 4:

From the first and the last clue we get the following
arrangement.

O
SA

1.

L
SL

From the third clue we know that the cricketer from India is
at the extreme left end of the row and he plays for RCB.
Also P is to the immediate right of him who plays for DC.

P
IND
RCB DC

O
SA

2.

L
SL

nd

Now from the 2 clue we know that N is to the immediate


left to the player who plays for KKR.
From the above consideration we get the final arrangement
as

M
IND,
RCB

P
N
AUS, ENG,
DC
MI

O
SA
KKR

L
SL
DD

Choice (1)
Choice (4)

15. From the choices we can say that X, U and Q can be


the first person to give the presentation in 2nd group but
V cannot be the person.
Choice (4)
16. Given R attends the Physics tuition, Q attends the
Maths tuition. Also P and U same tuition, T and V
attend same tuition and S does not attend the same
tuition as Q.
Hence S should attend a tuition with at least
2 students in it. i.e. S should either attends Physics or
Chemistry as there should be at least 2 students in
each tuition. Q should attend a tuition to which
3 people go.
Choice (4)
Quant ERPV

Persons
killed

Persons
injured

Trucks

19500

6630

12870

0.52

Bus

13500

4050

9450

0.43

Car

12000

4200

7800

0.54

Two
wheelers

21000

8400

12600

0.67

Others

9000

4050

4950

0.82

3.

The number of people who got injured by car accidents


was 7800
Choice (2)

4.

The number of persons killed in truck is 6630 and


number of persons injured in other type of accidents is
4950.
The required difference is 1680
Choice (4)

Solution for questions 14 and 15:


14. Given X gave the presentation before T, R gave the
presentation before V but after U, also W gave the
presentation after P and S but before U and X.
So, Q, U, X, T, R and V (need not be in the same order)
conducted the seminars after W. Hence W should be
giving his presentation in group 1 with P and S.
Choice (1)

Total
accidents

x = the ratio of the persons killed to the persons injured.


The required ratio is the highest for other types of
accidents.
Choice (2)

12. M plays for RCB and comes from India.

13. The player from Australia plays for DC.

The number of accidents caused because of two


28
75000 = 21000.
wheelers is
100
The accident severity index for two wheelers is 40.
i.e. for every 100 accidents, 40 persons are killed.
So for 21000 accidents 210 40 = 8400 persons are
killed.
Choice (3)

DI (Distribution)
Solutions for questions 1 and 2:

As policies mature in between 1997 and 2002 and a policy


matures on Feb 29th, the policy should mature on 29th Feb
2000.
Now the policy which matures on Jan 10th matured after
Feb 29th. Hence the policy on Jan 10th can be in 1997 or in
2002.
But from one of the clues the policy on May 21st is the last
and the Sep 17th policy is immediately before the May 21st
policy.
Hence the policies on Sep 17th and Jan 10th should mature
the same year.
The policy on August 8th is before Feb 29th.
Hence August 8th policy is for either 98 or 99.
The final arrangement is as follows.
1st

1.

The ratio of the floor areas of A1 and A2 is 1 : 4


Now the total work required to complete the work in
1
9
N 5
A1 = N (D) +
ND, where D = no. of
D =
6
4 6
24
hours
The total work required to complete the work in A
9

ND .
must be 4
24

2nd

3rd
4th
5th
6th
th
29
Feb
24th Aug' 8th Aug
10th Jan 17th Sep May 21st
2000
1997 1998/1999
2001
2001
2002
1.

The third matured policy is on 29th Feb 2000


Choice (1)

2.

In the year 2001, Atul receives money from two policies.


Choice (2)

24

LA(Linear Arrangement)
Solutions for questions 1 to 3:

Given S is in 3rd place from the left end and the positions of F, Q, G are also given.

S
UK

As two of F, Q, G are from USA and there is atleast one person between any two friends from USA, we can say that F and G
are from USA.
Also the extreme ends are occupied by friends from USA.

F
S
UK USA

USA

G
USA

USA

USA

Now the friends from UK are separated by atleast four friends. So the other friend from UK can come either in the 2nd or the
4th position from the right. But from one of the clues P is from Australia and is in between and J. Hence we can get it as.

USA

Aus

F
S
Q
UK USA Aus

G
USA UK

P
/J
J/
USA Aus USA

Now as R is adjacent to and the friends at extreme ends are of different gender, we can get the final arrangement as follows.

T
H
USA Aus

S
F
Q
UK USA Aus

G
R
USA UK

J
USA Aus USA

1.

F, G, and J stay in the USA. Therefore four female


friends stay in the USA.
Choice (4)

2.

Three friends are in between the friends from the UK


Choice (1)

3.

T is the only male friend from USA.

Choice (2)

Solution for question 4 and 5:

Given from (i) and (iv), the arrangement would be

From the (iii) clue, U and V have 2 persons in between


them.
Hence we have only one possibility.[since there is only one
person between Tarun and Qureshi]

U/V

V/U

From the (ii) clue we get the final arrangement as

S
4.

5.

U/V

V/U

Now D got the 4th rank D gets less than C. But the
condition that one pair should have same marks is not
satisfied. Hence E should get the highest marks.
So E > C > A
Now D cannot have more marks than C as D should have
marks less than 3 students, hence C > D.
As the first and the last ranked students do not have same
marks as any other student, B should get the same marks
as 'C'.
Hence all the conditions are satisfied and the final
arrangement is E > C = B> D > A.
1.

E got the highest marks.

Choice (4)

2.

B and C got the same marks.

Choice (2)

3.

The descending order is ECBDA.

Choice (1)

Solutions for questions 4 and 5:

Given at least 2 movies were released before R and there is


one movie released between S and T.
Hence R cannot be released on the 3rd or the 4th week of the
month. It can only be released on the last week.
Therefore Q is released on the 4th week. As S and T have
one movie released between them, P is released on the 2nd
week.
Hence the final order would be as S/T, P, T/S, Q, R.

Raju is 3 places away to the left of Qureshi.


Choice (4)

4.

R is released last.

Pradip is to the immediate right of Shyam.

5.

Only one movie is released before P.

Choice (3)
Choice (1)

Choice (3)
Line Graph + Table
LA (Sequencing)
Solutions for question 1:
Solutions for 1 to 3:
1.

From the clue, D got less than 3 other students hence D got
the 4th rank.
From the other clue E > C > A.
Let us assume B got the highest marks. Then the order is B
> E > C > A.

From the table we can identify that the profit/ton in 2000


is the highest for Steel. From the line graph it is clear
that the production of Steel is the highest.
Hence the profit of Steel should be the highest.
Choice (4)

25

Você também pode gostar